Download as pdf or txt
Download as pdf or txt
You are on page 1of 85

AUDITING THEORY d.

Appropriateness
PRELIM REVIEWER
(875 Multiple Choice Questions) 7. Which of the following best describes due care?
a. Reasonable infallibility
b. Tact in avoiding legal liability
SET 1 c. Requisite skill and diligence
d. Freedom from undue influence
1. Which of the following engagements is covered by the
Framework for Assurance Engagements? 8. Users of financial statements demand independent audit
a. Consulting engagements because
b. Preparation of tax returns a. Users demand assurance that fraud does not exist.
c. Independent financial statements audit b. Management may not be objective in reporting.
d. Agreed-upon procedures c. Users expect auditors to correct management errors.
d. Management relies on the auditor to improve internal
2. Independent auditing can best be described as a control.
a. Subset or branch of accounting.
b. Professional activity that measures and communicates 9. An audit of financial statements is conducted to determine
financial accounting data. if the
c. A discipline or professional activity that attests to a. Organization is operating efficiently and effectively
the fair presentation of financial statements. b. Client's internal control is functioning as intended.
d. Regulatory activity that prevents the issuance of c. Auditee is following specific procedures or rules set
improper financial information. down by some higher authority
d. Overall financial statements are stated in
3. An auditor should not render a report on: accordance with an identified financial reporting
a. Client internal control. framework.
b. Management performance.
c. The achievability of forecasts. 10. An audit designed to determine the extent to which the desired
d. Quarterly financial information. results of an activity established by the legislative or other
authorizing body are being achieved.
4. The assumption underlying an audit of financial statements a. Economy audit
is that they will be used by b. Efficiency audit
a. Different groups for different purposes. c. Program results audit
b. The general public in making investment decisions. d. Financial-related audit
c. The board of directors as basis of declaring cash
dividends. 11. Which of the following methods is most commonly used to
d. The regulatory agencies to verify information that is reduce information risk?
relevant to their supervisory functions. a. Allow users to verify information.
b. Allow all users to prepare the statements.
5. On every audit engagement, the CPA should comply with c. Users share information risk with management.
applicable PSAs d. Have the financial statements audited.
a. Without exception.
b. Except in examinations that result in a qualified report. 12. Which of the following best describes the primary purpose of
c. Except in examinations of interim financial statements. audit procedures?
d. Except in circumstances where the CPA is associated a. To detect errors or fraud
with unaudited financial statements. b. To comply with generally accounting principles
c. To gather sufficient, appropriate evidence
6. It refers to the attribute or quality of audit evidence. d. To verify the accuracy of account balances
a. Sufficiency
b. Relevance 13. It is essential that users regard CPA firms as:
c. Reliability a. Competent.

AT – Prelim Rev (875 MCQs) Red Sirug Page 1 of 85


b. Unbiased. 19. When the CPA is not independent, the CPA is precluded from
c. Technically proficient. issuing a
d. All of the above a. Review report.
b. Compilation report.
14. Which of the following is least likely an application of c. Tax planning report.
maintaining an attitude of professional skepticism? d. Management advisory report.
a. The auditor is alert to audit evidence that contradicts or
brings into question the reliability of documents or 20. The risk that the auditor will fail to uncover a material
management representations. misstatement is eliminated
b. In planning and performing an audit, the auditor a. If client has effective internal control.
assumes that management is dishonest. b. If client follows PFRS.
c. The auditor makes a critical assessment, with a c. When the auditor has complied with PSA.
questioning mind, of the validity of audit evidence d. Under no circumstances.
obtained.
d. The auditor does not consider representations from 21. No matter how competent a CPA may be, his opinion on
management as substitute for obtaining sufficient financial statements will be of little value to those who rely on
appropriate audit evidence to be able to draw reasonable him unless he
conclusions on which to base the audit opinion. a. Issues an unqualified opinion
b. Maintains his independence.
15. Which of the following is not one of the concepts in the c. Maintains a program of continuing education
framework of auditing theory? d. Serves his clients with professional concern of their best
a. Evidence interests.
b. Ethical conduct
c. Fair presentation 22. An audit can have a significant effect on
d. Conflict of interest a. Business risk.
b. Information risk.
16. To raise the standards of the profession, the Philippine c. The risk-free interest rate.
Accountancy Act of 2004 requires that the CPA profession be d. All of these.
integrated under the rules to be established by
a. The PICPA. 23. In government auditing, the three elements of expanded
b. The Board of Accountancy. scope auditing are
c. The Professional Regulation Commission. a. Goal analysis, audits of operations, audit of systems.
d. The Government Association of CPAs. b. Financial and compliance, economy and efficiency,
program results.
17. Management assertions are c. Pre-audit, post-audit, internal audit.
a. Stated in the footnotes to the financial statements. d. National government audit, local government audit,
b. Provided to the auditor in the assertions letter, but are not corporation audit.
disclosed in the financial statements of the entity.
c. Implied or express representations about the 24. Which of the following types of audits are most similar?
accounts in the financial statements. a. Operational audits and compliance audit.
d. Explicitly expressed representations about the financial b. Internal audits and independent financial statement audits.
statements. c. Independent financial statement audits and operational
audits.
18. In performing a financial statement audit, which of the d. Compliance audits and independent financial
following would an auditor least likely consider? statement audits.
a. Internal control.
b. Compliance with GAAP. 25. If it is probable that the judgment of a reasonable person
c. Fairness of the financial statement amounts. would have been changed or influenced by the omission or
d. Quality of management’s business decisions. misstatement of information, then the information is
a. Relevant.

AT – Prelim Rev (875 MCQs) Red Sirug Page 2 of 85


b. Pervasive. 32. Which of the following is not an assurance that the auditors
c. Material. give to the parties who rely on the financial statements?
d. Significant. a. Auditors give assurance that the financial statements
are accurate.
26. Which of the following is not among the factors that result to b. Auditors know how the amounts and disclosures in the
limitations of audit? financial statements were produced.
a. Use of testing c. Auditors gathered enough evidence to provide a
b. Human error reasonable basis for forming an opinion.
c. Going concern problem of the assurance client d. If the evidence allows the auditors to do so, auditors give
d. Evidence is basically persuasive rather than conclusive assurance in the form of opinion, as to whether the
financial statements taken as a whole are fairly presented
27. Which of the following services is not within the area of public in conformity with GAAP.
accounting?
a. Attest function 33. Auditing standards differ from auditing procedures in that
b. Taxation service procedures relate to
c. Write-up work for one client a. Audit judgments.
d. Management advisory services b. Audit principles.
c. Acts to be performed.
28. An audit committee must be comprised of outside directors. d. Measures of performance.
Which of the following is considered an outside director?
a. A consultant to the company. 34. Choose one of the following which would describe best the
b. A member of company management. phrase "generally accepted auditing standards":
c. The company's independent auditor. a. They identify the policies and procedures for the conduct
d. A retired executive from another company. of the audit.
b. They define the nature and extent of the auditor's
29. "The auditor would ordinarily expect to find evidence to responsibilities.
support management representations and not assume that c. They provide guidance to the auditor with respect to
they are necessarily correct". This is an example of: planning the audit and writing the audit report.
a. Due diligence. d. They set forth a measure a measure of the quality of
b. Unprofessional behavior. the performance of audit procedures.
c. An attitude of professional skepticism.
d. A rule in the code of Professional Conduct. 35. Auditing includes both a(an)
a. Evaluation process and a reporting process.
30. The audit process is b. Investigative process and a reporting process.
a. Regulated by the PICPA c. Documentation process and an evaluation process.
b. Performed only by CPAs d. Documentation process and a reporting process.
c. The only service a CPA is allowed to perform by law
d. A special application of the scientific method of 36. The audit committee of the board of directors of a company
inquiry is responsible for
a. Hiring the auditor
31. In determining the scope and nature of services to be b. The audit workpapers
performed in public practice, a CPA firm should c. Preparing the financial statements
a. Have in place internal control procedures d. Independence and obtaining evidence
b. Only perform accounting related services
c. Require independence for all services performed 37. Which of the following types of audits would be intended to
d. Determine that the performance of all services is determine whether a governmental entity is following sound
consistent with the firm's members' role as procurement practices?
professionals. a. Program audit
b. Operational audit
c. Compliance audit

AT – Prelim Rev (875 MCQs) Red Sirug Page 3 of 85


d. Financial statement audit c. Inquiry and analytical procedures
d. Analytical procedures and substantive tests of details of
38. Which of the following is the authoritative body designated to transactions and account balances
promulgate auditing standards?
a. BOA 44. The BOA vice-chairman shall be
b. PICPA a. Appointed by the PRC
c. FRSC b. Elected by BOA members
d. AASC c. Appointed by the President of the Philippines
d. Appointed by the PRC upon recommendation by the
39. Which of the following statements is true? PICPA
a. An independent auditor must be a CPA.
b. Compliance audits are conducted to determine 45. The term of BOA vice-chairman is
adherence to rules and regulations set by the auditor. a. One year.
c. An audit, if properly conducted, ensures that fraud is b. Three years subject to renewal.
prevented. c. At the discretion of BOA members.
d. After conducting an audit and release of the auditor’s d. Same with the term of BOA chairman.
report, the primary responsibility on the fairness of the
financial statements is shifted to the auditor. 46. An audit designed to evaluate the efficiency and effectiveness
of an organization or some or part thereof would not come
40. The Philippine Standards on Assurance Engagements under the title of
(PSAEs) are to be applied in a. Performance audit.
a. The audit or review of historical financial information. b. Compliance audit.
b. Assurance engagements dealing with historical financial c. Management audit.
information.
d. Operational audit.
c. Compilation engagements and agreements to apply
agreed-upon procedures to information.
d. Assurance engagements dealing with subject matters 47. In financial statement audits, the audit should be conducted
other than historical financial information. in accordance with
a. PAS / PFRS
41. The Philippine Standards on Quality Control (PSQCs) are to b. Generally accepted auditing standards
be applied to c. Philippine Standards on Auditing
a. Assurance engagements only. d. Code of Ethics for CPAs in the Philippines
b. Review engagements only.
c. Compilation and review engagements only. 48. Which of the following statements does not properly describe
d. All services that fall under the AASC's engagement an element of the theoretical framework of auditing?
standards. a. Remoteness of users.
b. An audit benefits the public.
42. These statements are issued by the AASC to provide c. The data to be audited are verifiable.
interpretive guidance and practical assistance to auditors in d. Auditor should maintain independence with respect to
the implementation of PSAs and to promote good practice. the audit client.
a. PREPSs
b. PAEPs 49. The statement that the reviewer "is not aware of any material
c. PAPSs modification that should be made to the financial statements in
d. PRSPSs order for them to be in conformity with GAAP" is known as:
a. Positive assurance.
43. For the purpose of expressing negative assurance in the b. Reasonable assurance.
review report, the practitioner should obtain sufficient c. Negative assurance.
appropriate evidence primarily through d. Negligent performance.
a. Inquiry and confirmation
b. Confirmation and tests of controls 50. An independent audit aids in the communication of economic
data because the audit

AT – Prelim Rev (875 MCQs) Red Sirug Page 4 of 85


a. Lends credibility to the financial statements. judge it necessary to depart from a PSA, even though
b. Confirms the accuracy of management's financial such a departure may result to more effective
representation. achievement of the objective of an audit.
c. Guarantees that financial data are fairly presented.
d. Assures the readers of financial statements that any 56. In the case of an audit of financial statements, which of
fraudulent activity has been corrected. the following would not be a valid criterion in ascertaining
the degree of correspondence between assertions and
51. The purpose of an audit of financial statements is to established criteria?
a. Obtain an absolute level of assurance that the financial a. International Accounting Standards
statements as a whole are free from material b. Philippine Accounting Standards
misstatement. c. Philippine Standards on Auditing
b. Relieve management or those charged with governance d. Philippine Financial Reporting Standards
of the responsibility for the preparation and presentation
of the financial statements in accordance with the 57. Which of the following statements about independent
applicable financial reporting framework. financial statement audit is correct?
c. Enhance the degree of confidence of intended users in a. The audit of financial statements relieves management
the financial statements. of its responsibilities for die financial statements.
d. Assure the future viability of the entity by expressing an b. An audit is designed to provide limited assurance that
opinion on the entity's financial statements. the financial statements taken as a whole are free from
material misstatement.
52. Inherent limitations in an audit stem from the following factors c. The auditor's opinion is not an assurance as to
except the future viability of the entity as well as the
a. Use of testing. effectiveness and efficiency with which
b. Incompetence of the auditor. management has conducted the affairs of the entity.
c. Internal control limitation. d. The procedures required to conduct an audit in
accordance with PSAs should be determined by the
d. Most audit evidence is persuasive rather than conclusive.
client who engaged the services of the auditor.
53. Auditing services may include which of the following? 58. The reason an independent auditor gathers evidence is to
a. Attesting to financial statements a. Detect fraud
b. Evaluation of a division’s performance for management b. Evaluate management
c. Examination of the economy and efficiency of c. Evaluate internal controls
governmental operations d. Form an opinion on the financial statements
d. All of the above
59. The auditor communicates the results of his or her work
54. The review of unaudited financial statements consists of: through the medium of the
a. Inquiry of management and analytical procedures. a. Engagement letter.
b. Inquiry of management and documentation of internal b. Audit report.
controls. c. Management letter.
c. Analytical procedures and compliance with laws and d. Audited financial statements.
regulations.
d. Internal control evaluation and management 60. The expertise that distinguishes auditors from accountants is
representation. in the
a. Ability to interpret PAS/PFRS.
55. The Philippine Standards on Auditing issued by AASC: b. Accumulation and interpretation of evidence.
a. Apply to independent examination of financial c. Ability to interpret generally accepted accounting
statements of any entity when such an examination is principles.
conducted for the purpose of expressing an opinion. d. Requirement to possess education beyond the
b. Need to be applied on all audit related. bachelor's degree.
c. Must not apply to other related activities of auditors.
d. Require that in no circumstances would an auditor may

AT – Prelim Rev (875 MCQs) Red Sirug Page 5 of 85


61. The market for auditing services is driven by a. Mastery of a particular intellectual skill, acquired by
a. The regulatory authority of the Securities and Exchange training and education.
Commission. b. Adherence by its members to a common code of values
b. A demand by external users of financial statements. and conduct established by its administrating body,
c. Pronouncements issued by the Auditing and Assurance including maintaining an outlook which is essentially
Standards Council. objective.
d. Congress. c. Acceptance of a duty to society as a whole.
d. All of these.
62. An independent audit is important to readers of financial
statements because it 67. Professional judgment:
a. Provides a measure of management's stewardship a. Is not used in making decisions about materiality and
function audit risk.
b. Objectively examines and reports on management's b. Is necessary in the evaluation of management's
financial statements judgments in applying the entity's applicable
c. Reports on the accuracy of information in the financial financial reporting framework.
statements c. Should be exercised in planning and performing an audit
d. Measures and communicates the financial data included of financial statements but need not be documented.
in financial statements d. Can be used as the justification for the decisions made by
the auditor that are not supported by the facts and
63. Independent auditing can best be described as circumstances of the engagement.
a. A branch of accounting.
b. A regulatory function that prevents the issuance of 68. The auditor's judgment concerning the overall fairness of
improper financial information. presentation of financial position, results of operation, and
c. A professional activity that measures and communi- changes in cash flow is applied within the framework of
cates financial and business data. a. Generally accepted auditing standards which include
d. A discipline which attests to the results of the concept of materiality
accounting and other functional operations and data. b. Generally accepted accounting principles.
c. Philippine Financial Reporting Standards
64. The primary purpose of an independent financial statement d. Quality control
audit is to
a. Provide a basis for assessing management's 69. One of the conditions that give rise to a demand for an
performance external audit of financial statements is expertise. Which
b. Comply with government regulatory requirements of the following best describes the meaning of expertise as
c. Assure management that the financial statements are used in this context?
unbiased and free from material error a. Auditors usually rely on the work of an expert as
d. Provide users with an unbiased opinion about a basis for evaluating some assertions embodied in
the fairness of information reported in the financial the financial statements.
statements b. Users usually lack the necessary expertise to verify
the reliability of the financial information.
65. The criteria for evaluating quantitative information vary. For c. As experts, auditors are expected to detect all material
example, in the case of an independent audit of misstatements in the financial statements.
financial statements by CPA firms, the criteria are d. The readers of the financial statements must possess
usually the: the necessary expertise to be able to understand the
a. Philippine Standards on Auditing financial statements.
b. Philippine Financial Reporting Standards
c. National Internal Revenue Code 70. Regulation of the accounting profession include:
d. Securities and Exchange Commission Regulations a. Public Regulation as provided for in the Philippine
Accountancy Act of 2004.
66. A profession is distinguished by certain characteristics b. Regulation within the profession, through the
including: implementation of the Code of Ethics.

AT – Prelim Rev (875 MCQs) Red Sirug Page 6 of 85


c. Regulation within the firm, through the implementation of partnership name for a period of not more than ___ years after
a system of quality control. becoming a sole proprietor.
d. All of these. a. 1
b. 2
71. This body is created to assist the BOA in the attainment of the c. 3
objective to continually upgrade accounting education in the d. 4
Philippines, thus making Filipino CPAs globally competitive.
a. PICPA CPE Council 78. The death, or disability of an individual CPA and/or the
b. Education Technical Council dissolution and liquidation of a firm or partnership of CPAs
c. Accounting Teachers' Conference shall be reported to the BOA not later than ___ days from the
d. Association of CPAs in Education date of such death, dissolution or liquidation.
a. 15
72. Which of the following is not a requirement for a person b. 30
applying for the CPA Board Examinations? c. 60
a. Good moral character d. 90
b. Degree of BS Accountancy
c. Natural-born Filipino citizen 79. It refers to the inculcation, assimilation and acquisition of
d. No conviction of any criminal offense involving moral knowledge, skills, proficiency and ethical and moral values,
turpitude. after the .initial registration of a professional that raise and
enhance the professional's technical skills and competence.
73. Its function is to conduct a quality review on applicants for a. Professional Development
registration to practice public accountancy and render a report b. Continuing Professional Education
which shall be attached to the application for registration. c. Continuing Professional Development
a. Quality Review Board d. Professional Growth and Development
b. Quality Review Group
c. Board of Quality Reviewers 80. The following statements relate to CPE credit units. Which is
d. Quality Review Committee incorrect?
a. One credit hour of CPE program, activity or source shall
74. Which of the following is the accredited national professional be equivalent to one (1) credit unit.
organization of CPAs? b. Any excess credit units in one year may be carried over
a. IFAC to the succeeding years within the three-year period.
b. ASC c. The total CPE credit units for registered accounting
c. AASB professionals shall be sixty (60) credit units for three (3)
d. PICPA years, provided that a minimum of fifteen (15) credit units
shall be earned in each year.
75. The ETC shall be composed of d. Excess credit units earned may be carried over to the
a. 7 members. next three-year period including credit units earned
b. 8 members. for doctoral and master's degrees.
c. 15 members
d. 14 members 81. The following statements relate to the exemption from CPE
requirements. Which is false?
76. Which of the following does not have representations in the a. A registered professional shall be permanently exempted
FRSC? from CPE requirements upon reaching the age of 65
a. PICPA years old.
b. BIR b. A registered professional shall be permanently
c. BOI exempted from CPE requirements upon reaching the
d. BOA age of 60 years old.
c. A registered professional who is working or practicing
77. A partner surviving the death or withdrawal of all the other his/her profession abroad shall be temporarily exempted
partners in a partnership may continue to practice under the from compliance with CPE requirements during the

AT – Prelim Rev (875 MCQs) Red Sirug Page 7 of 85


period of his/her stay abroad provided that he/she has a. Negligence in the performance of duties, or lack of
been out of the country for at least two years professional competence.
immediately prior to the date of renewal. b. Intolerance of violations of the Philippine
d. A registered professional who is furthering his/her studies Accountancy Act.
abroad shall be temporarily exempted from compliance c. Final judgment of crimes involving moral turpitude.
with CPE requirements during the period of his/her stay d. Rigging of the CPA licensure examination results.
abroad provided that he/she has been out of the country
for at least two years immediately prior to the date of 87. The Professional Regulatory Board of Accountancy consists of:
renewal. a. 1 Chairman and 4 members.
b. 1 Chairman and 5 members.
82. Which of the following statements concerning a CPA's c. 1 Chairman and 6 members.
disclosure of confidential client information is ordinarily correct? d. 1 Chairman and 7 members.
a. Disclosure should not be made even if such disclosure
will protect the CPA's professional interests in legal 88. The Chairman and the members of the BOA shall be
proceedings. appointed by the:
b. Disclosure should be made only if there is a legal, or a. Commissioner of the SEC.
professional duty to make the disclosure. b. President of the PICPA.
c. Disclosure may be made to any party on consent of c. President of the Republic of the Philippines.
the client. d. Retiring chairman of the Board of Accountancy.
d. Disclosure may be made to any government agency
without subpoena. . 89. Members of the Professional Regulatory Board of
Accountancy shall serve for:
83. This refers to those persons who hold a valid certificate issued a. Three years, subject to renewal.
by the Board of Accountancy. b. Seven years, not subject to renewal.
a. Senior accounting practitioner. c. Three years, subject to reappointment.
b. Professional accountant. d. Three years, not subject to reappointment.
c. Professional accountant in public practice.
d. Audit associate. 90. Which of the following is not mentioned as a power of the
Board?
84. The principle of professional behavior requires a CPA to a. To monitor the conditions affecting the practice of
a. Be straightforward and honest in performing professional accountancy.
services. b. To supervise the registration, licensure and practice of
b. Perform professional services with due care, competence accountancy.
and diligence. c. To sentence to imprisonment CPAs who have been
c. Be fair and should not allow prejudice or bias, conflict of found guilty of violations of the Accountancy Act.
interest or influence of others to override objectivity. d. To issue, suspend, revoke, or reinstate the Certificate of
d. Act in a manner consistent with the good reputation Registration for the practice of the accountancy
of the profession and refrain from any conduct which profession.
might bring discredit to the profession.
91. Which of the following is the sectoral organization for CPAs in
85. This fundamental principle requires a CPA not to use or Public Practice?
disclose information acquired during the course of performing a. ACPACI
professional services without proper and specific authority. b. ACPAPP
a. Objectivity. c. ACPAE
b. Professional Behavior. d. GACPA
c. Professional Competence and Due care.
d. Confidentiality. 92. This is the standard-setting body that replaces the Accounting
Standards Council (ASC) and is tasked to develop and issue
86. Which of the following is not a ground for suspension or standards which will represent generally accepted accounting
removal of members of BOA? principles in the Philippines:

AT – Prelim Rev (875 MCQs) Red Sirug Page 8 of 85


a. Accounting Standards Board. expiry date, due for periodic renewal, duly signed by the
b. Financial Reporting Standards Committee. Chairperson of the PRC and issued by the PRC to a
c. Financial Reporting Standards Council. registered CPA upon payment of the registration fees:
d. Auditing and Assurance Standards Council. a. Certificate of registration
b. Certificate of accreditation
93. This standard-setting body replaces the Auditing Standards c. Professional identification card
and Practices Council (ASPC) and is tasked to develop and d. Professional tax receipt
issue Philippine Standards on Auditing and related
Interpretations: 99. A professional identification card has a validity of:
a. Accounting Standards Board. a. Six years
b. Financial Reporting Standards Committee. b. Five years
c. Financial Reporting Standards Council. c. Four years
d. Auditing and Assurance Standards Council. d. Three years

94. The Chairpersons of the FRSC and the AASC shall be 100. Foreign CPAs
appointed by: a. Are allowed to practice accountancy in the Philippines
a. The Philippine Institute of Certified Public Accountants. without restriction.
b. The President of the Republic of the Philippines. b. Are never allowed to practice accountancy in the
c. The Professional Regulatory Board of Accountancy. Philippines.
d. The Professional Regulation Commission. c. May be allowed to practice accountancy in the Philippines,
but only in areas outside public practice.
95. A candidate successfully passes the Board exams if he/she d. May be allowed to practice accountancy in the
obtains Philippines, subject to the rules and regulations on
a. A general average of sixty-five percent (65%). reciprocity.
b. A general average of seventy-five percent (75%).
c. A general average of seventy-five percent (75%) in at SET 2
least a majority of the subjects given in the Board exams.
d. A general average of seventy-five percent (75%), with 1. These are independent professional services that improve the
no grades lower than sixty-five percent (65%) in any quality of information for decision-making.
given subject. a. Audit services
b. Assurance services
96. In the event a candidate obtains the rating of seventy-five c. Attestation services
percent (75%) and above in at least a majority of subjects as d. Management consultancy
provided for in RA9298, and one subject has a rating of 64%,
he shall be considered as: 2. Which of the following types of audits is performed most
a. A failed candidate. frequently by CPAs on a fee basis and for more than one
b. A conditioned candidate. client?
c. A successful passer of the CPA Board exams. a. Government audits
d. Cannot be determined without additional information b. Compliance audits
c. Internal audits
97. A certificate under seal, bearing a registration number, issued d. Financial statement audits
to an individual, by the PRC, upon recommendation by the
BOA, signifying that the individual has complied with all the 3. The independent auditor lends credibility to client
legal and procedural requirements for such issuance: financial statements by
a. Certificate of accreditation a. Stating in the auditor’s management letter that the
b. Certificate of registration examination was made in accordance with PSAs
c. Certificate of quality review b. Attaching an auditor’s opinion to the client’s
d. Certificate of identification financial statements.
c. Testifying under oath about client financial statements.
98. This bears the registration number, date of issuance with an d. Maintaining clear-cut distinctions between

AT – Prelim Rev (875 MCQs) Red Sirug Page 9 of 85


management representations and auditor’s 10. The practitioner makes a critical assessment of validity of
representations. audit evidence, with a questioning mind and being alert to
conditions which may indicate possible misstatement due to
4. Internal auditing relates to an error or fraud.
a. Audit which is performed by a practitioner as an a. Objectivity and integrity
independent contractor. b. Professional skepticism
b. Audit which is incidentally concerned with the c. Independence is mental attitude
detection and prevention of fraud. d. Sufficient appropriate evidence
c. Audit which serves the needs of management
d. Audit wherein the auditor should be independent of 11. In the auditing environment, failure to meet the PSAs is
management both in fact and in mental attitude often
a. An accepted practice.
5. Independent auditors cannot be totally independent b. A suggestion of negligence.
a. Because they don’t audit financial statements c. Conclusive evidence of negligence.
b. Since they do not possess the CPA license d. Tantamount to criminal behavior.
c. As long as employer-employee relationship exists
d. Unless their immediate supervisor is a CPA 12. The following phrases relate to the definition of auditing.
Which one is incorrect?
6. The Philippine Standards on Auditing issued by the Auditing a. Systematic process
and Assurance Standards Council (AASC) b. Objectively obtaining and obtaining evidence
a. Must be followed in all situations. c. Assertions about economic actions and events
b. Are interpretations of generally accepted auditing d. Degree of correspondence between assertions
standards and GAAS
c. Are optional guidelines which an auditor may choose to
follow or not follow when conducting an audit. 13. Non-assurance engagements include all of the following
d. Are the equivalent of laws for audit practitioners. except
a. Compliance audit.
7. The auditor's best defense when material misstatements in b. Management consulting.
the financial statements are not uncovered in the audit is that c. Agreed-upon procedures.
a. The financial statements are client's responsibility. d. Preparation of tax returns where no conclusion is
b. The audit was conducted in accordance with PSAs expressed.
c. Client is guilty of contributory negligence.
d. The audit was conducted in accordance with generally 14. The subject matter of an assurance engagement can take the
accepted accounting principles. following forms except
a. The entity’s internal control.
8. The auditor considers materiality from: b. Historical or prospective financial statements.
a. AASC perspective c. Evaluation of a capital investment proposal.
b. Client perspective d. Performance of an entity that could indicate efficiency
c. Independence perspective and effectiveness.
d. Reasonable user perspective
15. In performing an assurance engagement, a professional
9. Which of the following is more difficult to evaluate accountant typically
objectively? a. Provides management consulting advice.
a. Compliance with government regulations b. Supplies litigation support services.
b. Efficiency and effectiveness of operations c. Assesses control risk at a low level.
c. All of the above are equally difficult d. Expresses a conclusion about an assertion.
d. Presentation of financial statements in accordance
with the PFRS 16. Which of the following is not an assurance engagement?
a. Risk Assessment Service
b. Information System Reliability Service

AT – Prelim Rev (875 MCQs) Red Sirug Page 10 of 85


c. Business Performance Measurement d. Inform any known third party users of the inappropriate
d. Management Consulting Service use of his/her name in connection with the information.

17. The three commonly-sought types of assurance services are 23. A CPA should not submit unaudited financial statements of a
a. Audits, reviews, and compilations. non-public entity to a client or others unless, as a minimum,
b. Audits, reviews, and other assurance services. the CPA complies with the provisions applicable to
c. Audits, compilations, and other assurance services. a. Audit engagements.
d. Reviews, compilations, and other assurance services. b. Review engagements.
c. Assurance engagements.
18. Which of the following statements is true? d. Compilation engagements.
a. Assurance engagements do not require independence.
b. The term “auditor” is broader in scope compared to the 24. You own Dude, Inc., which manufactures wooden tables. You
term “practitioner.” need to hire some accountants to prepare your monthly
c. Assurance engagements performed by professional financial statements. The preparation of your financial
accountants are intended to enhance the credibility statements is referred to as a(n)
of information. a. Audit.
d. The degree or level of assurance that may be provided by b. Review.
the practitioner is inversely related to the scope of c. Compilation.
procedures performed and their results. d. Special report.

19. Which of the following forms may be the subject matter of 25. Distribution of a report is always restricted when
assurance engagements? a. Negative assurance is given.
a. Behavior b. A review has been performed.
b. Historical financial information c. There is a positive expression of opinion.
c. Systems and processes d. Agreed-upon procedures have been performed.
d. All of the above
26. Which of the following methods is most commonly used to
20. Engagement risk is influenced by the risks associated with the reduce information risk?
following except a. Allow all users to prepare the statements.
a. Nature and form of the subject matter. b. Allow users to verify information.
b. Nature and form of the criteria applied to the subject c. Have the financial statements audited.
matter. d. Users share information risk with management.
c. Unreasonably low professional fee.
d. Nature and extent of the process used to collect and 27. The phrase in our opinion in the auditor’s report is intended to
evaluate evidence. inform users that auditors
a. Guarantee fair presentation of the financial statements.
21. Absolute assurance is not attainable as a result of such b. Act as insurers of the accuracy of the financial statements.
factors as: c. Certify the material presented in the statements by
a. The use of judgment. management.
b. The use of selective testing. d. Based their conclusions about the statements on
c. The inherent limitations of control systems. professional judgment.
d. All of the above
28. The need for assurance services arises because:
22. If the auditor learns that an entity is inappropriately using the a. There is a potential bias in providing information.
auditor’s name in association with financial information, he/she b. Economic transactions are less complex than they were a
should: decade ago.
a. Remain silent. c. There is a consonance of interests of the preparer and
b. Seek legal advice if necessary. the user of the financial statements.
c. Require management to cease doing so. d. Most users today have access to the system that
generates the financial statements they use.

AT – Prelim Rev (875 MCQs) Red Sirug Page 11 of 85


a. Reprimand
29. This tax service includes the determination of the tax b. Fine or imprisonment
consequences of planned or potential transactions, followed c. Revocation of CPA certificate
by making suggestions on the most desirable course of action d. Suspension of CPA certificate
such as to legally minimize the tax liability while achieving the
client’s objectives. 36. Which of the following is an incorrect statement relating to the
a. Tax shelter theoretical framework of auditing?
b. Tax evasion a. Effective internal control structure reduces the probability
c. Tax planning of fraud or irregularities in an organization.
d. Tax compliance b. Application of generally accepted accounting principles
results in a fair presentation of financial statements.
30. Which of the following characteristics is not considered c. When examining financial data for the purpose of
necessary in determining whether the criteria are suitable? expressing an independent opinion thereon, the auditor
a. Reliability acts exclusively in the capacity of an auditor.
b. Relevance d. In collecting evidence, auditors should maintain an
c. Neutrality attitude of trust about their clients' assertions.
d. Sufficiency
37. The market for auditing services is driven by
31. In an assurance engagement, this refers to the information a. Congress at the state level
obtained by the practitioner in arriving at the conclusions on b. A demand by external users of financial statements
which the conclusion is based. c. The regulatory authority of the Securities and Exchange
a. Criteria Commission
b. Assertions d. Pronouncements issued by the Auditing Standards and
c. Evidence Practices Council
d. Generally accepted auditing standards
38. Which of the following statements reflects an auditor's
32. Compilation is an example of which one of the following types responsibility for detecting fraud and error?
of services? a. An auditor is responsible for detecting employee errors
a. Auditing and simple fraud, but not for discovering fraud involving
b. Review employee collusion or management override.
c. Consulting b. An auditor should design the audit to provide
d. Accounting reasonable assurance of detecting errors and fraud
that are material to the financial statements.
33. The Framework of Philippine Standards on Auditing c. An auditor should plan the audit to detect errors and fraud
distinguishes audits from related services. Related services that are caused by departures from the applicable FRF.
include all of the following, except d. An auditor is not responsible for detecting errors and
a. Reviews. fraud unless the application of PSAs and PAPs would
b. Compilations. result in such detection.
c. Agreed-upon procedures.
d. Management consulting. 39. The adequacy of disclosures in the financial statements and
footnotes is the primary responsibility of the:
34. Assurance services involve which of the following? a. Client
a. Relevance as well as reliability. b. Auditor in charge of field work
b. Nonfinancial information as well as traditional financial c. Partner assigned to engagement
statements. d. Staff member who drafted the statements
c. Electronic databases as well as printed reports.
d. All of the above. 40. Generally accepted auditing standards do not require auditors
of financial statements to
35. Which of the following is not one of the penalties that can be a. Assess the risk of occurrence of errors and fraud
imposed by the BOA?

AT – Prelim Rev (875 MCQs) Red Sirug Page 12 of 85


b. Design audits to provide reasonable assurance of c. Must be of good moral character and must not have been
detecting errors and frauds convicted of crimes involving moral turpitude.
c. Report all the findings of errors and fraud to police d. Must be a duly registered CPA with more than ten (10)
authorities years of work experience in any scope of practice of
d. Understand the nature or errors and fraud accountancy.

41. The risk that the auditor's own work will lead to the decision 46. Which statement is incorrect regarding the term of office of the
that material misstatements do not exist in the financial chairman and the members of the Board of Accountancy
statements, when in fact such misstatements do exist, is (BOA)?
a. Control risk a. The Chairman and members of the Board shall hold
b. Detection risk office for a term of three years.
c. Inherent risk b. A person may serve the BOA for not more than twelve
d. Audit risk years.
c. A member of the BOA may continuously serve office
42. With respect to the concept of materiality, which one of the for more than nine years.
following statements is correct? d. No person who has served two (2) successive complete
a. Materiality is a matter of professional judgment. terms shall be eligible for reappointment until the lapse of
b. Materiality is determined by reference to AASC's one (1) year.
guidelines.
c. Materiality depends on the nature of a transaction rather 47. The BOA shall submit to the PRC the ratings obtained by each
than the peso amount of the transaction. candidate within how many calendar days after the
d. Materiality depends only on the peso amount of an item examination?
relative to other items in the financial statements. a. 5 days
b. 10 days
43. The following statements relate to the Board of Accountancy. c. 15 days
Which statement is correct? d. 30 days
a. The Board consists of a Chairman and six members.
b. The chairman and members are appointed by the 48. Which of the following is not one of the grounds for
President of the Philippines upon recommendation of proceedings against a CPA?
PICPA. a. Insanity.
c. Majority of the board members shall as much as possible b. Immoral or dishonorable conduct.
be in public practice. c. Gross negligence or incompetence in the practice of his
d. The Professional Regulation Commission may remove profession.
from the Board any member whose certificate to practice d. Engaging in public practice while being employed in
has been removed or suspended. a private enterprise.

44. The APO shall submit its nominations with complete 49. The primary duty to enforce the provisions of RA 9298 and its
documentation to the Commission not later than ______ prior IRR rests with
to the expiry of the term of an incumbent chairman or member. a. The PRC
a. 30 days b. The BOA
b. 60 days c. The AASC
c. 90 days d. The PRC and BOA
d. 120 days
50. The PICPA shall renew its Certificate of Accreditation once
45. A member of the BOA shall, at the time of his/her appointment, every how many years after the date of the Resolution
possess the following qualifications, except granting the petition for re-accreditation and the issuance of
a. Must be a natural-born citizen and resident of the the said certificate upon submission of the requirements?
Philippines. a. 2 years
b. Must not be a director or officer of the APO at the time of b. 3 years
his/her appointment. c. 4 years

AT – Prelim Rev (875 MCQs) Red Sirug Page 13 of 85


d. 6 years
56. In conducting an appraisal of the economy and efficiency with
51. The practice of accountancy includes the following except: which company resources are used, an internal auditor's
a. Serving as audit examiner for the Commission on Audit. responsibility is to:
b. Working as a Dean of a College that grants a degree of a. Verify the accuracy of asset valuation.
BS Accountancy. b. Verify the existence of assets.
c. Being appointed as the marketing manager of a c. Determine whether operating standards have been
business enterprise. established.
d. Provision of assurance services to more than one client d. Review the reliability of operating information.
and on a fee basis.
57. Because an external auditor is paid a fee by a client
52. Which of the following is one of the functions of Quality company, he
Review Committee: a. Is absolutely independent and may conduct an audit.
a. To promulgate accounting and auditing standards that will b. May be sufficiently independent to conduct an audit.
be generally accepted in the Philippines. c. Is never considered to be independent.
b. To conduct a review on applicants for registration to d. Must receive approval of the Securities and Exchange
practice public accountancy and render a report Commission before conducting an audit.
which shall be attached to the application for
registration. 58. Which of the following is a conceptual difference between the
c. To adopt a Code of Ethics for the practice of accountancy. attestation standards and generally accepted auditing
d. To evaluate periodically the performance of educational standards?
institutions offering accountancy education. a. The attestation standards do not permit an attest
engagement to be part of a business acquisition study or
53. No matter competent a CPA may be, his opinion on a feasibility study.
financial statements will be of little value to those who rely b. The attestation standards provide a framework for
upon him unless he: the attest function beyond historical financial
a. Maintains his independence. statements.
b. Maintains professional competence. c. The requirement that the practitioner be independent in
c. Maintains a program of continuing education. mental attitude is omitted from the attestation standard.
d. Serves his clients with professional concern. d. None of the standards of fieldwork in generally accepted
auditing standards are included in the attestation
54. Which of the following statements about independent financial standards.
statement audit is incorrect?
a. The term "scope of the audit" refers to audit procedures 59. The exercise of due professional care requires that an auditor
deemed necessary in the circumstances to achieve the a. Uses error-free judgment.
objective of the audit. b. Considers internal control, including tests of controls.
b. The auditor's opinion enhances the credibility of the c. Be responsible for fulfilling his or her duties
financial statements. diligently and carefully.
c. The phrase used to express the auditor's opinion is d. Examines all corroborating evidence available.
"present fairly, in all material respects".
d. The risk that the auditor will fail to uncover material 60. The objectives of the Philippine accountancy Act of 2004 are
misstatement is eliminated when the auditor the following except:
conducts the audit in accordance with PSAs. a. The standardization and regulation of accounting
education.
55. For what reason does an independent auditor gather b. The examination for registration of certified public
evidence? accountants.
a. To assess management performance c. The integration of the accounting profession.
b. To detect fraud d. The supervision, control, and regulation of the practice of
c. To detect misstatements accountancy in the Philippines.
d. To form an opinion on the financial statements

AT – Prelim Rev (875 MCQs) Red Sirug Page 14 of 85


61. A CPA is in public accounting practice when he/she practitioners.
a. Holds, or is appointed to, in an accounting professional c. Issued an unqualified opinion on the 2010 financial
group in government or in a government-owned and/or statements when fees for the 2009 audit were unpaid.
controlled corporation where decision making requires d. Purchased a bookkeeping firm's practice of monthly write-
professional knowledge in the science of accounting. ups for a percentage of fees received over a three-year
b. Represents his/her employer before government period.
agencies on tax and other matters related to accounting.
c. Represents his/her clients before government 66. The confidential relationship applies to
agencies on tax and other matters related to a. All services provided by CPAs.
accounting. b. Only audit and attestation services.
d. Teaches accounting, auditing, management advisory c. Audit and tax services, but no MAS services.
services, accounting aspect of finance, business law, d. Audit and MAS services, but not tax services.
taxation, and other technically related subjects.
67. A CPA’s retention of records as a means of enforcing
62. The underlying reason for a code of professional conduct for payment of an overdue audit fee is an act that is:
any profession is a. A violation of generally accepted auditing standards.
a. That it is required by Congress. b. Considered acceptable by the Code of Ethics.
b. That it provides a safeguard to keep unscrupulous people c. Ill advised since it would impair the CPA’s
out. independence with respect to the client.
c. The need for public confidence in the quality of d. Considered discreditable to the profession.
service of the profession.
d. That it allows Professional Regulation Commission to 68. Which of the following acts of the CPA would be most
have a yardstick to measure deficient performance. likely a violation of the Code of Ethics?
a. Assisting a client in preparing a financial forecast.
63. If a firm, or a network firm, has a direct financial interest in an b. Acting as auditor of a non-charitable organization with
audit client of the firm, the self-interest threat created would be audit client serving as president.
so significant no safeguard could reduce the threat to an c. Accepting a fee in a tax matter that is contingent upon
acceptable level. The action appropriate to permit the firm to the result of an administrative proceeding.
perform the engagement would be to d. Having an immaterial loan to the president of an
a. Dispose of the financial interest. audit.
b. Dispose of a sufficient amount of it so that the remaining
interest is no longer material. 69. CPAs should not be liable to any party if they perform their
c. Either a or b services with:
d. Neither a nor b a. Ordinary negligence.
b. Regulatory providence.
64. Which of the following statements is incorrect? c. Due professional care.
a. CPAs lose their independence if they acquire any direct d. Good faith.
financial interest in a client.
b. CPAs lose their independence if they acquire any 70. Under the Code of Ethics, a CPA may not:
indirect financial interest in a client. a. Perform bookkeeping services for an audit client.
c. CPAs lose their independence if they acquire a material b. Perform advisory services for an audit client.
indirect financial interest in a client. c. Have any direct financial interest in a client during
d. CPAs lose their independence if they have a material the period covered by the financial statements.
direct financial interest in a client. d. Having any joint, closely held investments with a
principal stockholder of an audit client during the
65. A violation of the ethical standards would most likely have period of audit engagement which is not material in
occurred when a CPA relation to his net worth.
a. Made arrangement with a bank to collect notes issued by
a client in payment of fees due. 71. The Code of Ethics would be violated if a CPA accepted a
b. Joined an accounting firm made up of three non-CPA fee for services and the fee was:

AT – Prelim Rev (875 MCQs) Red Sirug Page 15 of 85


a. Fixed by a public authority. because:
b. Based on a price quotation submitted in competitive a. This practice establishes fees which are
bidding. commensurate with the value of the services.
c. Based on the results of judicial proceedings in a tax b. Attorneys in tax practice customarily set contingent
matter. fees.
d. Payable after a specified finding was attained in a c. Determination by taxing authorities are a matter of
review of financial statements. judicial proceedings which do not involve third parties.
d. The consequences are based upon findings of
72. The CPA shall not concurrently engage in any business or judicial proceedings or the findings of tax
occupation which impairs his objectivity in rendering authorities.
professional services or which is inconsistent with his
practice or employment. This provision of the code of 77. Ethically, the auditor could:
ethics is applicable to: a. Advertise only as to his expertise in preparing income
a. Only to CPAs engaged in public accountancy. tax returns.
b. CPAs engaged in public accountancy or employed in b. Base his audit fee on a percentage of the proceeds of
a private enterprise. his client’s stock issue.
c. CPAs engaged in public accountancy or employed in c. Perform an examination for a financially
a private enterprise or government agency. distressed client at less than his customary fees.
d. All CPAs engaged in public accountancy, or d. Own preferred stock in a corporation which is an audit
employed in a private enterprise, a government client.
agency or an educational institution.
78. The Code of Ethics contains both general ethical
73. Ultimately, the decision as to whether the CPA is principles that are aspirational in character and also a:
independent or not, will be made by the: a. List of violations that would cause the automatic
a. Audit Committee suspension of the CPA’s license.
b. Client b. Description of the CPA’s procedures for responding to
c. Public an inquiry from trial board.
d. Auditor c. Set of specific, mandatory rules describing levels
of conduct the CPA must observe.
74. In which of the following instances would the d. List of specific crimes that would be considered as
independence of the CPA not be considered to be acts discreditable to the profession.
impaired? The CPA has been retained as the auditor of a
brokerage firm 79. According to the Code of Ethics, the characteristics
a. Which owes the CPA audit fees for more than one distinguishing the accountancy profession include the
year. following except:
b. In which the CPA has a large active margin account. a. To meet public interest requirement.
c. In which the CPA’s brother is the controller. b. Adherence by its members to a common code of values
d. Which owes the CPA audit fees for the current and conduct established by its administering body.
year services and just filed a petition for c. Acceptance of a duty to society as a whole.
bankruptcy. d. Mastery of a particular intellectual skill, acquired by
training and education.
75. A CPA, while performing an audit, strives to achieve
independence in appearance in order to: 80. A practicing CPA is allowed by the CPA Code of Ethics to
a. Reduce risk and liability. do the following except:
b. Become independent in fact. a. Announce the change in office location in a
c. Comply with GAAS. newspaper.
d. Maintain public confidence in the profession. b. List his firm name in the building lobby directory in
good taste and modest size.
76. Contingent fees charged by CPAs engaged in tax practice c. Include his tax account number and membership in
are permitted under the rules of professional conduct PICPA on his stationary.

AT – Prelim Rev (875 MCQs) Red Sirug Page 16 of 85


d. List his office telephones in the PLDT directory to b. Firm-wide safeguards
box or bold type. c. Engagement specific safeguards
d. Safeguards created by the client.
81. Without the consent of the client, a CPA should not
disclose confidential information contained in working 87. A CPA firm would be reasonable assured of meeting its
papers to a: responsibility to provide professional services that conform
a. Voluntary quality control review board. with professional standards by:
b. CPA firm that has purchased the CPA’s a. Adherence to GAAS
accounting practice. b. Maintaining an attitude of independence in its
c. National court that has issued a valid subpoena. engagements.
d. Disciplinary body created under statute. c. Having an appropriate a system of quality control.
d. Joining professional societies that enforce ethical
82. A CPA who is engaged to prepare an income tax return standards.
has a duty to prepare it in such a manner that the tax is:
a. The legal minimum. 88. The Philippine Code of Ethics requires that lead engagement
b. Computed in conformity with generally accepted partners of listed entities be rotated at least once every:
accounting principles. a. 2 years
c. Supported by the client’s audited financial statements. b. 3 years
d. Not subject to change upon audit. c. 5 years
d. 7 years
83. The principle of Professional behavior requires a CPA to:
a. Perform professional services with due care, 89. The following are represented both to the Financial Reporting
competence and diligence. Standards Council (FRSC) and Auditing and Assurance
b. Be straightforward and honest in performing Standards Council (AASC), except:
professional services. a. Bangko Sentral ng Pilipinas
c. Act in a manner consistent with the good reputation b. Securities and Exchange Commission
of the profession and refrain from any conduct c. Bureau of Internal Revenue
which might bring discredit to the profession. d. Board of Accountancy
d. Be fair and should not allow prejudice or bias, conflict of
interest or influence of others to override objectivity or 90. Which of the following auditor concerns most likely could be
affect his professional or business judgments. so serious that the auditor concludes that a financial statement
audit cannot be conducted?
84. The communication to the public of facts about a CPA which a. The entity has no formal written code of conduct
are not designed for the deliberate promotion of that CPA. b. The integrity of the entity’s management is suspect
a. Advertising c. Procedures requiring segregation of duties are subject to
b. Solicitation management override
c. Indirect promotion d. Management fails to modify prescribed controls for
d. Publicity changes in conditions

85. Which of the following does not belong to the group: 91. Which of the following is a violation of the code of professional
a. Independence of mind ethics for certified public accountants?
b. Independence of fact a. A CPA permits his/her name to be used in a client's
c. Independence in mental attitude advertising as having verified financial data and/or
d. Independence in appearance statistical facts with respect to the client's products.
b. Based on information obtained in an audit, a CPA
86. Safeguards that may eliminate or reduce threats to reports an illegal act of his client to government
independence to an acceptable level fall in the following authorities.
categories except: c. Three years after a partner has retired, the remaining
a. Safeguards created by the profession, legislation or partners continue to practice under a firm name that
regulation includes the name of the retired partner. The retired

AT – Prelim Rev (875 MCQs) Red Sirug Page 17 of 85


partner has severed all connections with the CPA firm. 96. Which of the following does not constitute a practice of
d. A CPA running for public office uses the professional accountancy?
designation "CPA" after his name on posters employed in a. A person holding out himself as one skilled in the
connection with his election campaign. knowledge, science and practice of accounting and as a
qualified person to render professional services as a CPA
92. A CPA in public accounting is prohibited from performing to more than one client.
which of the following actions? b. A person representing his/her employer before
a. Permit the publication of his being the author of a book. government agencies on tax arid other accounting related
b. Be a party to a stratagem which permits a non-CPA to matters.
practice accountancy. c. A person in educational institution teaching accounting,
c. Allow his wife to acquire shares in a corporation's capital auditing, business law, taxation or other technically
stock. related subjects.
d. Act as a stock and transfer agent. d. A person is appointed as a marketing director of a
government owned and controlled corporation.
93. A CPA who has been retained by a client that operates in an
industry totally new to him 97. The criteria for evaluating quantitative information vary. For
a. May not accept such an engagement. example, in the audit of historical financial statements by CPA
b. May accept the engagement only if the accounting firm firms, the criteria are usually:
specializes in the audit of commercial banks. a. Generally accepted auditing standards.
c. May accept the engagement after attaining a suitable b. Acceptable financial reporting framework
level of understanding of the transactions and c. Regulations of the Securities and Exchange Commission
accounting practices unique to commercial banking. d. Regulations of the Bureau of Internal Revenue
d. May accept the engagement because training as a CPA
transcends unique industry characteristics. 98. The firm is to be staffed by personnel who have attained and
maintained the technical standards and professional
94. Which of the following is not an objective of requiring competence required to enable them to fulfill their
registration of individual CPAs and Firms of partnership of responsibilities with due care is the objective of what quality
CPAs engaged in public accounting practice? control policy?
a. The required registration will give equal opportunities a. Ethical requirements
to CPAs in the practice of their profession. b. Leadership responsibilities
b. It will enable the Board of Accountancy to formulate and c. Assignment
implement rules and regulations more effectively for the d. Monitoring
enhancement and maintenance of high professional,
ethical, and technical standards of the accounting 99. In pursuing a CPA firm's quality control objectives, a CPA firm
profession. may maintain records indicating which partners or employees
c. To protect the public against fraud, deception, unethical of the CPA firm were previously employed by the CPA firm's
practices and from the consequences of ignorance, clients. Which quality control element would this be most
incompetence and incapacity in the practice of public likely to satisfy?
accounting. a. Monitoring.
d. It will help PRC and BOA to identify and impose b. Human resources.
corresponding sanctions and penalties on individual c. Ethical requirements.
CPAs. d. Engagement performance.

95. As used in Republic Act No. 9298, this term refers to the 100. An engagement quality control review is required to be
area of practice of accountancy performed:
a. Section a. Immediately after the re-assessment of control risk.
b. Sector b. At engagement completion after the report is issued.
c. Segment c. For all audits of financial statements of listed entities.
d. Line d. For all types of audits, regardless of the subject matter of
the engagement.

AT – Prelim Rev (875 MCQs) Red Sirug Page 18 of 85


requires specialized skills and knowledge beyond those
ordinarily possessed by the practitioner.
SET 3 c. A responsible party is the person who is responsible
for the subject matter or the subject matter
1. The degree of certainty that the practitioner has attained and information.
wishes to convey is: d. In all assurance engagements, the responsible party is
a. A conveyance. the engaging party, i.e., the party that engages the
b. An assertion. practitioner
c. A declaration.
d. An assurance. 7. The assurance report is often addressed to the intended users
which may be:
2. The level of assurance provided by an audit of detecting a a. The party responsible for the subject matter.
material misstatement is referred to as: b. Established by agreement between the professional
a. Positive assurance. accountant and the responsible party.
b. High assurance. c. Both A and B
c. Reasonable assurance. d. Neither A nor B
d. Negative assurance.
8. When performing an assurance service, professional
3. Of the following, which is the broadest concept? accountants use standards or benchmarks to evaluate or
a. Internal control audit. measure the subject matter of an assurance engagement.
b. Audit of financial statements. This is known as:
c. Attestation services. a. PFRS.
d. Assurance services. b. Assertion.
c. Criteria.
4. In performing attestation services, a CPA will normally: d. Conclusion.
a. Recommend uses for information.
b. Improve the quality of information, or its context, for 9. Which of the following services provides positive assurance
decision makers. through attestation?
c. Perform market analyses and cost estimates. a. Tax services
d. States a conclusion about a written assertion. b. Review
c. Auditing
5. Which of the following is not one of the requirements before d. Accounting services
accepting an assurance engagement?
a. The practitioner should be competent and independent. 10. Positive assurance is expressed through:
b. The responsible party and the intended user of a. Attestation
assurance report should be from different b. Declaration
organizations. c. Conclusion
c. The practitioner should accept the engagement only if the d. Stating whether anything has come to the auditor's
subject matter is the responsibility of another party attention that indicates that the financial statements are
d. The practitioner should accept the engagement only if the not presented fairly.
subject matter is identifiable and in the form that can be
subjected to evidence gathering procedures. 11. Which of the following statements best describes review
services?
6. The following statements relate to the three parties involved in a. Review engagements focus on providing assurance on
an assurance engagement. Which is correct? the internal controls of a public company.
a. The responsible party and the intended users should b. Review engagements focus on providing assurance on
always be from different entities. the assertions contained in the financial statements of a
b. A practitioner should not accept an assurance en- public company.
gagement when the subject matter of the engagement c. Review engagements focus on providing advice in a
three-party contract.

AT – Prelim Rev (875 MCQs) Red Sirug Page 19 of 85


d. Review engagements focus on providing limited b. Provide a moderate level of assurance that the
assurance on financial statements of a private information is free of material misstatement.
company. c. Provide a high, but not absolute, level of assurance that
the information is free of material misstatement.
12. A review of a company's financial statements by a CPA firm: d. Carry out those procedures of an audit nature to
a. Is similar in scope to an audit and adds similar credibility which the auditor and the entity and any appropriate
to the statements. third parties have agreed and to report on factual
b. Is significantly less in scope than an audit and results in a findings.
report which provides positive assurance, although not
absolute assurance. 17. Assurance engagements should exhibit the following elements
c. Concludes with the issuance of a report expressing the except
CPA's opinion as to the fairness of the financial a. A subject matter
statements. b. Suitable criteria
d. Is designed to provide only moderate assurance. c. Evidence
d. Appropriate professional fees
13. The objective of a review of financial statements is to:
a. Express an opinion on the overall financial statements. 18. Which of the following is not an element of assurance
b. Carry out audit procedures agreed on with the client and engagements?
other users of report. a. Subject matter
c. State whether anything has come to the auditor's b. Evidence
attention that indicates that the financial statements c. Engagement process
are not presented fairly. d. Suitable criteria
d. Assist the client in the preparation of the financial
statements. 19. Assurance services involve which of the following?
a. Relevance as well as reliability.
14. When providing consulting services, the CPA acts primarily as b. Nonfinancial information as well as traditional financial
a(n): statements.
a. Independent practitioner. c. Electronic databases as well as printed reports.
b. Expert on compliance with industry standards. d. All of the above.
c. Objective advisor on the use of information.
d. Confidential reviewer. 20. Assurance engagement include the following, except
a. An engagement conducted to provide a high level of
15. Which of the following statements concerning consulting assurance that the subject matter conforms in all material
services is incorrect? respects with identified suitable criteria.
a. The performance of consulting services for audit clients b. An engagement conducted to provide a moderate level of
does not, in and of itself, impair the auditor's assurance that the subject matter is plausible in the
independence. circumstances.
b. Consulting services differ fundamentally from the CPA's c. An engagement in accordance with the Philippine
function of attesting to the assertions of other parties. Standard on Assurance Engagement(s).
c. Consulting services ordinarily involve external d. An engagement to perform agreed-upon procedures.
reporting.
d. Most CPAs, including those who provide audit and tax 21. The subject matter of an assurance engagement may include
services, also provide consulting services to their clients. I. Historical or prospective financial information
II. Internal controls
16. In an engagement to perform agreed-upon procedures, an III. Compliance with regulation
auditor is engaged to: a. I and II only
a. Use accounting expertise as opposed to auditing b. I and III only
expertise to collect, classify, and summarize financial c. II and III only
information. d. I, II, and III

AT – Prelim Rev (875 MCQs) Red Sirug Page 20 of 85


22. In assertion-based assurance engagements, the evaluation or
measurement of the subject matter against criteria is 27. Which of the following standards are to be applied to
performed by the compilation engagements and engagements to perform
a. Intended users agreed-upon procedures to information?
b. Responsible party a. PSRSs
c. Practitioner b. PSAs
d. AASC c. PSAEs
d. PSREs
23. Which of the following statements is true concerning evidence
in an assurance engagement? 28. The Philippine Standards on Review Engagements (PSREs)
a. The reliability of evidence is influenced not by its nature are to be applied in
but by its source. a. The audit of historical financial information.
b. Sufficiency is the measure of the quantity of evidence. b. Assurance engagements dealing with subject matters
c. Obtaining more evidence may compensate for its poor other than historical financial information.
quality. c. The review of both historical and prospective financial
d. Appropriateness is the measure of the quality of evidence, information.
that is, its reliability and persuasiveness. d. The review of historical financial information.

24. An unqualified conclusion is not appropriate for either 29. PSRE 2400 (Engagements to Review Financial Statements),
reasonable or limited assurance engagement when: as amended by the AASC in February 2008, applies to
I. Circumstances prevent the practitioner from obtaining a. Reviews of any historical financial information of an audit
evidence required to reduce assurance engagement risk to client.
the appropriate level. b. Reviews of historical financial or other information by a
II. The responsible party or the engaging party imposes a practitioner other than the entity's auditor.
restriction that prevents the practitioner from obtaining c. Reviews of historical financial or other information of an
evidence required to reduce assurance engagement risk to audit client.
the appropriate level. d. Reviews of any historical financial information by a
a. I only practitioner other than the entity's auditor.
b. II only
c. Either I or II 30. When performing a compilation engagement, the accountant
d. Neither I nor II is required to
a. Make inquiries of management to assess the reliability
25. Reducing assurance engagement risk to zero is very rarely and completeness of the information provided.
attainable or cost beneficial as a result of the following factors, b. Assess internal controls.
except c. Verify matters and explanations.
a. The use of selective testing. d. Obtain a general knowledge of the business and
b. The fact that much of the evidence available to the operations of the entity.
practitioner is persuasive rather than conclusive.
c. The practitioner may not have the required assurance 31. Each page of the financial information compiled by the
knowledge and skills to gather and evaluate evidence. accountant should include the following reference, except
d. The use of judgment in gathering and evaluating a. "Unaudited"
evidence and forming conclusions based on that evi- b. "Compiled, Negative Assurance Expressed"
dence. c. "Compiled without Audit or Review"
d. "Refer to Compilation Report"
26. Which of the following standards are to be applied, as
appropriate, in the audit of historical financial information? 32. An auditor may accept an engagement to perform specified
a. PSAEs procedures on the specific subject matter of specified
b. PSREs elements, accounts, or items of a financial statement if
c. PSAs a. The report does not list the procedures performed.
d. PSRSs b. The auditor is also the entity's continuing auditor.

AT – Prelim Rev (875 MCQs) Red Sirug Page 21 of 85


c. The financial statements are prepared in accordance with assurance
a special purpose framework. b. Designed to enable the accountant to express a negative
d. Use of the report is restricted. assurance
c. Not designed to enable the accountant to express
33. An engagement to perform agreed-upon procedures may any form of assurance
involve the auditor in performing certain procedures d. Less extensive than review procedures but more
concerning extensive than agreed-upon procedures
I. Individual items of financial data.
II. A single financial statement. 38. Which of the following procedures is normally performed in
III. A complete set of financial statements. connection with a compilation engagement?
a. I and II only a. Inquire of management about subsequent events
b. II and III only b. Making inquiries of management concerning actions
c. I and III only taken at board meeting
d. I, II, and III c. Applying analytical review procedures
d. Collect, classify and summarize financial information.
34. The following contain basic principles, essential procedures
and related guidance, consistent with the concepts in the 39. An engagement to apply agreed-upon procedure engagement
Framework, for the performance of assurance engagements, may be accepted, provided
except a. The CPA has audited the financial statements of the
a. PSAs client.
b. PSREs b. The CPA is independent with respect to the client.
c. PSAEs c. The distribution of the report will be limited only to
d. PSRSs specified parties involved.
d. The adequacy of the procedures to be performed will be
35. CPAs in public practice who perform assurance engagements determined by the CPA.
are governed by the following, except
a. Framework for Assurance Engagements 40. Pronouncements of Auditing and Assurance Standards
b. Philippine Standards on Quality Control Council (AASC) do not cover
c. Code of Ethics for Professional Accountants in the a. Review engagement
Philippines b. Compilation engagement
d. AASC's Engagement Standards (PSAs, PSREs, c. Consultancy
PSAEs, and PSRSs) d. Agreed-upon procedures engagement

36. The Philippine Framework for Assurance Engagements 41. What is the most likely course of action that will be taken by
a. Provides a frame of reference for CPAs in public practice an auditor in assessing management integrity?
when performing audits, reviews, and compilations of a. Tour the client premises.
historical financial information. b. Research the prospective client’s related parties.
b. Contains basic principles, essential procedures, and c. Research the background and histories of officers
related guidance for the performance of assurance d. Review the minutes of the board of directors
engagements.
c. Establishes standards and provides procedural 42. Which of the following is an example of an assurance
requirements for the performance of assurance en- engagement?
gagements. a. Management advisory services
d. Defines and describes the elements and objectives of b. Reporting on financial statements prepared using
an assurance engagement, and identifies other comprehensive basis of accounting.
engagements to which PSAs, PSREs, and PSAEs c. compilation of financial information
apply. d. preparation of tax returns

37. The procedures employed in doing compilation are: 43. Which of the following services provides a moderate level of
a. Designed to enable the accountant to express a limited assurance about the client's financial statements?

AT – Prelim Rev (875 MCQs) Red Sirug Page 22 of 85


a. Forecasts and projections c. The degree of cooperation the predecessor received
b. Compliance with contractual agreement concerning the inquiry of the client’s lawyer.
c. Review d. The predecessor’s assessments of inherent risk and
d. Compilation judgments about materiality.

44. When an independent auditor is approached to perform an 48. Which of the following factors would most likely cause a CPA
audit for the first time, he/she should make inquiries of the to decide not to accept a new audit engagement?
predecessor auditor. Inquiries are necessary because the a. The CPA’s lack of understanding of the prospective
predecessor may be able to provide the successor with client’s internal auditor’s computer-assisted audit
information that will assist the successor in determining techniques.
whether b. Management’s disregard of its responsibility to
a. A certain amount of audit fee will be acceptable to the maintain an adequate internal control environment.
client. c. The CPA’s inability to determine whether related-party
b. An unqualified opinion may be issued. transactions were consummated on terms equivalent to
c. The audit documentation by the predecessor auditor can arm’s-length transactions.
be utilized. d. Management’s refusal to permit the CPA to perform
d. The engagement proposal should be accepted. substantive tests before the year-end.

45. Before accepting an audit engagement, a successor auditor 49. The following are fundamental principles that are mostly
should make specific inquiries of the predecessor auditor applied when performing procedures on acceptance of new
regarding the predecessor's clients except:
a. Evaluation of all matters of continuing accounting a. Confidentiality.
significance. b. Professional competence.
b. Understanding as to the reasons for the change of c. Independence.
auditors. d. Professional competence and due care.
c. Awareness of the consistency in the application of GAAP
between periods. 50. Before continuing the client relationship, the auditor should
d. Opinion of any subsequent events occurring since the perform the following except:
predecessor's audit report was issued. a. Communicate with the predecessor auditor.
b. Evaluate client in case of change of client’s business.
46. Before accepting an engagement to audit a new client, a CPA c. Evaluate client’s integrity.
is required to obtain d. Evaluate financial condition of the client.
a. An understanding of the prospective client’s industry and
business. 51. A successor auditor most likely would make specific inquiries
b. The prospective client’s signature to the engagement of the predecessor auditor regarding
letter. a. Specialized accounting principles of the client's industry.
c. A preliminary understanding of the prospective client’s b. Disagreements with management as to auditing
control environment. procedures.
d. The prospective client’s consent to make inquiries of c. The competency of the client's internal audit staff.
the predecessor auditor, if any. d. The uncertainty inherent in applying sampling procedures.

47. Before accepting an audit engagement, a successor auditor 52. Which of the following will an auditor least likely discuss with
should make specific inquiries of the predecessor auditor the former auditors of a potential client prior to acceptance?
regarding a. Disagreements with management regarding accounting
a. Disagreements the predecessor had with the client principles
concerning auditing procedures and accounting b. Integrity of management
principles. c. Fees charged for services
b. The predecessor’s evaluation of matters of continuing d. Reasons for changing audit firms
accounting significance.
53. Prior to the acceptance of an audit engagement with a client

AT – Prelim Rev (875 MCQs) Red Sirug Page 23 of 85


who has terminated the services of the predecessor auditor, b. Reduce audit risk by lowering the preliminary levels of
the CPA should materiality.
a. Accept the engagement without contacting the c. Engage financial experts who are familiar with the nature
predecessor auditor since the CPA can include audit of the industry.
procedures to verify the reason given by the client for the d. Design special substantive tests to compensate for the
termination. lack of industry expertise.
b. Contact the predecessor auditor without advising the
prospective client and request a complete report of the 58. The element of the audit planning process most likely to be
circumstance leading to the termination with the agreed upon with the client before implementation of the audit
understanding that all information disclosed will be kept strategy is the determination of the
confidential. a. Timing of inventory observation procedures to be
c. Advise the client of the intention to contact the performed.
predecessor auditor and request permission for the b. Evidence to be gathered to provide a sufficient basis for
contact. the auditor's opinion.
d. Not communicate with the predecessor auditor because c. Procedures to be undertaken to discover litigation, claims,
this would in effect be asking the auditor to violate the and assessments.
confidential relationship between auditor and client. d. Pending legal matters to be included in the inquiry of the
client's attorney.
54. Auditing standards require a successor auditor to
communicate with the predecessor auditor. The Code of 59. A written understanding between the auditor and the client
Professional Conduct requires confidentiality; therefore, the concerning the auditor's responsibility for the discovery of
client's permission must be obtained before the illegal acts is usually set forth in a(an)
communication can be made by a. Letter of audit inquiry.
a. The predecessor auditor. b. Client representation letter.
b. The successor auditor. c. Management letter.
c. Both the successor and predecessor auditor. d. Engagement letter.
d. Neither, since this is one of the exceptions to
confidentiality in the Code of Professional Conduct. 60. The scope and nature of an auditor's contractual obligation to
a client is ordinarily set forth in the
55. Which of the following is not done during the client selection a. Opinion paragraph of the auditor’s report.
and retention phase of planning? b. Management letter.
a. Obtain and review financial information c. Engagement letter.
b. Consider the need for special skills d. Scope paragraph of the auditor’s report.
c. Ensure that the firm has sufficient resources to complete
the engagement in a timely manner. 61. The primary reason an engagement letter is obtained by audit
d. Obtain an understanding of internal controls firms prior to starting the work is that
a. It provides an insurance policy for companies entering
56. Which of the following factors most likely would influence an into the agreement.
auditor’s determination of the auditability of an entity’s b. It clarifies the responsibilities of management and
financial statements? those of the auditor.
a. The complexity of the accounting system. c. It communicates the type of opinion that will be rendered
b. The existence of related-party transactions. on the engagement
c. The adequacy of the accounting records. d. It defines the firm's policies and procedures regarding
d. The operating effectiveness of control procedures. new clients.

57. Prior to beginning fieldwork on a new audit engagement in 62. An engagement letter should be written before the start of an
which a CPA does not possess industry expertise, the CPA audit because
should a. It specifies the client's responsibility for preparing
a. Obtain knowledge of matters that relate to the nature schedules and making the records available to the auditor.
of the entity's business and industry. b. It may limit the auditor's legal liability by specifying the

AT – Prelim Rev (875 MCQs) Red Sirug Page 24 of 85


auditor's responsibilities. c. After performing our preliminary analytical
c. It specifies the basis for billing the audit for the upcoming procedures we will discuss with you the other
year. procedures we consider necessary to complete the
d. All of the above engagement.
d. Our engagement is subject to the risk that material
63. Which of the following matters is generally included in an misstatements or fraud, if they exist, will not be detected.
auditor’s engagement letter?
a. Management’s responsibility for the entity’s 67. Arrangements concerning which of the following are least
compliance with laws and regulations. likely to be included in engagement letter?
b. The factors to be considered in setting preliminary a. Internal auditors
judgments about materiality. b. Predecessor auditor
c. Management’s vicarious liability for illegal acts committed c. Stockholders
by its employees. d. Appraisers
d. The auditor’s responsibility to search for significant
internal control deficiencies. 68. When an auditor believes that an understanding with the client
has not been established, he or she should ordinarily
64. Which of the following is not included in the engagement letter? a. Perform the audit with increased professional skepticism.
a. Objectives of the engagement b. Decline to accept or perform the audit.
b. Management's responsibilities c. Review the client’s financial reports.
c. A clear explanation of the services to be performed on d. Obtain information about the client’s business and the
the engagement industry where it operates.
d. Representations that the financial statements were
prepared in accordance with generally accepted 69. Select the proper reply as to the allowable form of the
accounting principles understanding with a client when an audit is being performed.
a. While preferably written, it may be oral; but in all
65. After an auditor had been engaged to perform the first audit cases it should be documented in the working papers.
for a nonpublic entity, the client requested to change the b. While preferably written, it may be oral, in which case it
engagement to a review. In which of the following situations need not be documented in the working papers.
would there be a reasonable basis to comply with the client's c. The understanding may be in any form, such as oral or
request? written.
a. Management refused to sign the client representation d. The understanding must be obtained in written form and
letter. included in the working papers.
b. The client's bank required an audit before committing
to a loan, but the client subsequently acquired 70. In auditing the financial statements of Star Corp., Land
alternative financing. discovered information leading Land to believe that Star’s prior
c. The auditor was prohibited by the client from year’s financial statements, which were audited by Tell,
corresponding with the client's legal counsel. require substantial revisions. Under these circumstances,
d. The auditing procedures were substantially complete and Land should
the auditor determined that an unqualified opinion was a. Notify Star’s audit committee and stockholders that the
warranted, but there was a disagreement concerning the prior year’s financial statements cannot be relied on.
audit fee. b. Request Star to reissue the prior year’s financial
statements with the appropriate revisions.
66. Which of the following statements would least likely appear in c. Notify Tell about the information and make inquiries about
an auditor’s engagement letter? the integrity of Star’s management.
a. Fees for our services are based on our regular per diem d. Request Star to arrange a meeting among the three
rates, plus travel and other out-of-pocket expenses. parties to resolve the matter.
b. During the course of our audit we may observe
opportunities for economy in, or improved controls over, 71. Orange Corp. has a few large accounts receivable that total
your operations. P2,000,000. Yellow Corp. has a large number of small
accounts receivable that also total P2,000,000. The

AT – Prelim Rev (875 MCQs) Red Sirug Page 25 of 85


importance of an error in any one account is, therefore, 77. The management responsibility to detect and prevent fraud
greater for Orange Corp. than for Yellow Corp. This is an and error is accomplished by
example of the auditor's concept of: a. Having an annual audit of financial statements.
a. Materiality. b. Implementing adequate quality control system.
b. Reasonable assurance. c. Implementing adequate accounting and internal
c. Comparative analysis control system.
d. Relative risk. d. Issuing a representation letter to the auditor.

72. The relationship between materiality and audit risk is: 78. Which of the following statements best describes the auditor's
a. Indeterminable responsibility to detect material errors and fraud?
b. Direct. a. The auditor is responsible for the failure to detect material
c. Inverse. errors and frauds only when such failure results from the
d. Materiality and audit risk have no relationship misapplication of generally accepted accounting
principles.
73. An auditor should design a written audit program so that: b. The auditor is responsible for the failure to detect material
a. All material transactions will be selected for substantive errors and frauds only when the auditor fails to confirm
testing. receivables or observe inventories.
b. Substantive tests prior to the balance sheet date will be c. The audit should be designed to provide reasonable
minimized. assurance that material errors and fraud are detected.
c. The audit procedures selected will achieve specific d. Extended auditing procedures are required to detect
audit objectives. unrecorded transactions even if there is no evidence that
d. Each account balance will be tested under either tests of material errors and frauds may exist.
controls or tests of transactions.
79. Which of the following procedures would an auditor be most
74. Audit programs are modified to suit the circumstances on likely to perform in planning a financial statement audit?
particular engagements. A complete audit program for an a. Reviewing investment transactions of the audit period to
engagement generally should be developed determine whether related parties were created.
a. Prior to beginning the actual audit work. b. Performing analytical procedures to identify areas
b. After the auditor has completed an evaluation of the that may represent specific risks.
existing internal accounting control. c. Obtaining a written representation letter from the client to
c. After reviewing the client's accounting records and emphasize management's responsibilities.
procedures. d. Reading the minutes of stockholder and director meetings
d. When the audit engagement letter is prepared. to discover whether any unusual transactions have
occurred.
75. Which of the following is not a potential effect of an auditor's
decision that a lower acceptable audit risk is appropriate? 80. A CPA is conducting the first audit of a client’s financial
a. More evidence is required. statements. The CPA hopes to reduce the audit work by
b. Less evidence is required. consulting with the predecessor auditor and reviewing the
c. Special care is required in assigning experienced staff. predecessor's working papers. This procedure is
d. Review of the working papers by personnel who were not a. Acceptable if the CPA refers in the audit report to reliance
assigned to the engagement. upon the predecessor auditor's work.
b. Required if the CPA is to render an unqualified opinion.
76. To obtain an understanding of a continuing client’s business in c. Unacceptable because the CPA should bring an
planning an audit, an auditor most likely would independent viewpoint to a new engagement.
a. Read specialized industry journals. d. Acceptable if the client and the predecessor auditor
b. Review prior year working papers and the permanent agree to it.
file for the client.
c. Reevaluate the client’s internal control environment. 81. PSA 315 requires that the auditor should obtain an
d. Perform tests of details of transactions and balances. understanding of relevant industry, regulatory and other
external factors including the applicable financial reporting

AT – Prelim Rev (875 MCQs) Red Sirug Page 26 of 85


framework. Which of the following is not among the items that c. Tests of records and of responses to inquiries
relate to industry conditions? d. Inquiry and analytical procedures
a. Energy, supply and cost
b. Inflation and currency revaluation 88. Which statement is incorrect regarding the pronouncements of
c. Market and competition AASC?
d. Cyclical or seasonal activity a. The PSAs and Interpretations may also have application,
as appropriate, to other related activities of auditors.
82. The risk of material financial statement misstatement may be b. PSAs contain basic principles and essential procedures
greater when the following conditions exist except (identified in bold type black lettering) together with
a. When there is greater management intervention to related guidance in the form of explanatory and other
specify the accounting treatment. material.
b. When there is sufficient personnel with appropriate c. PSAs need only be applied to material matters.
accounting and financial reporting skills. d. The Interpretations have the same authority as the
c. When there is greater manual intervention for data PAPSs.
collection and processing.
d. Complex calculations or accounting principles is involved. 89. The amount of audit fees depend largely on the
a. Size and capitalization of the company under audit.
83. The following are assurance engagements except b. Amount of profit for the year.
a. Tax consulting c. Availability of cash.
b. Financial statements audit d. Volume of audit work and degree of competence and
c. Review of financial statements responsibilities involved.
d. Information system reliability services
90. In determining audit fees, an auditor may take into account
84. Engagements frequently performed by professional each of the following except
accountants that are not assurance engagements include the a. Volume and intricacy of work involved.
following except b. Number and cost of manhours needed.
a. Compilation c. Degree of responsibility assumed.
b. Agreed-upon procedures. d. Size and amount of capital of client.
c. Compliance audit
d. Management consulting. 91. Under this method of billing a client, the external auditors
charges on the basis of time spent by principals/partners,
85. When the professional accountant has obtained sufficient supervisors, seniors and juniors at predetermined rates
appropriate evidence to conclude that the subject matter agreed upon with the client
conforms in all material respects with identified suitable criteria, a. Maximum fee basis
he or she can provide what level of assurance? b. Flat sum basis
a. None c. Retainer basis
b. High d. Per diem basis
c. Absolute
d. Moderate 92. Which of the following will impair the independence of a CPA
in public practice?
86. The Framework of PSA applies to a. He has his name and address listed on a one-page
a. Taxation section of the telephone book.
b. Consultancy b. He obtained a loan from a bank under the normal lending
c. Accounting advice procedures, terms and requirements of that bank.
d. Compilation c. He holds one share of the client's capital stock.
d. He failed to disclose a client's departure from GAAP.
87. Which of the following procedures ordinarily performed during
an audit are also performed in review? 93. When CPAs are able to maintain an independence attitude in
a. Assessment of accounting and internal control systems fulfilling their responsibility, it is referred to as independence in
b. Test of controls a. Fact.

AT – Prelim Rev (875 MCQs) Red Sirug Page 27 of 85


b. Appearance. b. All Certified Public Accountants
c. Conduct. c. Two lawyers and one CPA
d. Total. d. One or two lawyers and one or two CPAs for a total of
three
94. When the users of financial statements have confidence in the
independence of the CPA, it is referred to as in independence 99. Which statement is correct regarding the relationship between
in internal auditing and the external auditor?
a. Fact. a. Some judgments relating to the audit of the financial
b. Appearance. statements are those of the internal auditor.
c. Conduct. b. The external audit function's objectives vary according to
d. Total. management's requirements.
c. Certain aspects of internal auditing may be useful in
95. Which of the following statements is incorrect? determining the nature, timing and extent of external
a. CPAs lose their independence if they acquire any direct audit procedures.
financial interest in a client. d. The external auditor is responsible for the audit opinion
b. CPAs lose their independence if they have a material expressed, however that responsibility may be reduced
direct financial interest in a client. by any use made of internal auditing.
c. CPAs lose their independence if they acquire any
indirect financial interest in a client. 100. Which of the following is incorrect regarding the Philippine
d. CPAs lose their independence if they acquire a material Standards on Assurance Engagements (PSAE)?
indirect financial interest in a client. a. It provides an overall framework for assurance
engagements intended to provide either a high or
96. Which of the following statements is not a distinction between moderate level of assurance.
independent auditing and internal auditing? b. When a professional accountant is engaged to perform
a. Independent auditors represent third party users external an assurance engagement for which specific standards
to the auditee entity, whereas internal auditors report exist, those standards apply.
directly to management. c. It provides basic principles and essential procedures
b. Internal auditors are employees of the auditee, whereas for engagements intended to provide a moderate
independent auditors are independent contractors. level of assurance.
c. The internal auditor's span of coverage goes beyond d. If no specific standards exist for an assurance
financial auditing to encompass operational and engagement, PSAE apply.
performance auditing.
d. Although independent auditors strive for both validity
and relevance of evidence, internal auditors are SET 4
concerned almost exclusively with validity.
1. A professional accountant has a professional duty or right to
97. Which of the following is a correct qualification of the disclose confidential information in each of the following,
Chairman and Two Commissioners of the Commission on except:
Audit? a. To comply with technical standards and ethics
a. A citizen of the Philippines. requirements.
b. At least 40 years of age upon appointment. b. To disclose to BIR fraudulent scheme committed by
c. Must not have been candidates for any elective the client on payment of income tax.
position preceding appointment. c. To comply with the quality review of a member body or
d. CPA’s with no less than 5 years of auditing experience or professional body
members of Philippine bar who have been engaged in d. To respond to an inquiry or investigation by a member
law practice for at least 5 years. body or regulatory body.

98. The 1986 Constitution provides that the Chairman and 2. Which of the following best describes the passing of
Commissioners of the Commission on Audit shall be confidential information from a client to its auditor? The
a. All lawyers information:

AT – Prelim Rev (875 MCQs) Red Sirug Page 28 of 85


a. Should in no circumstances be conveyed to third parties. 8. The confidential relationship will be violated if, without client's
b. Is not legally protected and can be subpoenaed by a permission, the CPA provides working papers about client to
competent court. a. A court of law which subpoenas them.
c. Can only be released for peer reviews after receiving b. Another CPA firm as part of a peer review.
permission from the client. c. Another CPA firm which has just purchased the
d. Should be conveyed to the public if it affects the CPA's entire practice.
"correctness" of the financial statements. d. An investigative or disciplinary body which is conducting
a review of the CPA's practice.
3. If an auditor had a substantial stock investment in a client that
s(he) was auditing, which of the following would be true? 9. A member in public practice may perform for a contingent fee
a. The auditor would lack independence. any professional services for a client for whom the member
b. The auditor would be violating the PASB standards. or member's firm performs
c. The auditor would be violating the Institute of a. An audit.
Management Accounting standards b. A review.
d. The auditor would be violating the IIA standards. c. A compilation used only by management.
d. An audit of prospective financial information.
4. The CPA must not subordinate his or her professional
judgment to that of others in every 10. Which one of the following contingent fee is allowed?
a. Engagement. a. All services performed by a CPA firm.
b. Audit engagement. b. Non-attestation services.
c. Engagement except tax services. c. Non-attestation services, unless the CPA firm was
d. Engagement except management advisory services. also performing attestation services for the same
client.
5. Which of the following is an indication of lack of objectivity of d. Attestation services.
an auditor?
a. The auditor believes that accounts receivable may 11. A violation of the ethical standards would most likely have
not be collectible, but accepts management's opinion occurred when a CPA
without an independent evaluation. a. Made arrangement with a bank to collect notes issued by
b. In preparing client's tax return, the CPA encourages client a client in payment of fees due.
to take a deduction which the CPA believes is valid, but b. Joined an accounting firm made up of three non-CPA
for which there is some but not complete support. practitioners.
c. Both a and b above would be a violation c. Issued an unqualified opinion on the 2006 financial
d. Neither would be a violation statements when fees for the 2005 audit were unpaid.
d. Purchased a bookkeeping firm's practice of monthly write-
6. A CPA in public practice shall not disclose any confidential ups for a percentage of fees received over a three-year
client information without the specific consent of the client. period.
The confidentiality rule is violated if CPA disclosed
information without client's consent as a result of a 12. The concept of materiality would be least important to an
a. Subpoena or summons. auditor when considering the
b. Peer review. a. Decision whether to use positive or negative
c. Request by client's largest stockholder. confirmations of accounts receivable.
d. Complaint filed with the trial board of the Board of b. Adequacy of disclosure of a client's illegal act.
Accountancy.: c. Discovery of weaknesses in a client's internal control
structure.
7. The confidential relationship applies to d. Effects of a direct financial interest in the client upon
a. All services provided by CPAs. the CPA's independence.
b. Only audit and attestation services.
c. Audit and tax services, but no MAS services. 13. Which of the following is a violation Confidentiality rule of the
d. Audit and MAS services, but not tax services. Code of Ethics?
a. The CPA, in response to a court subpoena, submits

AT – Prelim Rev (875 MCQs) Red Sirug Page 29 of 85


auditor-prepared workpapers as evidence of possible d. Member would not be independent.
illegal acts perpetrated by the client.
b. The CPA discloses to the board of directors a scheme 18. In which of the following instances would the independence of
concocted by top management to intentionally inflate the CPA not be considered to be impaired? The CPA has
earnings. been retained as the auditor of a brokerage firm
c. The CPA warns Client B as to the inadvisability of a. Which owes the CPA audit fees for more than one year.
acquiring Client A. The CPA bases this warning on b. In which the CPA has a large active margin account.
knowledge of Client A's financial condition and a c. In which the CPA's brother is the controller.
belief that the management of Client A lacks integrity. d. Which owes the CPA audit fees for services in the
This knowledge was obtained by the CPA as a result current year and has just filed a petition for
of auditing Client A during the past several year is. bankruptcy.
d. The CPA, when questioned in court, admits to knowledge
of certain illegal acts perpetrated by the client. 19. In performing an audit, Jackson, CPA, discovers that the
professional competence necessary for the engagement is
14. An auditor who accepts an audit engagement and does not lacking. Jackson informs management of the situation and
possess the industry expertise of the business entity, should recommends another local firm, and management engages
a. Engage, financial experts familiar with the nature of the this other firm. Under these circumstances,
business entity. a. Jackson may request compensation from the other
b. Obtain a knowledge of matters that relates to the firm for any professional services rendered to it in
nature of the entity's business. connection with the engagement.
c. Refer a substantial portion of the audit to another cpa b. Jackson may accept a referral fee from the other firm.
who will act as the principal auditor. c. Jackson has violated the AICPA Code of Professional
d. First inform management that an unqualified opinion Conduct because of nonfulfillment of the duty of
cannot be issued. performance.
d. Jackson's lack of competence should be construed to be
15. In determining estimates of fees, an auditor may take into a violation of generally accepted auditing standards.
account each of the following, except the:
a. Value of the service to the client 20. Which of the following fee arrangements is in violation of the
b. Degree of responsibility assumed by undertaking the Code of Professional Conduct?
engagement. a. A fee based on whether the CPA's report on the
c. Skills required to perform the service. client's financial statements results in the approval of
d. Attainment of specific findings. a bank loan.
b. A fee based on the outcome of a bankruptcy proceeding.
16. A CPA, while performing an audit, strives to achieve c. A fee based on the nature of the service rendered and
independence in appearance in order to the CPA's particular expertise instead of the actual time
a. Reduce risk and liability. spent on the engagement.
b. Comply with the generally accepted standards of d. A fee based on the fee charged by the prior auditor.
fieldwork.
c. Become independent in fact. 21. Richard, CPA, performs accounting services for Norton
d. Maintain public confidence in the profession Corporation. Norton wishes to offer shares to the public and
asks Richard to audit the financial statements. Richard refers
17. The IFAC Code of Professional Conduct will ordinarily be Norton to Cruz, CPA, who is more competent in the area of
considered to have been violated when the member registration statements. Cruz performs the audit of Norton's
represents that specific consulting services will be performed financial statements and subsequently thanks Richard for the
for a stated fee and it is apparent at the time of the referral by giving Richard a portion of the audit fee. Richard
representation that the accepts the fee. Who, if anyone, has violated professional
a. Actual fee would be substantially higher. ethics?
b. Actual fee would be substantially lower than the fees a. Only Richard
charged by other members for comparable services. b. Both Richard and Cruz
c. Fee was a competitive bid. c. Only Cruz

AT – Prelim Rev (875 MCQs) Red Sirug Page 30 of 85


d. Neither Richard nor Cruz 27. The extent of audit planning will vary according to the
following:
22. Which of the following would not fit the description of a a. Auditor’s experience with the entity.
related-party transaction? b. The nature and complexity of the audit engagement.
a. An unusually large sale of merchandise to the c. Size of the entity.
company's best and largest customer. d. All of the above.
b. Sales of merchandise between a parent company and its
subsidiary. 28. The audit plan should (select the exception)
c. Exchanges of equipment between two companies owned a. Be flexible
by the same person. b. Precede performance of procedures
d. Loans to corporate officers at market rates of interest with c. Succeed action
a regular repayment schedule. d. Be cost-beneficial

23. The audit work performed by each assistant should be 29. The establishment of an overall audit strategy involves
reviewed to determine whether it was adequately performed I. Determining the characteristics of the engagement that
and to evaluate whether the define its scope.
a. Results are consistent with the conclusions to be II. Ascertaining the reporting objectives of the engagement
presented in the auditor's report. to plan the timing of the audit and the nature of the
b. Audit procedures performed are approved in the communications required.
professional standards. III. Considering the important factors that will determine the
c. Audit has been performed by persons having adequate focus of the engagement team's efforts.
technical training and proficiency as auditors. a. I and II only
d. Auditor's system of quality control has been maintained at b. I and III only
a high level. c. II and III only
d. I, II, and III
24. Risk assessment procedures include the following, except
a. Analytical procedures. 30. In the planning stage of an audit engagement, the auditor is
b. Confirmation of accounts receivable. required to perform audit procedures to obtain an
c. Observation and inspection. understanding of the entity and its environment, including its
d. Inquiries of management internal control. These procedures are called
a. Substantive tests
25. PSA 315 requires: b. Tests of controls
a. Obtaining an understanding of the entity and its c. Risk assessment procedures
environment d. Dual-purpose tests
b. Identifying and assessing the risks of material
misstatement 31. In planning the audit engagement, the auditor should consider
c. Discussion among engagement team members about the each of the following, except
risk of material misstatement in the financial statements. a. Matters relating to the entity's business and the industry
d. All of the above in which it operates.
b. Materiality level and audit risk.
26. A time budget is an estimate of the total hours an audit is c. The kind of opinion (unmodified, qualified, or adverse)
expected to take. The following are among the factors to be that is likely to be expressed.
considered in developing this budget, except d. The entity's accounting policies and procedures.
a. Client's size as indicated by its gross assets, sales,
number of employees. 32. The auditor's understanding of the entity and its environment
b. Location of client facilities. consists of an understanding of the following aspects except
c. The competence and experience of available staff. a. Industry, regulatory, and other external factors, including
d. Whether the audit is performed during the interim or the applicable financial reporting framework.
at year-end. b. Nature of the entity, including the entity's selection and
application of accounting policies.

AT – Prelim Rev (875 MCQs) Red Sirug Page 31 of 85


c. Measurement and review of the entity's financial d. Assignment of firm personnel with appropriate levels of
performance. capabilities and competence to respond to anticipated
d. Entity's selection and screening process of significant risks.
marketing and production personnel.
37. Which of the following would not be found in the corporate
33. PSA 315 requires that the auditor should obtain an charter?
understanding' of relevant industry, regulatory and other a. The date of incorporation.
external factors including the applicable financial reporting b. The rules and procedures adopted by the
framework. Which of the following is not an example of stockholders.
matters relating to regulatory environment that the auditor c. The kinds and amount of capital stock authorized.
would usually consider? d. The types of business activity that the corporation is
a. Regulatory framework for a regulated industry. allowed to conduct.
b. Product technology relating to the entity's product.
c. Legislation and regulation significantly affecting the 38. Which of the following would not usually be included in the
entity's operation. minutes of the board of directors and/or stockholders?
d. Taxation. a. Declaration of dividends.
b. Authorization of long-term loans.
34. PSA requires that the auditor should obtain an understanding c. Authorization of individuals to sign checks.
of the entity's objectives and strategies, and the related d. The duties and powers of the corporate officers.
business risks that may result in material misstatement of the
financial statements. Which of the following is not an example 39. Which of the following is not an inherent risk that is common to
of business risks that may have financial consequences and all clients in certain industries?
may affect the financial statements? a. Accounts receivable collection in the consumer loan
a. A contracting customer base due to industry industry.
consolidation that may increase the risk of misstatement b. Potential inventory obsolescence in the fashion clothes
associated with the valuation of receivables. industry.
b. Contracting economy. c. Brand loyalty in the cosmetics industry.
c. New accounting requirements. d. Reserve for loss in the casualty insurance industry.
d. Use of new IT.
40. Transactions with related parties are important to the auditors
35. Auditors perform analytical procedures in the planning stage because they will be disclosed in the financial statements if
of an audit for the purpose of: material. Generally accepted accounting principles would not
a. Determining which of the financial statement assertions require disclosure of
are the most important for the client's financial statements. a. Loans to officers during the year which had been
b. Determining the nature, timing, and extent of audit repaid before the balance sheet date.
procedures for auditing the inventory. b. The nature of the related-party relationship.
c. Deciding the matters to cover in an engagement. c. A description of transactions, including peso amounts.
d. Identifying unusual conditions that deserve more d. The amounts due from and to related parties.
auditing effort.
41. Which of the following would not be classified as a related-
36. For initial audits, additional matters the auditor may consider party transaction?
in the overall audit strategy and audit plan include the a. Sales of merchandise between affiliated companies.
following except b. An advance of one week's salary to an employee.
a. Major issues including the application of accounting c. Loans or credit sales to the principal owner or client.
principles or any auditing and reporting standards d. Exchanges of equipment between two companies owned
discussed with management. by the same person.
b. Confirmation of material accounts receivable balance
at the end of the year. 42. Most auditors assess inherent risk as high for related parties
c. Planned audit procedure to obtain sufficient appropriate and related-party transactions because
audit evidence regarding opening balances. a. Of the accounting disclosure requirement.

AT – Prelim Rev (875 MCQs) Red Sirug Page 32 of 85


b. Of the lack of independence between the parties. c. Objectives and strategies and the related business risks
c. Both a and b. that may result in a material misstatement of the financial
d. It is required by generally accepted accounting principles. statements
d. All of these.
43. Experience has shown that certain conditions in an
organization are symptoms of possible management fraud. 48. There are fundamental principles that the professional
Which of the following conditions would not be considered an accountant has to observe when performing assurance
indicator of possible fraud? engagements. The requirement of which principle is of
a. Managers are regularly assuming subordinates' duties particular importance in an assurance engagement in ensuring
b. Managers are subject to formal performance reviews that the conclusion of the professional accountant has value to
on a regular basis. the intended user?
c. Managers are dealing in matters outside their profit a. Integrity
center's scope b. Confidentiality
d. Managers are not complying with corporate directives c. Professional competence
and procedures d. Objectivity

44. The nature of the entity refers to the following, except: 49. Existing accountant, as defined in the Code of Ethics, means
a. The types of investments that it is making and plans to a. A professional accountant employed in industry,
make. commerce, the public sector or education.
b. Other external factors, such as general economic b. Those persons who hold a valid certificate issued by the
conditions. Board of Accountancy.
c. The way that the entity is structured and how it is c. A professional accountant in public practice
financed. currently holding an audit appointment or carrying
d. The entity's operations, its ownership, and governance. out accounting, taxation, consulting or similar
professional services for a client.
45. These result from significant conditions, events, d. A sole proprietor, or each partner or person occupying a
circumstances, actions or inactions that could adversely affect position similar to that of a partner and each staff in a
the entity’s ability to achieve its objectives and execute its practice providing professional services to a client
strategies or through the setting of inappropriate objectives irrespective of their functional classification (e.g., audit,
and strategies. tax or consulting) and professional accountants in a
a. Business failure practice having managerial responsibilities.
b. Information risk
c. Business obstacles 50. How did the Code of Ethics define public interest?
d. Business risk a. A distinguishing mark of a profession is the acceptance of
its responsibility to the public.
46. These are the operational approaches by which management b. The accountancy profession's public consists of clients,
intends to achieve its objectives. credit grantors, governments, employers, employees,
a. Business risk approaches investors, the business and financial community, and
b. Strategies others who rely on the objectivity and integrity of
c. Planning methods professional accountants.
d. Operational plans. c. The collective well-being of the community of people
and institutions the professional accountant serves.
47. The auditor's understanding of the entity and its environment d. The standards of the accountancy profession are
consists an understanding of the following aspects: heavily determined by the public interest.
a. Industry, regulatory and other external factors, including
the applicable financial reporting framework 51. Independent auditors of financial statements perform audits
b. Nature of the entity, including the entity's selection and that reduce and control
application of accounting policies a. Business risk faced by investors
b. Information risk faced by investors
c. Complexity of financial statements

AT – Prelim Rev (875 MCQs) Red Sirug Page 33 of 85


d. Timeliness of financial statements financial statements, the auditor
a. Guarantees the fair presentation of the financial
52. Which of the following statements is true? statements.
a. Professional standards prohibit CPAs from performing b. Confirms the accuracy of the financial statements.
non-assurance engagements. c. Enhances the credibility of the financial statements.
b. Absolute assurance is attainable owing to the fact that d. Assures the readers that fraudulent activities of
much of the evidence available to the CPA is persuasive employees have been detected.
rather than conclusive.
c. The CPA’s conclusion provides a level of assurance 58. Most of the independent auditor's work in formulating
about the subject matter. an opinion on financial statements consists of
d. The responsible party expresses a conclusion that a. Studying and evaluating internal control
provides a level of assurance as to whether the subject b. Obtaining and examining evidential matter
matter conforms, in all material respects, with the c. Examining cash transactions
identified suitable criteria. d. Comparing recorded accountability with assets

53. Which of the following best describes the operational audit? 59. The overall objective of internal auditing is to
a. It requires the constant review by internal auditors of the a. Attest to the efficiency with which resources are
administrative controls as they relate to operations of employed.
the company. b. Ascertain that controls are costs justified.
b. It concentrates on implementing financial and c. Provide assurance that financial data have been
accounting control in a newly organized company. accurately recorded.
c. It concentrates on seeking out aspects of d. Assist members of the organization in the effective
operations in which waste would be reduced by the discharge of their responsibilities.
introduction of controls.
d. It attempts and is designed to verify the fair 60. Internal auditing is an independent appraisal function
presentation of a company's results of operations. established within an organization to examine and evaluate its
activities. To that end, internal auditing provides assistance to
54. What is the proper organizational role of internal auditing? a. External auditors
a. To serve as an independent, objective assurance and b. Stockholders
consulting activity that adds value to operations. c. Management and the board of directors
b. To assist the external auditor in order to reduce external d. Government
audit fees.
c. To perform studies to assist in the attainment of more 61. An audit which is undertaken in order to determine whether
efficient operations. the auditee is following specific procedures or rules set down
d. To serve as the investigative arm of the audit committee by some higher authority is classified as a(n)
of the board of directors. a. Audit of financial statements.
b. Compliance audit.
55. Which of the following is not one of the limitations of an audit? c. Operational audit.
a. The use of testing d. Production audit.
b. Limitations imposed by client
c. Human error 62. The most important function of operational audit report is to:
d. Nature of evidence that the auditor obtains a. Direct management to take specified actions.
b. State the auditor’s opinion or conclusion.
56. Which of the following terms does not belong to the group c. Report findings and recommendations.
a. Financial audit d. Report the objective of the audit.
b. Internal audit
c. External audit 63. Operational audit differs in many ways from an audit of
d. Independent audit financial statements. Which of the following is the best
example of one of these differences?
57. By providing high level of assurance on audit reports on

AT – Prelim Rev (875 MCQs) Red Sirug Page 34 of 85


a. The usual audit of the financial statement covers four about the efficiency and effectiveness of an entity's operating
basic statements, whereas operational audit is usually activities in relation to specified objectives is a(n):
limited either the statement of financial position or the a. External audit
b. Compliance audit
income statement.
c. Operational audit
b. Operational audit do not necessarily result in the d. Financial statement audit
preparation of a report.
c. The operational audit deals with pre-tax income. 69. A operational audit is primarily oriented toward
d. The boundaries of an operational audit are often a. Future improvements to accomplish management’s
drawn from an organization chart and are not limited goals.
to a single accounting period. b. Past protection provided by existing internal control.
c. Operational information is in accordance with
generally accepted accounting principles.
64. Which of the following actions would be an appropriate
d. Financial statements are fairly presented.
response by companies to improve the public's perception of
their financial reporting?
70. As used in auditing, which of the following statements best
a. Requiring internal auditors to report all significant findings of
describes "assertions”?
fraud and illegal activity to the company president.
a. Assertions are the auditor's findings to be communicated
b. Increased adoption of audit committees.
in his audit report.
c. Keeping external and internal auditing work separated to
b. Assertions are found only in the notes to the financial
maintain independence.
statements.
d. None of the above.
c. Assertions are the representations of management as to
the reliability of the information system.
65. Which of the following is considered a primary reason for
d. Assertions are the representations of management as
creating an internal audit department?
to the fairness of presentation of the financial
a. To safeguard resources entrusted to the organization.
statements.
b. To evaluate and improve the effectiveness of
control processes.
71. The internal auditing department's responsibility for deterring
c. To ensure the accuracy, reliability, and timeliness of
fraud is to
financial and operating data used in management's
a. Exercise operating authority over fraud prevention
decision making.
activities.
d. To relieve management of the responsibility for
b. Establish an effective internal control system.
establishing effective controls.
c. Maintain internal control.
d. Examine and evaluate the system of internal control.
66. What is the overall objective of internal auditing?
a. To attest to the efficiency with which resources are used.
72. Internal auditors review the adequacy of the company's
b. Ascertain that the cost of internal control is justified.
internal control system primarily to
c. To ascertain that financial statements present
a. Help determine the nature, timing, and extent of tests
accurately the financial position, operating results, and
necessary to achieve audit objectives.
changes in cash and stockholders' equity.
b. Determine whether the internal control system ensures
d. To help members of the organization to effectively
that financial statements are fairly presented.
discharge their responsibilities.
c. Determine whether the internal control system
provides reasonable assurance that the company's
67. Which of the following types of audit uses laws and
objectives and goals are met efficiently and
regulations as its criteria?
economically.
a. Operational audit
d. Ensure that material weaknesses in the system of internal
b. Financial statement audit
control are corrected.
c. Compliance audit
d. Performance audit
73. Internal auditors should review the means of physically
safeguarding assets from losses arising from
68. An audit that involves obtaining and evaluating evidence

AT – Prelim Rev (875 MCQs) Red Sirug Page 35 of 85


a. Procedures that are not cost justified. a. Auditors usually rely on the work of an ex pert as
b. Exposure to the elements. a basis for evaluating some assertions embodied in
c. Underusage of physical facilities. the financial statements.
d. Misapplication of accounting principles. b. The readers of the financial statements must possess
the necessary expertise to be able to understand the
74. The essence of the attest function is to financial statements.
a. Detect fraud. c. Users usually lack the necessary expertise to verify
b. Examine individual transactions so that the auditor can the reliability of the financial information.
certify as to their validity. d. As experts, auditors are expected to detect all material
c. Determine whether the client's financial statements misstatements in the financial statements.
are fairly stated.
d. Ensure the consistent application of correct accounting 79. Operational audits generally have been conducted by internal and
procedures. COA auditors, but may be performed by certified public
accountants. A primary purpose of an operational audit is to
75. A typical objective of an operational audit is to determine provide:
whether an entity's a. A measure of management performance in meeting
a. Internal control structure is adequately operating as organizational goals.
designed b. The results of internal examinations of financial and
b. Operational information is in accordance with accounting matters to a company's top-level management.
generally accepted accounting principles. c. Aid to the independent-auditor, who is conducting the
c. Specific operating units are functioning efficiently examination of the financial statements.
and effectively d. A means of assurance that internal accounting controls are
d. Financial statements present fairly the results of functioning as planned.
operations
80. Governmental audi ting often ex tends beyond
76. Audits of financial statements include an expression of a examinations leading to the expression of opinion on the
conclusion about which of the following financial statement fairness of financial presentation and includes audits of
characteristics? efficiency, economy, effectiveness, and also:
a. Governance. a. Accuracy
b. Reliability. b. Evaluation
c. Relevance. c. Compliance
d. Timeliness. d. Internal control

77. There are four conditions that give rise to the need for 81. The internal auditing profession has advanced primarily as a
independent audits of financial statements. One of these result of
conditions is consequence. In this context, consequence a. Increased interest by Bachelor of Science in Accoun-
means that the: tancy (BSA) graduates and experienced auditors.
a. Users of the statements may not fully understand the b. The limitation of financial statement audit scope.
consequences of their actions. c. Job qualification specifications that include added emphasis
b. Auditor must anticipate all possible consequences on background knowledge and skills.
of the report issued. d. Increased complexity and sophistication of business
c. Impact of using different accounting methods may operations.
not be fully understood by the users of the statements.
d. Financial statements are used for important 82. A typical objective of an operational audit is for the auditor to
decisions. a. Determine whether the financial statements fairly
present the entity's operations.
78. One of the conditions that give rise to a demand for an b. Evaluate the feasibility of attaining the entity's
external audit of financial statements is expertise. Which operational objectives.
of the following best describes the meaning of expertise as c. Make recommendation for improving performance.
used in this context? d. Report on the entity's relative success in maximizing

AT – Prelim Rev (875 MCQs) Red Sirug Page 36 of 85


profits. d. Not independent and, therefore, may issue a review
report, but may not issue an auditor's opinion.
83. An objective of a performance audit is to determine
whether an entity's 88. Which of the following most completely describes how
a. Operational information is in accordance with independence has been defined by the CPA profession?
government auditing standards. a. Performing an audit from the viewpoint of the public.
b. Specific operating units are functioning b. Avoiding the appearance of significant interests in the
economically and efficiently. affairs of an audit client.
c. Financial statements present fairly the results of c. Possessing the ability to act with integrity and
operations. objectivity.
d. Internal control is adequately operating as designed. d. Possessing the ability to act professionally and
accordance with a professional code of ethics.
84. What is the responsibility of an auditor who is engaged to
audit the financial statements of a government entity? 89. To emphasize auditor independence from management,
a. Assess control risk with respect to each component of many corporations follow the practice of
internal control. a. Appointing a partner of the CPA firm conducting the
b. Assume responsibility for assuring that the entity complies examination to the corporation's audit committee.
with applicable laws and regulations. b. Establishing a policy of discouraging social contact
c. Obtain an understanding of the possible financial between employees of the corporation and the staff of the
statement effects of laws and regulations having direct independent auditor.
and material effects on amounts reported. c. Requesting that a representative of the independent
d. Design the audit to provide reasonable assurance that the auditor be on hand at the annual stockholders' meeting.
statements are free of material misstatements resulting d. Having the independent auditor report to an audit
from illegal acts having direct or indirect effects. committee of outside members of the board of
directors.
85. Solicitation consists of the various means that CPA firms use
to engage new clients. Which one-of the following would not 90. In determining independence with respect to any audit
be an example of solicitation? engagement, the ultimate decision as to whether or not the
a. Accepting new clients that approach the firm. auditor is independent must be made by the
b. Taking prospective clients to lunch. a. Auditor.
c. Offering seminars on current tax law changes to potential b. Client.
clients. c. Audit committee.
d. Advertisements in the yellow pages of a phone book. d. Public.

86. Which of the following activities is not prohibited for the CPA 91. When a CPA who is not independent is associated with
firm's attestation service clients? financial statements, he would be precluded from expressing
a. Competitive bidding on audit jobs. an opinion because
b. Contingent fees on audit jobs. a. The public would be aware of his lack of independence
c. Commissions for obtaining client services on audit jobs. and would place little or no faith in his opinion.
d. Referral fees on audit jobs. b. He would place himself in the position of suffering an
adverse decision in a possible liability suit.
87. If requested to perform a review engagement for a nonpublic c. He would be in the position of auditing his own work.
entity in which an accountant has an immaterial direct financial d. Any auditing procedures he might perform would not
interest, the accountant is be in accordance with generally accepted auditing
a. Independent because the financial interest is immaterial standards.
and, therefore, may issue a review report.
b. Not independent and, therefore, may not be associated 92. In which of the following circumstances would a CPA be
with the financial statements. bound by ethics to refrain from disclosing any confidential
c. Not independent and, therefore, may not issue a information obtained during the course of a professional
review report. engagement?

AT – Prelim Rev (875 MCQs) Red Sirug Page 37 of 85


a. The CPA is issue a summons enforceable by a court d. An auditor should assess whether the entity has
order which orders the CPA to present confidential reportable measures of economy and efficiency
information. that are valid and reliable.
b. A major stockholder of a client company seeks
accounting information from the CPA after 96. Which of the following is incorrect about responsibility for
management declined to disclose the requested financial statements?
information. a. Management is responsible for fair presentation of the
c. Confidential client information is made available as part of financial statements.
a quality review of the CPA's practice by a peer review b. Auditor is responsible for expressing an opinion on the
team authorized by the PICPA. financial statements.
d. An inquiry by a disciplinary body of PICPA requests c. Audit of financial statements does not reduce
confidential client information. management's responsibility.
d. Fair presentation of financial statements is an implicit
93. Which of the following best describes why publicly-traded part of the auditor's responsibility.
corporations follow the practice of having the outside auditor
appointed by the board of directors or elected by the 97. Which of the following is correct?
stockholders? a. The evidence which the auditor accumulates remains the
a. To comply with the regulations of the Accounting same from audit to audit, but the general objectives vary,
Standards Council. depending on the circumstances.
b. To emphasize auditor independence from the b. The general audit objectives remain the same from
management of the corporation. audit to audit, but the evidence varies, depending on
c. To encourage a policy of rotation of the independent the circumstances.
auditors. c. The circumstances may vary from audit to audit, but the
d. To provide the corporate owners with an opportunity to evidence accumulated remains the same.
voice their opinion concerning the quality of the auditing d. The general audit objectives may vary from audit to audit,
firm selected by the directors. but the circumstances remain the same.

94. The objective of governmental effectiveness or program 98. The work of each assistant needs to be reviewed by personnel
auditing is to determine if the desired results of a program of at least equal competence. Which of the following is not one
are being achieved. What is the first step in conducting of the objectives of this requirement?
such an audit? a. The conclusions expressed are consistent with the result
a. Identify the legislative intent of the program being of the work performed and support the opinion.
audited. b. The work performed and the results obtained have been
b. Collect quantifiable data on the program's success adequately documented.
or failure. c. The objectives of the audit procedures have been
c. Determine the time frame to be audited. achieved.
d. Evaluate the system used to measure results. d. All available evidences have been obtained,
evaluated and documented.
95. Which of the following statements is a standard
applicable to financial statement audits in accordance 99. Every independent audit engagement involves both auditing
with Government Auditing Standards? standards and auditing procedures. The relationship between
a. An auditor should briefly describe in the auditor's report the the two may be illustrated by how they apply from
method of statistical sampling used in per forming tests engagement to engagement. The best representation of this
of controls and substantive tests. application is that, from one audit engagement to the next,
b. An auditor should report on the scope of the a. Both auditing standards and auditing procedures are
auditor's testing of internal control. applied uniformly.
c. An auditor should determine the extent to which b. Auditing standards are applied uniformly but auditing
the entity's programs achieve the desired level of procedures are optional.
results. c. Auditing standards are applied uniformly but auditing
procedures may vary.

AT – Prelim Rev (875 MCQs) Red Sirug Page 38 of 85


d. Auditing standards may vary but auditing procedures are d. The manner in which the auditor emphasizes to
applied uniformly. engagement team members the need to maintain a
questioning mind and to exercise professional skepticism
100. The CPA should not undertake an engagement if his fee is to in the gathering and evaluation of audit evidence.
be based upon
a. A percentage of audited net income. 5. Which of the following matters would an auditor most likely
b. Per diem rates plus expenses. consider when establishing the scope of the audit?
c. The findings of a tax authority. a. Audit areas where there is a higher risk of material
d. The complexity of the service rendered. misstatement.
b. The expected audit coverage, including the number
and locations of the entity's components to be
SET 5 included.
c. The entity's timetable for reporting, such as at interim and
1. The establishment of the overall audit strategy involves final stages.
a. Determining the characteristics of the engagement that d. The discussion with the entity's management concerning
defines its scope. the expected communications on the status of audit work
b. Ascertaining the reporting objectives of the engagement throughout the engagement and the expected
to plan the timing of the audit and the nature of the deliverables resulting from the audit procedures.
communications required.
c. Considering the important factors that will determine the 6. An auditor should consider materiality when
focus of the engagement team's efforts. I. Determining the nature, timing, and extent of audit
d. All of the answers procedures.
II. Evaluating the effect of misstatements.
2. Which of the following is least likely to be required on an audit? a. I only
a. Review accounting estimates for biases b. II only
b. Test appropriateness of journal entries and adjustment c. Both I and II
c. Make a legal determination of whether fraud has d. Neither I nor II
occurred
d. Evaluate the business rationale for significant, unusual 7. It is the amount or amounts set by the auditor at less than
transactions materiality for the financial statements as a whole to reduce to
an appropriately low level the probability that the aggregate of
3. Which of the following should be included in the audit plan? uncorrected and undetected misstatements exceeds
I. The nature, timing and extent of planned risk assessment materiality for the financial statements as a whole.
procedures. a. Lower materiality
II. The nature, timing and extent of planned further audit b. Lesser materiality
procedures at the assertion level. c. Performance materiality
a. I only d. Materiality
b. II only
c. Both I and II 8. A basic premise underlying analytical procedures is that
d. Neither I nor II a. The study of financial ratios is an acceptable alternative
to the investigation of unusual fluctuations.
4. Which of the following matters would an auditor least likely b. Plausible relationships among data may reasonably
consider when setting the direction of the audit? be expected to exist and continue in the absence of
a. The selection of the engagement team and the known conditions to the contrary.
assignment of audit work to the team members. c. These procedures cannot replace tests of details of
b. The engagement budget which includes consideration of transactions and balances.
the appropriate amount of time to allot for areas where d. Statistical tests of financial information may lead to the
there may be higher risks of material misstatement. detection of material misstatements in the financial
c. The availability of client personnel and data. statements.

AT – Prelim Rev (875 MCQs) Red Sirug Page 39 of 85


9. Which of the following statements concerning analytical b. The client has recently purchased an expensive copy
procedures is true? machine.
a. Analytical procedures are more efficient, but not more c. Assets costing less than P5,000 are expensed even
effective, than tests of details of transactions. when the expected life exceeds one year.
b. Analytical procedures used as risk assessment d. Inventory is comprised of diamond rings.
procedures use data aggregated at a high level.
c. Analytical procedures can replace tests of controls in 15. The responsibility for the detection and prevention of errors,
gathering audit evidence to support the assessed level of fraud and noncompliance with laws and regulations rests with:
control risk. a. Auditor
d. Analytical procedures usually involve comparisons of b. Client's legal counsel
ratios developed from recorded amounts with ratios c. Internal auditor
developed by management. d. Client management

10. In relation to audit planning, the auditor should document the 16. The following statements relate to the auditor's responsibility
following: for the detection of errors and fraud. Which of the statement
a. The overall audit strategy. is correct?
b. The detailed audit plan. I. Due to the inherent limitations of the audit, there is a
c. Significant changes made during the audit engagement. possibility that material misstatements in the financial
d. All of the answers statements may not be detected.
II. The subsequent discovery of material misstatement of
11. Audit programs generally include procedures to test actual the financial information resulting from fraud or error does
transactions and resulting balances. These procedures are not, in itself, indicate that the auditor failed to follow the
primarily designed to basic principles and essential procedures of an audit.
a. Detect irregularities that result in misstated financial a. I only
statements. b. II only
b. Test the adequacy of internal control. c. Both statements are true
c. Gather corroborative evidence. d. Both statements are false
d. Obtain information of informative disclosures.
17. Which of the following is not an assurance that the auditors
12. An audit process is a well-defined methodology for organizing give to the parties who rely on the financial statements?
an audit to ensure that a. Auditors know how the amounts and disclosures in the
a. The evidence gathered is both sufficient and competent. financial statements were produced.
b. All appropriate audit objectives are specified. b. Auditors gathered enough evidence to provide a
c. All appropriate audit objectives are met. reasonable basis for forming an opinion.
d. All of these c. Auditors give assurance that the financial statements
are accurate.
13. For a particular assertion, control risk is the risk that d. If the evidence allows the auditors to do so, auditors give
a. Controls will not detect a material misstatement that assurance in the form of opinion, as to whether the
occurs. financial statements taken as a whole are fairly presented
b. Audit procedures will fail to detect a weak control system. in conformity with GAAP.
c. The prescribed control procedures will not be applied
uniformly. 18. Material misstatements in financial statements may arise from
d. A material misstatement will occur in the accounting all of the following except
process. a. Fraud
b. Error
14. Which of the following is most likely to require special planning c. Limitations of the audit
considerations related to asset valuation? d. Noncompliance with laws and regulations
a. Accelerated depreciation methods are used for
amortizing the costs of factory equipment. 19. Which of the following is an example of an error?
a. Defalcation

AT – Prelim Rev (875 MCQs) Red Sirug Page 40 of 85


b. Misapplication of accounting policies. a. The responsibility for the prevention and detection of
c. Suppression or omission of the effects of transactions fraud and error rests with management.
from the records or documents. b. The auditor should plan and perform the audit with an
d. Recording of transactions without substance. attitude of professional skepticism, recognizing that
conditions or events may be found that fraud or error may
20. An intentional act by one or more individuals among exist.
management, employees, or third parties which results in c. The likelihood of detecting fraud is ordinarily higher
misrepresentation of financial statements refers to than that of detecting error.
a. Illegal acts d. The auditor is not and can not be held responsible for the
b. Error prevention of fraud and error.
c. Fraud
d. Noncompliance 26. Which of the following is an "error" as distinguished from
"fraud"?
21. The primary factor that distinguishes errors from fraud is a. Lapping
a. Whether the misstatement is perpetrated by an employee b. Embezzlement of company's fund
or by a member of management c. Clerical mistakes in the processing of transactions
b. Whether the underlying cause of misstatement is d. Window dressing
intentional or unintentional.
c. Whether the misstatement is concealed. 27. Which of the following could be an example of fraud?
d. Whether the underlying cause of misstatement relates to a. Misappropriation of assets or group of assets.
misapplication of accounting principles or to clerical b. Errors in the application of the accounting principles.
processing. c. Clerical errors in accounting data underlying the financial
statements.
22. The factor that distinguishes an error from fraud is d. Misinterpretation of facts that existed when financial
a. Materiality. statements were prepared.
b. Intent.
c. Whether it is peso amount or a process. 28. Which of the following is an example of fraudulent financial
d. Whether it is a caused by the auditor or the client. reporting?
a. Misappropriating collections on accounts receivable
23. Fraudulent financial reporting is often called b. Using inappropriate assumptions in accounting
a. Misappropriation or theft of assets estimate
b. Management fraud c. Stealing inventory
c. Defalcation d. Payments to fictitious employees or vendors
d. Employee fraud
29. These refer to events or conditions that may indicate an
24. In comparing management fraud with employee fraud, the incentive or pressure to commit fraud or provide an
auditor's risk of failing to discover the fraud is opportunity to commit fraud.
a. Greater for employee fraud because of the larger number a. Fraud conditions
of employees in the organization. b. Fraud risk factors
b. Greater for management fraud because of c. Fraudulent activities
management's ability to override existing internal d. Fraud events
controls.
c. Greater for employee fraud because of the higher crime 30. The following are fraud risk factors except:
rate among blue collar workers. a. Incentive or pressure to commit fraud
d. Greater for management fraud because managers are b. Opportunity to commit fraud
inherently smarter than employees. c. Attitude or rationalization to commit fraud
d. All of the above
25. Which of the following statements about fraud or error is
incorrect?

AT – Prelim Rev (875 MCQs) Red Sirug Page 41 of 85


31. Certain characteristics or circumstances may increase the
susceptibility of assets to misappropriation. Opportunities to 36. Noncompliance with laws and regulations are also called
misappropriate assets increase due to the following except: a. Irregularities
a. Inventory items that are small in size, of high value, or in b. Illegal acts
high demand. c. Misappropriation
b. Known or anticipated future employee layoffs. d. Defalcation
c. Easily convertible assets, such as bearer bonds,
diamonds, or computer chips. 37. The responsibility for the detection and prevention of
d. Fixed assets which are small in size, marketable, or noncompliance with laws and regulations rests with
lacking observable identification of ownership. a. The auditor
b. The client's legal counsel
32. The auditor is most likely to presume that a high risk of fraud c. The auditor’s legal counsel
exists if d. The client management
a. The client is a multinational company that does business
in numerous foreign countries. 38. The term “noncompliance” as used in PSA 250 refers to acts
b. The client does business with several related parties. of omission or commission by the entity being audited, either
c. Inadequate segregation of duties places an employee intentional or unintentional, which are contrary to the
in a position to perpetrate and conceal thefts. prevailing laws or regulations. Such acts include the following
d. Inadequate employee training results in lengthy EDP except
exception reports each month. a. Transactions entered into by the entity
b. Transactions entered into in the name of the entity
33. All of the following conditions are indicators of possible c. Transactions entered into by the entity on its behalf by its
pressures on an entity except management or employees
a. The industry in which the entity operates is declining. d. Personal misconduct (unrelated to the business
b. There is inadequate working capital due to declining activities of the entity) by the entity’s management or
profits or too rapid expansion. employees.
c. The client is heavily dependent on one or a few products
or customers. 39. As part of audit planning, CPAs should design audit programs
d. There is a significant and prolonged understaffing of for each individual audit and should include audit steps and
the accounting department. procedures to
a. Ensure that only material items are audited.
34. Which of the following is most likely to be a response to the b. Provide assurance that the objectives of the audit are
auditor's assessment that the risk of material misstatement met.
due to fraud for the existence of inventory is high? c. Detect and eliminate fraud.
a. Observe test counts of inventory at certain locations d. Increase the amount of management information
on an unannounced basis. available.
b. Perform analytical procedures rather than taking test
counts. 40. How can the audit program best be described at the beginning
c. Request that inventories be counted prior to year end. of the audit process?
d. Request that inventory counts at the various locations be a. Conclusive
counted on different dates so as to allow the same b. Confirmed
auditor to be present at every count. c. Optional
d. Temporary
35. The term used to refer to acts of omission or commission by
the entity being audited, either intentional or unintentional, 41. Internal auditing can affect the scope of the external auditor's
which are contrary to the prevailing laws and regulations is audit of financial statements by
a. Misappropriation. a. Decreasing the external auditor's need to perform
b. Fraud. detailed tests.
c. Illegal acts. b. Eliminating the need to observe the physical inventory
d. Noncompliance. taking.

AT – Prelim Rev (875 MCQs) Red Sirug Page 42 of 85


c. Allowing the external auditor to limit his/her audit to the while fraud involves unintentional mistakes or omissions.
performance of substantive test procedures. b. Errors are more likely to provide an indication that an
d. Limiting direct testing by the external auditor to illegal act has occurred.
management assertions not directly tested by internal c. Errors are unintentional mistakes or omissions, while
auditing. fraud involves intentional misstatements.
d. There is no difference as errors and fraud have the same
42. The external auditor should obtain a sufficient understanding meaning.
of the internal audit function because
a. The understanding of the internal audit function is an 47. Which of the following statements is correct relating to the
important substantive test to be performed by the external auditor’s consideration of fraud?
auditor. a. The auditor’s interest in fraud consideration relates
b. The audit programs, working papers, and reports of to fraudulent acts that cause a material misstatement
internal auditors may often be used as a substitute for the of financial statements.
work of the external auditor's staff. b. A primary factor that distinguishes fraud from error is that
c. The procedures performed by the internal audit staff may fraud is always intentional, while errors are generally, but
eliminate the external auditor's need for considering not always, intentional.
internal control. c. While an auditor should be aware of the possibility of
d. The work performed by internal auditors may be a fraud, management, and not the auditor, is responsible
factor in determining the nature, timing, and extent of for detecting fraud.
the external auditor's procedures. d. Fraud always involves a pressure or incentive to commit
fraud, and a misappropriation of assets.
43. In determining whether the work of the internal auditors is
likely to be adequate for purposes of the audit, the external 48. Which of the following factors or conditions is an auditor least
auditor shall evaluate the internal auditor's likely to plan an audit to discover?
a. Efficiency and experience a. High turnover of senior management.
b. Competence and objectivity b. Inadequate monitoring of significant controls.
c. Independence and review skills c. Inability to generate positive cash flows from operations.
d. Training and supervisory skills d. Financial pressures affecting employees.

44. In assessing the technical competence of an internal auditor, 49. Which of the following statements concerning audit planning is
an external auditor most likely would obtain information about incorrect?
the a. Planning is a continual and iterative process.
a. Organizational level to which the internal auditor reports. b. Planning is a discrete phase of an audit.
b. Quality of working paper documentation, reports, and c. In a recurring audit, planning often begins shortly after (or
recommendations. in connection with) the completion of the previous audit
c. Influence of management on the internal auditor's duties. and continues until the completion of the current audit
d. Entity's commitment to integrity and ethical values. engagement.
d. In planning an audit, the auditor considers the timing of
45. Given that an audit in accordance with generally accepted certain planning activities and audit procedures that are
auditing standards is influenced by the possibility of material to be completed prior to the performance of further audit
errors and fraud, the auditor should conduct the audit with an procedures.
attitude of
a. Professional responsiveness. 50. The generally accepted auditing standards of fieldwork pertain
b. Conservative advocacy. most directly to:
c. Professional skepticism. a. Improving internal control as a result of the audit.
d. Objective judgment. b. Due professional care in the performance of the audit.
c. The required training and proficiency of the auditors.
46. The primary difference between financial statement errors and d. The planning of the audit.
fraud is that
a. Errors are intentional misstatements by management, 51. With respect to the auditor's planning of a year-end

AT – Prelim Rev (875 MCQs) Red Sirug Page 43 of 85


examination, which of the following statements is always true? b. Auditors must specifically consider fraud risk from
a. An engagement should not be accepted after the fiscal overstating liabilities.
year-end. c. Auditors must specifically consider fraud risk from
b. It is an acceptable practice to carry out substantial management override of controls.
parts of the examination at interim dates. d. All of the above are true
c. An inventory count must be observed at the balance
sheet date. 56. Which of the following is an example of fraudulent financial
d. The client's audit committee should not be told of the reporting?
specific audit procedures that will be performed. a. The treasurer diverts customer payments to his personal
due, concealing his actions by debiting an expense
52. An initial (first-time) audit requires more audit time to complete account, thus overstating expenses.
than a recurring audit. One of the reasons for this is that b. Company management changes inventory count tags
a. New auditors are usually assigned to an initial audit. and overstates ending inventory, while understating
b. A larger proportion of customer accounts receivable need cost of goods sold.
to be confirmed on an initial audit. c. An employee steals inventory and the “shrinkage” is
c. The client's business, industry, and internal control recorded in cost of goods sold.
are unfamiliar to the auditor and need to be carefully d. An employee steals small tools from the company and
studied. neglects to return them; the cost is reported as a
d. Predecessor auditors need to be consulted. miscellaneous operating expense.

53. Which of the following is not required by PSA No. 315, 57. The type of transactions that ordinarily have a high inherent
"Consideration of Fraud in a Financial Statement Audit"? risk because they involve management judgments or
a. Conduct a continuing assessment of the risks of material assumptions are referred to as
misstatement due to fraud throughout the audit. a. Estimation transactions.
b. Conduct a discussion by the audit team of the risks of b. Nonroutine transactions.
material misstatement due to fraud. c. Related-party transactions.
c. Conduct inquiries of the audit committee as to their views d. Routine transactions.
about the risks of fraud and their knowledge of any fraud
or suspected fraud. 58. Professional skepticism
d. Conduct the audit with professional skepticism, a. Assumes that management is either dishonest or
which includes an attitude that assumes balances are assumes unquestioned honesty.
incorrect until verified by the auditor. b. Either assumes that management is honest or dishonest.
c. Neither assumes that management is dishonest nor
54. Inherent risk is defined as the susceptibility of an account assumes unquestioned honesty.
balance or class of transactions to error that could be material d. None of the above is a correct statement
assuming that there were no related internal controls. Of the
following conditions, which one does not increase inherent risk? 59. Which of the following conditions identified during fieldwork of
a. The board of directors approved a substantial bonus for an audit is most likely to affect the auditor’s assessment of the
the president and chief executive officer, and also risk of misstatement due to fraud?
approved an attractive stock option plan for themselves. a. Year-end adjusting journal entries.
b. The client has entered into numerous related party b. Checks for significant amounts outstanding at year-end.
transactions during the year under audit. c. Missing documents.
c. Internal control over shipping, billing, and recording d. Computer generated documents.
of sales revenue is weak.
d. The client has lost a major customer accounting for 60. Which of the following best describes what is meant by the
approximately 30% of annual revenue. term “fraud risk factor?”
a. Factors whose presence indicates that the risk of fraud is
55. Which of the following is true? high.
a. Auditors are responsible for detecting all fraudulent b. Factors whose presence requires modification of planned
financial reporting. audit procedures.

AT – Prelim Rev (875 MCQs) Red Sirug Page 44 of 85


c. Factors whose presence often have been observed in control risk.
circumstances where frauds have occurred.
d. Reportable conditions identified during an audit. 65. When considering fraud risk factors relating to management’s
characteristics, which of the following is least likely to indicate
61. Which of the following may cause management to intentionally a risk of possible misstatement due to fraud?
understate profits? a. Failure to correct known reportable conditions on a timely
a. Management wants to create "cookie jar" reserves for a basis.
rainy day. b. Use of unusually conservative accounting practices.
b. The company is under scrutiny by tax authorities. c. Nonfinancial management’s preoccupation with the
c. The company is suffering a large loss and wants to take a selection of accounting principles.
"big bath." d. Significant portion of management’s compensation
d. All of the above represented by bonuses based upon achieving unduly
aggressive operating results.
62. Which of the following characteristics most likely would
heighten an auditor’s concern about the risk of intentional 66. With respect to errors and fraud, the auditor should plan to
manipulation of financial statements? a. Search for errors or fraud that would have a material
a. Turnover of senior accounting personnel is low. effect on the financial statements.
b. Insiders recently purchased additional shares of the b. Discover errors or fraud that would have a material
entity’s stock. effect on the financial statements.
c. Management places substantial emphasis on meeting c. Search for errors that would have a material effect and for
earnings projections. fraud that would have either material or immaterial effects
d. The rate of change in the entity’s industry is slow. on the financial statements.
d. Search for fraud that would have a material effect and for
63. Which of the following statements reflects an auditor’s errors that would have either material or immaterial
responsibility for detecting misstatements due to errors and effects on the financial statements.
fraud?
a. An auditor is responsible for detecting employee errors 67. Auditor responsibility for identifying "direct effect" illegal acts
and simple fraud, but not for discovering fraud involving differs from their responsibility for detecting
employee collusion or management override. a. Errors.
b. An auditor should design the audit to provide b. Indirect-effect illegal acts.
reasonable assurance of detecting misstatements c. Fraud.
due to errors and fraud that are material to the d. Management fraud.
financial statements.
c. An auditor should plan the audit to detect misstatements 68. Management’s attitude toward aggressive financial reporting
due to errors and fraud that are caused by departures and its emphasis on meeting projected profit goals most likely
from GAAP. would significantly influence an entity’s control environment
d. An auditor is not responsible for detecting misstatements when
due to errors and fraud unless the application of GAAS a. External policies established by parties outside the entity
would result in such detection. affect its accounting practices.
b. Management is dominated by one individual who is
64. An auditor is required to obtain a basic understanding of the also a shareholder.
client's internal control to plan the audit. The auditor may then c. Internal auditors have direct access to the board of
decide to perform tests of controls on all internal control directors and the entity’s management.
procedures d. The audit committee is active in overseeing the entity’s
a. That would aid in preventing fraud. financial reporting policies.
b. Documented in the flowchart.
c. Considered to be weaknesses that might allow errors to 69. Which of the following is least likely to be required on an audit?
enter the accounting system. a. Review accounting estimates for biases.
d. Considered to be strengths for which the auditor b. Test appropriateness of journal entries and adjustment.
desires further reduction in the assessed level of

AT – Prelim Rev (875 MCQs) Red Sirug Page 45 of 85


c. Make a legal determination of whether fraud has integrity must be taken seriously and pursued vigorously.
occurred. Which of the following may lead the auditor to suspect
d. Evaluate the business rationale for significant unusual management dishonesty?
transactions. a. The client has been named as a defendant in a product
liability suit.
70. Which of the following is most likely to be an overall response b. The client has experienced a decrease in revenue from
to fraud risks identified in an audit? increased import competition.
a. Supervise members of the audit team less closely and c. A new statutory regulation making customer licenses
rely more upon judgment. more difficult to obtain may adversely affect the client's
b. Only use certified public accountants on the engagement. operations.
c. Place increased emphasis on the audit of objective d. The president and chief executive officer of the client
transactions rather than subjective transactions. corporation has held numerous meetings with the
d. Use less predictable audit procedures. controller for the purpose of discussing accounting
practices that will maximize reported profits.
71. The most likely explanation why the auditor’s examination
cannot reasonably be expected to bring all illegal acts by the 75. Which of the following methods may be used to commit
client to the auditor’s attention is that fraudulent financial reporting?
a. Illegal acts are perpetrated by management override of a. Understate liabilities
internal control. b. Fail to provide adequate disclosure
b. Illegal acts by clients often relate to operating c. Overstate revenues
aspects rather than accounting aspects. d. Each of the above can be used to commit fraudulent
c. The client’s internal control may be so strong that the financial reporting
auditor performs only minimal substantive testing.
d. Illegal acts may be perpetrated by the only person in the 76. The absence of which of the following internal controls
client’s organization with access to both assets and the increases the opportunity for fraud?
accounting records. a. Appropriate segregation of duties or independent checks
b. Job applicant screening for employees with access to
72. Which of the following relatively small misstatements most assets
likely could have a material effect on an entity’s financial c. Mandatory vacations for employees with access to assets
statements? d. The absence of any of the above increases the
a. A piece of obsolete office equipment that was not retired. opportunity for fraud
b. An illegal payment to a foreign official that was not
recorded. 77. Whom should auditors contact when they suspect a fraud?
c. A petty cash fund disbursement that was not properly a. Senior management
authorized. b. Expected perpetrators of the fraud
d. An uncollectible account receivable that was not written c. Audit committee of the board of directors
off. d. A and C

73. Which of the following might be considered a "red flag" 78. Auditors would normally interview all but which of the following
indicating possible fraud in a large manufacturing company individuals as part of their assessment of fraud risk?
with several subsidiaries? a. Senior management
a. A consistent record of above average return on b. Audit committee
investment for all subsidiaries c. Various employees whose duties do not include normal
b. The existence of a financial subsidiary financial reporting responsibilities
c. Use of separate bank accounts for payrolls by each d. All of the above
subsidiary
d. Complex sales-transactions and transfers of funds 79. After studying and evaluating a client's existing internal control,
between affiliated companies an auditor has concluded that the policies and procedures are
well-designed and functioning as intended. Under these
74. Warning signs that cause the auditor to question management circumstances, the auditor would most likely

AT – Prelim Rev (875 MCQs) Red Sirug Page 46 of 85


a. Perform further control tests to the extent outlined in the to help assure that the
audit program. a. Audit work is properly supervised.
b. Determine the control policies and procedures that should b. Audit report contains only significant findings.
prevent or detect errors and fraud. c. Audit work is properly planned and documented.
c. Set detection risk at a higher level than would be set d. Work of different auditors is properly coordinated.
under conditions of weak internal control.
d. Set detection risk at a lower level than would be set under 85. An auditor should design the written audit program so that
conditions of weak internal control. a. All material transactions will be selected for substantive
testing.
80. An audit plan is a b. Substantive tests prior to the balance sheet date will be
a. Detailed plan of analytical procedures and all substantive minimized.
tests to be performed in the course of the audit. c. Each account balance will be tested under either tests of
b. Generic document that auditing firms have developed to controls or tests of transactions.
lead the process of the audit through a systematic and d. The audit procedures selected will achieve specific
logical process. audit objectives.
c. Budget of the time that should be necessary to complete
each phase of the audit procedures. 86. In the preparation of an audit program, which of the following
d. Document that provides an overview of the company items is not essential?
and a general plan for the audit work to be a. Assessment of inherent risk
accomplished, timing of the work, and other matters b. The preparation of a budget identifying the costs of
of concern to the audit. resources needed
c. A review of material from prior audits
81. An audit program is d. An understanding of controls established by management
a. A generic document that auditing firms have developed to
lead the process of the audit through a systematic and 87. Which of the following is not a consideration in the
logical process. development of audit programs?
b. An overview of the company and a general plan for the a. Internal control over the recording of plant asset additions
audit work to be accomplished. and repairs and maintenance expenditures is found to be
c. The detailed plan of audit procedures to be weak.
performed in the course of the audit. b. The members of the board of directors are elected by
d. A budget of the time that should be necessary to the stockholders during the annual meeting.
complete each phase of the audit procedures. c. The client is a private university located in Southern
Philippines.
82. Which of the following concepts is most useful in assessing d. The client constructed a major addition to its central
the scope of an auditor's program relating to various accounts? manufacturing facility during the year under audit.
a. Attribute sampling
b. Management fraud 88. The element of the audit planning process most likely to be
c. Materiality agreed upon with the client before implementation of the audit
d. The reliability of information strategy is the determination of the
a. Procedures to be undertaken to discover litigation, claims,
83. One of the primary roles of an audit program is to and assessments.
a. Provide for a standardized approach to the audit b. Timing of inventory observation procedures to be
engagement. performed.
b. Serve as a tool for planning, directing, and c. Evidence to be gathered to provide a sufficient basis for
controlling audit work. the auditor's opinion.
c. Document an auditor's understanding of the internal d. Pending legal matters to be included in the inquiry of the
control. client's attorney.
d. Delineate the audit risk accepted by the auditor.
89. When a CPA expresses an opinion on the financial statements,
84. The principal reason for developing a written audit program is his responsibilities extend to

AT – Prelim Rev (875 MCQs) Red Sirug Page 47 of 85


a. The underlying wisdom of the client's management c. Not independent and, therefore, may not be associated
decision. with the financial statements.
b. Active participation in the implementation of the advice d. Not independent and, therefore, may issue a review
given to the client. report, but may not issue an auditor's opinion.
c. An ongoing responsibility for the client's solvency.
d. Whether the results of the client's operating 95. Solicitation consists of the various means that CPA firms use
decisions are fairly presented in the financial to engage new clients. Which one-of the following would not
statements. be an example of solicitation?
a. Advertisements in the yellow pages of a phone book.
90. The accuracy of information included in the footnotes that b. Accepting new clients that approach the firm.
accompany the audited financial statements of a company c. Taking prospective clients to lunch.
whose shares are traded on a stock exchange is the primary d. Offering seminars on current tax law changes to potential
responsibility of clients.
a. The stock exchange officials.
b. The company's management. 96. Quality control procedures are applicable to the individual
c. The independent auditor. audit engagement. The implementation of such quality control
d. The Securities and Exchange Commission. procedures is responsibility of the:
a. CPA firm.
91. The responsibility for adopting sound accounting policies, b. Engagement team.
maintaining adequate internal control, and making fair c. Quality control reviewer.
representations in the financial statements rests d. Manager assigned to the engagement.
a. With management
b. With the independent auditor 97. Which of the following statements is incorrect?
c. Equally with management and the auditor a. In an audit, the independent auditor attempts to
d. With the internal audit department. corroborate assertions made by the company’s
management in connection with each account, class of
92. Audit standards require an auditor to: transactions, and disclosures found in a set of financial
a. Perform procedures that are designed to detect all statements.
instances of fraud. b. Because of the attest function, financial statements
b. Provide reasonable assurance that the financial are the responsibility of the independent auditor.
statements are not materially misstated. c. The term “materiality” refers to any factor of a size or type
c. Issue an unqualified opinion only when the auditor is that would impact an outside decision-maker’s decision
satisfied that no instances of fraud have occurred. about a set of financial statements.
d. Design the audit program to meet financial statement d. The role of the independent auditor is to gain sufficient
users' expectations concerning fraud. appropriate evidence so as to provide reasonable
assurance that material misstatements do not exist in any
93. Generally, the decision to notify parties outside the client's of the assertions made by management.
organization regarding an illegal act is the responsibility of the
a. Outside legal counsel. 98. Which of the following activities is not prohibited for the CPA
b. Independent auditor. firm's attestation service clients?
c. Management. a. Referral fees on audit jobs.
d. Internal auditors. b. Competitive bidding on audit jobs.
c. Contingent fees on audit jobs.
94. If requested to perform a review engagement for a nonpublic d. Commissions for obtaining client services on audit jobs.
entity in which an accountant has an immaterial direct financial
interest, the accountant is 99. Family and personal relationships between a member of the
a. Independent because the financial interest is immaterial assurance team and a director, an officer or certain
and, therefore, may issue a review report. employees, depending on their role, of the assurance client,
b. Not independent and, therefore, may not issue a least likely create
review report. a. Self-review threat.

AT – Prelim Rev (875 MCQs) Red Sirug Page 48 of 85


b. Self-interest threat. least likely create
c. Intimidation threat. a. Self-interest threat.
d. Familiarity threat. b. Self-review threat.
c. Familiarity threat.
100. The Code of Professional Ethics states, in part, that a CPA d. Intimidation threat.
should maintain integrity and objectivity. Objectivity refers to
the CPA's ability to 6. A professional accountant has a professional duty or right to
a. Insist on all matters regarding audit procedures. disclose confidential information in each of the following,
b. Determine the materiality of items. except:
c. Determine accounting practices that were consistently a. To disclose to BIR fraudulent scheme committed by
applied. the client on payment of income tax.
d. Maintain an impartial attitude on all matters which b. To comply with technical standards and ethics
come under his review. requirements.
c. To comply with the quality review of a member body or
professional body
SET 6 d. To respond to an inquiry or investigation by a member
body or regulatory body.
1. Conflict between financial statement users and auditors often
arises because of the 7. Which of the following is not likely a threat to independence?
a. High cost of performing an audit. a. Acting as an advocate on behalf of an assurance client in
b. Expectation gap. litigation or in resolving disputes with third parties.
c. Technical vocabulary which the auditor uses in the report. b. Long association of a senior member of the assurance
d. Placement of the auditor's report in the back of the team with the assurance client.
client's annual report where it is hard to locate. c. Threat of replacement over a disagreement with the
application of an accounting principle.
2. Which of the following is best considered a fraud? d. Owning immaterial indirect financial interest in an
a. Inability to provide due diligence. audit client.
b. Intentional misrepresentation of financial information.
c. Declining to finish work on client in light of a valid contract.
d. Not acting professionally while performing services. 8. Examples of circumstances that may create self-interest threat
include:
3. Anyone identified to the auditor by name prior to the audit who a. Contingent fees relating to assurance engagements.
is to be the principal recipient of the auditor's report is a b. A direct financial interest or material indirect financial
a. Primary beneficiary. interest in an assurance client.
b. Third party. c. A loan or guarantee to or from an assurance client or any
c. Foreseen beneficiary. of its directors or officers.
d. Secondary beneficiary. d. All of the above

4. A director, an officer or an employee of the assurance client in 9. Examples of circumstances that may create self-review threat
a position to exert direct and significant influence over the least likely include
subject matter of the assurance engagement has been a a. Potential employment with an assurance client.
member of the assurance team or partner of the firm. This b. Preparation of original data used to generate financial
situation least likely create statements or preparation of other records that are the
a. Self-interest threat. subject matter of the assurance engagement.
b. Advocacy threat. c. A member of the assurance team being, or having
c. Intimidation threat. recently been, an employee of the assurance client in a
d. Familiarity threat. position to exert direct and significant influence over the
subject matter of the assurance engagement.
5. A former officer, director or employee of the assurance client d. Performing services for an assurance client that directly
serves as a member of the assurance team. This situation will affect the subject matter of the assurance engagement.

AT – Prelim Rev (875 MCQs) Red Sirug Page 49 of 85


d. All of the above
10. A CPA-lawyer, acting as a legal counsel to one of his audit
client, is an example of 14. Which of the following is incorrect regarding independence?
a. Advocacy threat a. Independence consists of independence of mind and
b. Familiarity threat independence in appearance.
c. Self-interest threat b. Independence is a combination of impartiality,
d. Self-review threat intellectual honesty and a freedom from conflicts of
interest.
11. Examples of circumstances that may create familiarity threat c. Independence of mind is the state of mind that permits
least likely include the provision of an opinion without being affected by
a. A former partner of the firm being a director, officer of the influences that compromise professional judgment,
assurance client or an employee in a position to exert allowing an individual to act with integrity, and exercise
direct and significant influence over the subject matter of objectivity and professional skepticism.
the assurance engagement. d. Independence in appearance is the avoidance of facts
b. Dealing in, or being a promoter of, share or other and circumstances that are so significant a reasonable
securities in an assurance client. and informed third party, having knowledge of all relevant
c. A member of the assurance team having an immediate information, including any safeguards applied, would
family member or close family member who is a director reasonably conclude a firm's or a member of the
or officer of the assurance client. assurance team's integrity, objectivity or professional
d. A member of the assurance team having an immediate skepticism had been compromised.
family member or close family member who, as an
employee of the assurance client, is in a position to exert 15. A CPA firm is considered independent when it performs which
direct and significant influence over the subject matter of of the following services for a publicly-traded audit client?
the assurance engagement. a. Serving as a member of the client's board of directors.
b. Determining which accounting policies will be adopted by
12. The following are modifications made to the IFAC Code to the client.
consider Philippine regulatory requirements and c. Accounting information system design and
circumstances, except implementation.
a. The period for rotation of the lead engagement d. Tax return preparation as approved by the board of
partner was changed from five to seven years. directors.
b. Advertising and solicitation by individual professional
accountants in public practice were not permitted in the 16. In connection with the examination of financial statements, an
Philippines. independent auditor could be responsible for failure to detect a
c. Additional examples relating to anniversaries and material fraud if
websites a. Statistical sampling techniques were not used on the
wherein publicity is acceptable, as provided in boa audit engagement.
resolution 19, series of 2000, were included. b. The fraud was perpetrated by one client employee, who
d. Payment and receipt of commissions were not permitted circumvented the existing internal control.
in the Philippines. c. The auditor planned the work in a hasty and
inefficient manner.
d. Accountants performing important parts of the work failed
13. The network firms are required to be independent of the client to discover a close relationship between the treasurer
a. For assurance engagements provided to an audit and the cashier.
client.
b. For assurance engagements provided to clients that are 17. Which of the following conditions suggests auditor negligence?
not audit clients, when the report is not expressly a. Failure to detect collusive fraud perpetrated by members
restricted for use by identified users. of middle management.
c. For assurance engagements provided to clients that are b. Failure to detect collusive fraud perpetrated by members
not audit clients, when the assurance report is expressly of top management.
restricted for use by identified users. c. Failure to detect errors occurring outside the internal

AT – Prelim Rev (875 MCQs) Red Sirug Page 50 of 85


control structure.
d. Failure to detect material errors under conditions of 22. Auditors would perform the following steps in which order?
weak internal control. a. Determine audit risk; assess control risk; determine
detection risk; set materiality.
18. Which of the following statements is correct? b. Set materiality; assess control risk; determine detection
a. Sufficiency refers to the quality of evidence, while risk; determine audit risk.
appropriateness refers to the quantity of evidence. c. Set materiality; determine audit risk; assess control
b. The reliability of evidence is influenced not by its nature risk; determine detection risk.
but by its source. d. Determine audit risk; set materiality; assess control risk;
c. The performance of consulting services for audit determine detection risk.
clients does not, in and of itself, impair the auditor’s
independence. 23. Which of the following statements is an not an example of an
d. A belief that management and those charged with inherent limitation of internal control?
governance are honest and have integrity relieves the a. The effectiveness of control procedures depends on
auditor of the need to maintain professional scepticism. segregation of duties.
b. Errors may arise from mistakes in judgments.
19. Which of the following statements is correct? c. Most internal controls tend to be directed at routine
a. The fair presentation of audited financial statements in transactions rather than non-routine transactions.
accordance with applicable financial reporting framework d. The cost-benefit relationship is a primary criterion in
is an implicit part of the auditor’s responsibility. designing internal control,
b. Professional judgment can be used as the justification for
the decisions made by the auditor that are not otherwise 24. Compliance with the independence requirement is necessary
supported by the facts and circumstances of the whenever a CPA performs:
engagement or sufficient appropriate evidence. a. Non-assurance services
c. Appropriateness is the measure of the quality of evidence, b. Professional services
that is, its reliability and persuasiveness. c. Tax consultancy services
d. Most CPAs, including those who provide audit and d. Assurance services
tax services, also provide consulting services to their
clients. 25. It is acceptable for the auditor to prepare:
a. The financial statements for the client.
20. Individual CPAs, Firms or Partnerships of CPAs, including b. The notes to financial statements for the client.
partners and staff members thereof shall register with the BOA c. A draft of the financial statements for the client.
and the PRC. Assuming that the application for registration of d. A draft of the financial statements and notes to the
Sisip and Co., CPAs was approved on August 30, 2005, which financial statements for the client.
of the following is true?
a. The registration will expire on Dec. 31, 2007. 26. When an accountant performs more than one level of service,
b. The registration must be renewed on September 30, he generally should issue a report that is appropriate for:
2007. a. The lowest level of service rendered.
c. The registration will expire on August 30, 2007 since the b. A compilation engagement.
validity of the certificate of registration is three years. c. The highest level of service rendered.
d. The registration will expire on Dec. 31, 2007 which is also d. A review engagement.
the last day of renewal of certificate of registration.
27. An accountant who reviews the financial statements should
21. Auditor’s responsibility for detecting noncompliance is limited issue a report stating that a review
to those: a. Is substantially less in scope than an audit.
a. Direct-effect noncompliance. b. Provides negative assurance that the internal control is
b. Material direct-effect noncompliance. functioning as designed.
c. Material indirect-effect noncompliance. c. Provides only a limited assurance that the financial
d. All noncompliance since they affect the financial statements are fairly presented.
statements directly or indirectly. d. Is substantially more in scope than a compilation

AT – Prelim Rev (875 MCQs) Red Sirug Page 51 of 85


28. Which of the following is required to be performed in an audit 33. Which of the following is not true regarding planning in an
but not in review engagement? electronic environment?
a. Complying with the "Code of Professional Ethics for a. The definition of auditing is not changed
Certified Public Accountants" promulgated by the Board b. The purposes of auditing is not changed
of Accountancy c. The procedures used are not changed
b. Planning the engagement d. Auditing standards are not changed
c. Agreeing on the terms of engagement
d. Studying and evaluating internal control structure 34. Which of the following concepts of materiality is incorrect?
a. Materiality is based on quantitative and non-quantitative
29. Which statement is incorrect regarding procedures and factors.
evidence obtained in a review engagement? b. Materiality is a matter of professional audit judgment.
a. The auditor should apply judgment in determining the c. Materiality does not apply if internal control is highly
specific nature, timing and extent of review procedures. effective.
b. The auditor should apply the same materiality d. Materiality is more closely related to the fieldwork and
considerations as would be applied if an audit opinion on reporting standards than to general standards.
the financial statements were being given.
c. There is a greater risk that misstatements will not be 35. Which of the following statements is incorrect about materiality?
detected in an audit than in a review. a. The concept of materiality recognizes that some matters
d. The judgment as to what is material is made by reference are important for fair presentation of financial statements
to the information on which the auditor is reporting and in conformity with GAAP, while other matters are not
the needs of those relying on that information, not to the important.
level of assurance provided. b. An auditor considers materiality for planning
purposes in terms of the largest aggregate level of
30. Which of the following is not a document or record that should misstatements that could be material to any one of
be examined early in the engagement? the financial statements.
a. Corporate charter and by-laws. c. Materiality judgments are made in light of surrounding
b. Contracts. circumstances and necessarily involve both quantitative
c. Management letter. and qualitative judgments.
d. Minutes of board of directors' and stockholders' meetings. d. An auditor’s consideration of materiality is influenced by
the auditor’s perception of the needs of a reasonable
31. Philippine Standards on Auditing require auditors to assess person who will rely on the financial statements.
the risk of material misstatements due to fraud
a. For first-time audits. 36. After discovering that a related-party transaction exists, the
b. Sufficient to find any frauds which may exist. auditor should be aware that the
c. For every audit. a. Transaction is assumed to be outside the ordinary course
d. Whenever it would be appropriate. of business.
b. Substance of the transaction could be significantly
32. In connection with the examination of financial statements, an different from its form.
independent auditor could be responsible for failure to detect a c. Adequacy of disclosure of the transaction is secondary to
material fraud if its legal form.
a. Accountants performing important parts of the work failed d. Financial statements should recognize the legal form of
to discover a close relationship between the treasurer the transaction rather than its substance.
and the cashier.
b. The auditor planned the work in a hasty and 37. Auditors focus on
inefficient manner. a. Areas where the risk of material errors and irregularities
c. Statistical sampling techniques were not used on the is least.
audit engagement. b. Areas where the risk of material errors and
d. The fraud was perpetrated by one client employee, who irregularities is greatest.
circumvented the existing internal control. c. All areas equally.

AT – Prelim Rev (875 MCQs) Red Sirug Page 52 of 85


d. A random selection of all areas. is not adequate.
b. Will not be affected since the auditor must arrive at an
38. Audit risk components consist of inherent, control and independently determined opinion.
detection risks. Which of them is are dependent variable(s)? c. Must be increased to support the auditor's belief.
a. Inherent risk d. Is not determinable.
b. Control risk
c. Detection risk 44. Why should the auditor plan more work on individual accounts
d. Inherent and control risks as lower acceptable levels of both audit risk and materiality
are established?
39. The risk that the audit will fail to uncover a material a. To find smaller errors
misstatement is eliminated b. To find larger errors
a. When the auditor has complied with generally accepted c. To increase the tolerable error in the accounts
auditing standards. d. To decrease the risk of overreliance
b. If a client has strong internal controls.
c. If a client follows generally accepted accounting 45. Which of the following most accurately summarizes what is
principles. meant by the term “material misstatement?”
d. Under no circumstances. a. Fraud and direct-effect illegal acts.
b. Fraud involving senior management and material fraud.
40. The probability of an auditor's procedures leading to the c. Material error, material fraud, and certain illegal acts.
conclusion that a material error does not exist in an account d. Material error and material illegal acts.
balance when, in fact, such error does exist is referred to as
a. Prevention risk. 46. The risk of fraudulent financial reporting increases in the
b. Inherent risk. presence of
c. Control risk. a. Substantial increases in sales.
d. Detection risk. b. Incentive systems based on operating income.
c. Improved control systems.
41. Which of the following does an auditor least likely perform in d. Frequent changes in suppliers.
assessing audit risk?
a. Understand the economic substance of significant 47. Which of the following is most likely to be considered a risk
transactions completed by the client. factor relating to fraudulent financial reporting?
b. Understand the entity and the industry in which it a. Domination of management by top executives.
operates. b. Negative cash flows from operations.
c. Gather audit evidence in support of recorded c. Large amounts of cash processed.
transactions. d. Small high-dollar inventory items.
d. Obtain an understanding of the client's system of internal
control. 48. When planning the audit, if the auditor has no reason to
believe that illegal acts exist, the auditor should
42. Which of the following is most likely to be an overall response a. Make inquiries of management regarding their
to fraud risks identified in an audit? policies and their knowledge of violations, and then
a. Supervise members of the audit team less closely and rely on normal audit procedures to detect errors,
rely more upon judgment. irregularities, and illegalities.
b. Use less predictable audit procedures. b. Still include some audit procedures designed specifically
c. Use only certified public accountants on the engagement. to uncover illegalities.
d. Place increased emphasis on the audit of objective c. Ignore the topic.
transactions rather than subjective transactions. d. Include audit procedures which have a strong probability
of detecting illegal acts.
43. If the auditor is convinced that the client has an excellent
internal control structure, the amount of audit evidence to be 49. Which statement best describes the emphasis of the systems
gathered. and substantive approaches in the audit plan?
a. Can be significantly less than where internal control a. A thoroughly designed systems approach to auditing can

AT – Prelim Rev (875 MCQs) Red Sirug Page 53 of 85


eliminate the need for any substantive procedures.
b. The systems approach focuses on detailed testing of 55. An audit program provides proof that
specific accounts for accuracy, while the substantive a. Sufficient appropriate evidence was obtained.
approach is the testing controls to make sure they are b. The work was adequately planned.
effective. c. There was a proper study and evaluation of internal
c. The systems approach focuses on testing controls to control.
make sure they are effective, while the substantive d. There was compliance with generally accepted standards
approach is the detailed testing of specific accounts of reporting.
for accuracy.
d. The systems approach focuses on the use of computer 56. The audit program usually cannot be finalized until the
systems to aid in the audit while the substantive approach a. Reportable conditions have been communicated to the
focuses on more manual tests. audit committee of the board of directors.
b. Engagement letter has been signed by the auditor and
50. Audit programs should be designed so that the client.
a. Most of the required procedures can be performed as c. Consideration of the entity’s internal control has
interim work. been completed.
b. The audit evidence gathered supports the auditor’s d. Search for unrecorded liabilities has been performed and
conclusions. documented.
c. Inherent risk is assessed at a sufficiently low level.
d. The auditor can make constructive suggestions to 57. A person or firm possessing special skill, knowledge and
management. experience in a particular field excluding accounting and
auditing.
51. The decision as to how much evidence to be accumulated for a. Quality control reviewer
a given set of circumstances is: b. Multiskilled personnel
a. Provided in the Philippines Standards on Auditing. c. Expert
b. Provided by following GAAP d. Taxation specialist
c. One requiring professional judgment
d. Determined by statistical analysis 58. The Code of Ethics for Professional Accountants in the
Philippines is applicable to professional services in the
52. The revised Code of Ethics is mandatory for all CPAs and is Philippines on or before:
applicable to professional services performed in the a. December 31, 2009
Philippines on or: b. July 1, 2004
a. Before June 30. 2008 c. June 30, 2008
b. After June 30, 2008 d. June 30, 2009
c. Before January 1,2008
d. After January 1, 2008 59. Which part of the Code of Ethics applies to professional
accountants in public practice?
53. Which of the following is not explicitly referred to in the Code a. Part A
of Ethics as source of technical standards? b. Part B
a. Commission on Audit (COA) c. Part A and Part B
b. Auditing and Assurance Standards Council (AASC) d. Part C
c. Securities and Exchange Commission (SEC)
d. Relevant legislation 60. This fundamental ethical principle prohibits association of
professional accountants with reports, returns,
54. In designing written audit programs, an auditor should communications and other information that contains materially
establish specific audit objectives that relate primarily to the false or misleading information or statements.
a. Financial statement assertions. a. Integrity
b. Timing of audit procedures. b. Objectivity
c. Cost-benefit of gathering evidence. c. Professional competence and due care
d. Selected audit techniques. d. Professional behavior

AT – Prelim Rev (875 MCQs) Red Sirug Page 54 of 85


errors or irregularities.
61. The principle professional competence and due care imposes b. It is usually easier for the auditor to uncover irregularities
which of the following obligations on professional accountants? than errors.
a. To maintain professional knowledge and skill at the c. It is usually easier for the auditor to uncover errors
level required to ensure that a client or employer than irregularities.
receives competent professional service. d. Usually, none of the above statements is true.
b. To comply with relevant laws and regulations and avoid
any action that discredits the profession. 67. The audit should not assume that management is dishonest,
c. Not to override or compromise his professional or but the possibility of dishonesty must be considered." This is
business judgment because of bias, conflict of interest or an example of
undue influence of others. a. Due diligence.
d. To be fair and truthful. b. Unprofessional behavior.
c. An attitude of professional skepticism.
62. Attainment of professional competence requires the following, d. An ethical requirement.
except:
a. Initially, a high standard of education. 68. In discovering material management fraud and an equally
b. Specific education, training, and examination in material error, the audit plan
professionally relevant subjects. a. Cannot be expected to provide the same degree of
c. Whether prescribed or not, a period of work experience. assurance.
d. A continuing awareness and an understanding of b. Provide no assurance of detecting either.
relevant technical professional and business c. Should be expected to provide the same degree of
developments. assurance.
d. Should provide complete assurance of detection.
63. A draft of statement, studies or standards should be discussed
by the Council en banc. How many members of the AASC are 69. In comparing management fraud with employee fraud, the
required to approve the draft for exposure? auditor's risk of failing to discover the fraud is
a. Eight a. Greater for employee fraud because of the larger number
b. Ten of employees in the organization.
c. Twelve b. Greater for employee fraud because of the higher crime
d. Majority rate among blue collar workers.
c. Greater for management fraud because of
64. How many members of the AASC are needed to approved the management's ability to override existing internal
exposed draft as PSA controls.
a. At least 8 d. Greater for management fraud because managers are
b. At least 10 inherently smarter than employees.
c. At least 12
d. Majority of the regular members 70. The risk that the audit will fail to uncover a material
misstatement is eliminated
65. An auditor's examination performed in accordance with a. When the auditor has complied with generally accepted
generally accepted auditing standards generally should auditing standards.
a. Be expected to provide assurance that illegal acts will be b. If client has good internal control.
detected where internal control is effective. c. If client follows generally accepted accounting principles.
b. Be relied upon to disclose violations of truth-in-lending act. d. Under no circumstances.
c. Encompass a plan to search actively for illegalities which
relate to operating aspects. 71. The practitioner’s report on an assurance engagement should
d. Not be relied upon to provide assurance that illegal always include the following, except
acts will be detected. a. A description of the engagement and identification of the
subject matter
66. Which of the following statements is true? b. Identification of the standards under which the
a. It is usually equally difficult for the auditor to uncover engagements was conducted.

AT – Prelim Rev (875 MCQs) Red Sirug Page 55 of 85


c. Identification of the criteria. 76. The term professional accountant in public practice includes
d. Reference to the work of an expert. the following, except:
a. A sole proprietor providing professional services to a
72. Which of the following is required if the professional client.
accountant uses experts who are not professional accountants? b. Each partner or person occupying a position similar to
a. The professional accountant is discouraged to engage that of a partner staff in a practice providing professional
the services of experts who are not a professional services to a client.
accountant. c. Professional accountants employed in the public
b. The ultimate responsibility for the professional service is sector having managerial responsibilities.
assumed by the expert who is not a professional d. A firm of professional accountants in public practice.
accountant.
c. The professional accountant must take steps to see 77. The term receiving accountant includes the following, except;
that such experts are aware of the ethical a. A professional accountant in public practice to whom
requirements of the profession. existing accountant has referred tax engagement.
d. Experts who are not professional accountants need not b. A professional accountant in public practice to whom the
be informed of ethical requirements because they are not client of the existing accountant has referred audit
members of the Accountancy profession. engagement.
c. A professional accountant in public practice who is
73. Which of the following is expected of AASC to do? consulted in order to meet the needs of the client.
a. AASC should normally expose its opinion on specific d. A professional accountant in public practice
queries from a practicing CPA. currently holding an audit appointment or carrying
b. AASC should normally expose a proposed interpretation out accounting, taxation, consulting or similar
of statements. professional services for a client.
c. To make the statements on Philippine Standards on
Auditing operative, the final statement shall be 78. A primary purpose for establishing a code of ethics within a
submitted to the Board of Accountancy for approval. professional organization is to:
d. When it is deemed necessary to expose a statement for a a. Demonstrate the acceptance of responsibility Jo the
comment on proposed interpretations of statements, the interest of those served by the profession.
exposure period is understandably shorter than those of b. Reduce the likelihood that members of the profession will
the regular drafts of standards. be sued for substandard work.
c. Ensure that all members of the profession posses
74. Required auditor communication to the Audit Committee approximately the same level of competency.
concerning noncompliance with laws and regulations that d. Require members of the profession to exhibit loyalty in all
were detected includes: matters pertaining to the affairs of the organization.
a. All those which are not adequately addressed by
management. 79. The communication to the public of facts about a professional
b. All those that constitute management fraud. accountant which are not designed for the deliberate
c. All material items. promotion of that professional accountant.
d. Any of such acts. a. Publicity
b. Indirect promotion
75. Firm includes the following except: c. Advertising
a. A sole practicing professional accountant, d. Solicitation
b. An entity that controls a partnership of professional
accountants. 80. Advertising, as defined in the Code of Ethics, means
c. An entity controlled by a partnership of professional a. The communication to the public of facts about a
accountants. professional accountant which are not designed for the
d. A sole practitioner, partnership or corporation of deliberate promotion of that professional accountant.
professional accountants. b. The approach to a potential client for the purpose of
offering professional services.

AT – Prelim Rev (875 MCQs) Red Sirug Page 56 of 85


c. The communication to the public of information as to c. Any disputes over significant accounting issues have
the services or skills provided by professional been settled to the auditor's satisfaction.
accountants in public practice with a view to d. Informative disclosures in the financial statements but not
procuring professional business. necessarily in the notes to financial statements are to be
d. Any of the given choices. regarded as reasonably adequate.

81. Which of the following is least likely the basis of determining 86. A CPA should maintain objectivity and be free of conflicts of
audit fees? interest when performing:
a. The skill and knowledge required for the type of work a. All attestation services, but not other professional
involved. services.
b. The degree of responsibility and urgency that the work b. All attestation and tax services, but not other professional
entails. services.
c. The expected outcome of the engagement. c. Audits, but not any other professional services.
d. The required level of training and experience of the d. All professional services.
person engaged on the work.
87. Which of the following has primary responsibility for the
82. Though PSAs do not provide "hard and fast rules," they performance of an audit?
provide subjective guidance which allow the auditors to: a. The partner in charge of the engagement
a. Only apply those standards that are important to the audit. b. The senior assigned to the engagement
b. Use adequate professional judgment when applying c. The managing partner of the firm
the standards. d. The manager assigned to the engagement
c. Tailor their audit to procedures requested by
management. 88. The most common type of audit report contains a(n):
d. Accurately interpret the Code of Ethics for CPAs. a. Unqualified opinion.
b. Qualified opinion.
83. Which one of the following is not a key attribute that is c. Adverse opinion.
essential to perform an assurance service? d. Disclaimer of opinion.
a. Independence
b. Accounting skills 89. Assurance services involve all the following except:
c. Established criteria or standards a. Improving the quality of information for decision purposes.
d. Subject matter knowledge b. Improving the quality of the decision model used.
c. Improving the relevance of information.
84. Upon completion of a typical audit, the auditor has d. Implementing a system that improves the processing
a. No assurance that all material errors and fraud have been of information.
found.
b. A low level of assurance that all material errors and fraud 90. Which of the following is the broadest and most inclusive
have been found. concept?
c. High level of assurance that all material errors and a. Audits of financial statements.
fraud have been found. b. Internal control audit.
d. Total assurance that al material errors and fraud have c. Assurance services.
been found. d. Compilation services.

85. An investor, while reading the financial statements of Star 91. Whenever a CPA professional is engaged to perform an audit
Corporation, learned that the statements are accompanied by of financial statements according to Philippine Standards on
an unqualified auditor's report. From this the investor may Auditing, he required to comply with those standards in order
conclude that: to
a. The auditor has ascertained that Silver's financial a. Eliminate audit risk.
statements have been prepared accurately. b. Eliminate the professional judgment in resolving audit
b. The auditor is satisfied that Silver is operationally efficient. issues.
c. Have a measure of the quality of audit performance.

AT – Prelim Rev (875 MCQs) Red Sirug Page 57 of 85


d. To reduce the audit program to be prepared by the a. Audits, review, and compilations
auditor. b. Reviews, compilations, and other attestation services
c. Audits, compilations, and other attestation services
92. Certain fundamental beliefs called "postulates" underlie d. Audits, reviews, and other attestation services
auditing theory. Which of the following is not a postulate of
auditing? 98. Which of the following types of audit uses as its criteria laws
a. Economic assertions can be verified. and regulations?
b. The auditor acts exclusively as an auditor. a. Financial statement audit
c. An audit has a benefit only to the owners. b. Operational audit
d. No long-term conflict exists between the auditor and the c. Compliance audit
management of the entity under audit. d. Financial audit

93. In all cases, audit reports must 99. Which of the following least likely limits the auditor’s ability to
a. Communicate the auditor’s finding to the general public. detect material misstatement?
b. Be signed by the individual who performed the audit a. The inherent limitations of any accounting and internal
procedures. control system.
c. Certify the accuracy of the quantitative information which b. Most audit evidences are conclusive rather than
was audited. being persuasive.
d. Inform readers of the degree of correspondence c. Audit is based on testing
between the quantifiable information and the d. Audit procedures that are effective in detecting ordinary
established criteria. misstatements are ineffective in detecting intentional
misstatements.
94. The framework for auditing and related services as addressed
by PSA excludes 100. Examples of circumstances that may create familiarity threat
a. Review least likely include
b. Compilation a. A member of the assurance team having an immediate
c. Tax services family member or close family member who is a director
d. Agreed upon procedure or officer of the assurance client.
b. A member of the assurance team having an immediate
95. It refers to the level of auditor’s satisfaction as to the reliability family member or close family member who, as an
of an assertion being made by one party for use by another employee of the assurance client, is in a position to exert
party. direct and significant influence over the subject matter of
a. Confidence level the assurance engagement.
b. Assurance level c. A former partner of the firm being a director, officer of the
c. Reasonableness level assurance client or an employee in a position to exert
d. Tolerable level direct and significant influence over the subject
matter of the assurance engagement.
d. Dealing in, or being a promoter of, share or other
96. Which of the following is true of the report based on agreed-
securities in an assurance client.
upon-procedures?
a. The report is restricted to those parties who have
SET 7
agreed to the procedures to be performed.
b. The CPA provides the recipients of the report limited
assurance as to reasonableness of the assertion(s) 1. In "auditing" accounting data, the auditor is concerned with
presented in the financial information. a. Analyzing the financial information to be sure that it
c. The report states that the auditor has not recognized any complied with government requirement.
basis that requires revision of financial statements. b. Determining if fraud has occurred.
d. The report should state that the procedures performed c. Determining whether recorded information properly
are limited to analytical procedures and inquiry. reflects the economic events that occurred during the
accounting period.
97. The three types of attestation services are:

AT – Prelim Rev (875 MCQs) Red Sirug Page 58 of 85


d. Determining if taxable income has been calculated 7. The audit of historical financial statements should be
correctly. conducted by the CPA professionals in accordance with
a. The auditor's judgment.
2. Which one of the following is an example of management b. The audit program.
expectations from the independent auditors? c. Philippine Financial Reporting Standards.
a. An expert providing a written communication as the d. Philippine Standards on Auditing.
product of the engagement.
b. Individuals who perform day-to-day accounting functions 8. In determining the primary responsibility of the external auditor
on behalf of the company. for an audit of a company's financial statements, the auditor
c. AN active participant in management decision-making. owes primary allegiance to:
d. An internal source of expertise on financial and other a. The management of the audit client because the auditor
matters. is hired and paid by management.
b. The audit committee of the audit client because that
3. Broadly defined, the subject matter of any audit consists of committee is responsible for coordinating and reviewing
a. Financial statements. all audit activities within the company.
b. Assertions. c. Stockholders, creditors, and the investing public.
c. Operating data. d. The Auditing and Assurance Standards Council, because
d. Economic data. it determines auditing standards and auditor's
responsibility.
4. An engagement in which a CPA firm arranges for a critical
review of its practices by another CPA firm. 9. Assurance services may include which of the following?
a. Attestation engagement a. Attesting to financial statements
b. Peer review engagement b. Examination of the economy and efficiency of
c. Quality control engagement governmental operations
d. Quality assurance engagement c. Evaluation of a division's performance for management
d. All of the given choices
5. The risk associated with a company's survival and profitability
is referred to as: 10. The auditor of financial statements must make very difficult
a. Information risk interpretations regarding authoritative literature. Additionally,
b. Business risk the auditor must
c. Control risk a. Proceed beyond PFRS to assess how the economic
d. Detection risk activity is portrayed in the financial statements.
b. Force management to make certain decisions regarding
6. An operational audit differs in many ways from an audit of their financial statements.
financial statements. Which of the following is the best c. Disregard independence in order to find the underlying
example of these differences? truth of the evidence.
a. Operation audits do not ordinarily result in the preparation d. Establish new criteria by which financial statements may
of a report. be compared.
b. The usual audit of financial statement covers the four
basic financial statements whereas the operational audit 11. Which one of the following is not a part of the attest process?
is usually limited either the balance sheet or the income a. Evaluating evidence against objective criteria
statement. b. Gathering evidence about assertions
c. The boundaries of an operation audit are often drawn c. Proving the accuracy of the books and records
from an organization chart and are not limited to a d. Communicating the conclusions reached
single accounting period.
d. The operational audit deals with operating profit while 12. Which one of the following is not a reason why the users of
financial audit considers both the operating and net financial statements desire for an independent assessment of
profits. the financial statement presentation?
a. Complexity of transactions affecting the financial
statements

AT – Prelim Rev (875 MCQs) Red Sirug Page 59 of 85


b. Lack of criteria on which to base information 18. Competence as a certified public accountant includes all of the
c. Remoteness of the user from the organization following except
d. All of them are potential reasons a. Consulting others if additional technical information is
needed.
13. An audit which determines whether organizational policies are b. Possessing the ability to supervise and evaluate the
being followed and whether external mandates are being met quality of staff work.
is referred to as c. Having the technical qualifications to perform an
a. A financial audit engagement.
b. A compliance audit. d. Warranting the infallibility of the work performed.
c. An operational audit.
d. None of the above 19. Which one of the following attributes is required of an auditor
in relation to audit clients?
14. Which of the following factors most likely would cause a CPA a. Loyalty
to decline a new audit engagement? b. Rationalization
a. The CPA does not understand the entity's operations and c. Independence
industry. d. Bias
b. The CPA is unable to review the predecessor auditor's
working papers. 20. To be independent, the auditor:
c. Management is unwilling to permit inquiry of its legal a. Must be impartial when dealing with the client.
counsel. b. Cannot place any reliance on the client's verbal and
d. Management acknowledges that the entity has had written assertions.
recurring operating losses. c. Is responsible only to third-party users of the financial
statements.
15. Auditing standards are d. Cannot perform any consulting services for an audit client.
a. Statutory in nature.
b. Rules imposed by the Securities and Exchange 21. Practitioner's independence:
Commission. a. Minimizes risk.
c. General guidelines to help auditors. b. Defends against liability
d. Rules imposed by the PICPA. c. Helps achieve public confidence
d. Achieves compliance with the standards of fieldwork.
16. Which of the following best describes what is meant by
generally accepted auditing standards? 22. The exercise of due professional care requires that an auditor
a. Pronouncements issued by the Auditing and Assurance a. Uses error-free judgment.
Standards Council. b. Considers internal control, including tests of controls.
b. Rules acknowledged by the accounting profession c. Examines all corroborating evidence available.
because of their universal compliance. d. Be responsible for fulfilling his or her duties
c. Procedures to be used to gather evidence to support diligently and carefully.
financial statements.
d. Measures of the quality of the auditor's performance. 23. The exercise of due professional care requires that an auditor
a. Critically review the judgment exercised at every
17. Generally Accepted Auditing Standards (GAAS) and level of supervision.
Philippine Standards on Auditing (PSA) should be looked b. Attain the proper balance of professional experience and
upon by practitioners as formal education.
a. Ideals to work towards, but which are not achievable. c. Reduce control risk below the maximum.
b. Benchmarks to be used on all audits, reviews, and d. Examine all available corroborating evidence.
compilations.
c. Maximum standards which denote excellent work. 24. The exercise of due professional care requires that an auditor
d. Minimum standards of performance which must be a. Use error-free judgment.
achieved on each audit engagement. b. Consider internal control, including tests of controls.
c. Critically review the work done at every level of

AT – Prelim Rev (875 MCQs) Red Sirug Page 60 of 85


supervision. 30. Which of the following statements best describes the primary
d. Examine all corroborating evidence available. purpose of Philippines Standards on Auditing?
a. They are authoritative statements, enforced through the
25. An auditor who accepts an audit engagement and does not Code of Ethics for Professional Accountants, that are
possess the industry expertise of the business entity, should intended to limit the degree of auditor judgment.
a. Engage financial experts familiar with the nature of the b. They are procedural outlines which are intended to
business entity. narrow the areas of inconsistency and divergence of
b. Obtain a knowledge of matters that relates to the auditor opinion.
nature of the entity's business. c. They are guides intended to set forth auditing procedures
c. Refer a substantial portion of the audit to another CPA that are applicable to a variety of situations.
who will act as the principal auditor. d. They are interpretations which are intended to clarify
d. First inform management that an unqualified opinion the meaning of "generally accepted auditing
cannot be issued. standards."

26. Which of the following underlies the application of generally 31. In financial statement audits, the audit process should be
accepted auditing standards, particularly the standards of conducted in accordance with
fieldwork and reporting? a. Philippine Financial Reporting Standards
a. Element of corroborating evidence b. International Accounting Standards
b. Element of reasonable assurance c. Philippine Standards on Auditing
c. Elements of materiality and risk d. Philippine Accounting Standards
d. Element of internal control
32. The Philippine Standards on Auditing issued by AASC
27. Which of the following is the authoritative body designated to a. Need to be applied on all audit related.
promulgate auditing standards? b. Require that in no circumstances would an auditor may
a. Financial Reporting Standards Council judge it necessary to depart from a PSA, even though
b. PICPA such a departure may result to more effective
c. Association of CPAs in Public Practice and PICPA achievement of the objective of an audit.
d. Auditing and Assurance Standards Council c. Apply to independent examination of financial
statements of any entity when such an examination is
28. Which of the following mostly describes the function of AASC? conducted for the purpose of expressing an opinion.
a. To monitor full compliance by auditors to PSAs. d. Must not apply to other related activities of auditors.
b. To assist the Board of Accountancy in conducting
administrative proceedings on erring CPAs in audit 33. An auditor needs not abide by a Philippines Standard on
practice. Auditing if the auditor believes that
c. To promulgate auditing standards, practices and a. The amount is insignificant.
procedures that shall be generally accepted by the b. The requirement of the PSA is impractical to perform.
accounting profession in the Philippines. c. The requirement of the PSA is impossible to perform.
d. To undertake continuing research on both auditing and d. Any of the above three is correct.
financial accounting in order to make them responsive to
the needs of the public. 34. Auditing standards differ from auditing procedures in that
procedures relate to:
29. The Philippine Standards on Auditing issued by the Auditing a. Measures of performance
and Assurance Standards Council (AASC) b. Acts to be performed.
a. Are interpretations of generally accepted auditing c. Audit judgments.
standards d. Audit principles.
b. Are the equivalent of laws for audit practitioners.
c. Must be followed in all situations. 35. Every independent audit engagement involves both auditing
d. Are optional guidelines which an auditor may choose to standards and auditing procedures. The relationship between
follow or not follow when conducting an audit. the two may be illustrated by how they apply from

AT – Prelim Rev (875 MCQs) Red Sirug Page 61 of 85


engagement to engagement. The best representation of this d. Government regulatory agencies
application is that, from one audit engagement to the next,
a. Auditing standards are applied uniformly but auditing 40. Which of the following is a distinguishing mark of the
procedures may vary. accountancy profession?
b. Auditing standards may vary but auditing procedures are a. A drive to excellence
applied uniformly. b. Acceptance of the responsibility to act in the public
c. Auditing standards are applied uniformly but auditing interest
procedures are optional. c. Professional objectivity
d. Both auditing standards and auditing procedures are d. Professional skepticism
applied uniformly.
41. A professional accountant is likened to a prudent father to his
36. Generally accepted accounting principles (GAAP) are son. This relates to what fundamental principle?
distinguished from generally accepted auditing standards a. Professional competence and due care
(GAAS) in that: b. Confidentiality
a. GAAP are the principles auditors follow when conducting c. Integrity
an audit, while GAAS are the standards for presentation d. Objectivity
of financial statements and underlying transactions.
b. GAAP are the principles for presentation of financial 42. Which fundamental principle is seriously threatened by an
statements and underlying transactions, while GAAS engagement that is compensated based on the net proceeds
are the standards that the auditors should follow on loans received by the client from a commercial bank?
when conducting an audit. a. Integrity
c. When GAAP are violated, sufficiently strong GAAS may b. Objectivity
make up for most GAAP deficiencies. c. Professional behavior
d. GAAP are promulgated by the SEC, while GAAS are d. Confidentiality
promulgated by the PFRC.
43. Which of the following values is not necessary for a
37. How frequent can a professional accountants have press and professional accountant?
other media releases commemorating their anniversaries in a. Honesty
public practice by informing the public of their achievements or b. Objectivity
accomplishments in contributing toward nation building or c. Integrity
enhancing the image or standards of the accounting d. A primary commitment to self-interest
profession?
a. 2 years 44. Professional accountants may encounter problems in
b. 3 years identifying unethical behavior or in resolving an ethical conflict.
c. 5 years When faced with significant ethical issues, professional
d. 6 years accountants should do the following, except
a. Follow the established policies of the employing
38. Which of the following is not allowed to be included in a organization to seek a resolution of such conflict
website of a firm of professional accountants? b. Review the conflict problem with the immediate superior if
a. Names of partners/principals with their educational the organization's policies do not resolve the ethical
attainment. conflict.
b. Membership in any professional body. c. If the problem is not resolved with the immediate
c. Awards received superior and the professional accountant determines
d. Listings of the firm's clients. to go to the next higher managerial level, the
immediate superior need not be notified of the
39. In their fiduciary role, the professional accountants owe their decision.
primary loyalty to: d. Seek counseling and advice on a confidential basis with
a. The accounting profession an independent advisor or the applicable professional
b. The general public accountancy body or regulatory body to obtain an
c. The client understanding of possible courses of action.

AT – Prelim Rev (875 MCQs) Red Sirug Page 62 of 85


leading to the admission of candidates to the CPA
45. As a resolution of the conflict in the application of fundamental licensure examination.
principles, the auditor, after considering the ethical issues and
relevant facts may do any of the following, except: 49. All of the following are represented to the Financial Reporting
a. Must immediately resign from the engagement or the Standards Council, except:
employing entity. a. Securities and Exchange Commission
b. Should weigh me consequences of each possible course b. Bureau of Internal Revenue
of action. c. Commission on Higher Education
c. Should consult with other appropriate persons within the d. Board of Accountancy
firm or employing organization for help to finally resolve
the matter.
d. The professional accountant may wish to obtain 50. A CPA whose certificate of registration has been revoked:
professional advice from, the relevant professional body a. Can no longer be reinstated.
without breaching confidentiality if significant conflict b. Is automatically reinstated as a CPA by the PRC after two
cannot be resolved. years if he has acted in an exemplary manner.
c. May be reinstated by the Professional Regulation
46. Which of the following is incorrect regarding integrity and Commission after two years if he has acted in an
objectivity? exemplary manner.
a. Integrity implies not merely honesty but fair dealing and d. May be reinstated as a CPA by the Board of
truthfulness. Accountancy after two years if he has acted in an
b. The principle of objectivity imposes the obligation on all exemplary manner.
professional accountants to be fair, intellectually honest
and free of conflicts of interest. 51. The Philippine Accountancy Act of 2004 provides that all
c. Professional accountants serve in many different Working papers made during an audit shall be the property of
capacities and should demonstrate their objectivity in the auditor. These working papers shall include the following,
varying circumstances. except:
d. Professional accountants should neither accept nor a. Working papers prepared by the CPA and his staff.
offer any gifts or entertainment. b. Analysis and schedule prepared and submitted to the
auditor by his client's staff.
47. If a professional accountant is billing an audit client a number c. Excerpts or copies of documents furnished to the auditor.
of hours greater than those actually worked, which of the d. Any report submitted by the auditor to his client.
following fundamental principles is likely violated?
a. Objectivity 52. Who are required to apply for accreditation with the
b. Integrity Professional Regulation Commission if the applicant is a
c. Professional due care partnership of Professional Accountants?
d. Confidentiality a. Managing partner only
b. All partners only
48. Which of the following is not a function of the Board of c. Partners and staff members
Accountancy as specified in the Philippine Accountancy Act of d. Partners, principals, and staff members
2004?
a. To investigate violations of the Accountancy Law and the 53. Individual CPAs, Firms or Partnerships of CPAs, including
rules and regulations promulgated therewith. partners and staff members thereof shall register with the BOA
b. To look from time to time into the conditions affecting the and the PRC. If the accreditation of Velasco and Co., CPAs,
practice of the accountancy profession. was renewed on September 30, 2008, the next renewal must
c. To create and direct accrediting agencies that are be on or before:
entrusted the functions of reviewing higher a. September 30, 2010
educational institutions' policies and practices b. September 30, 2011
leading to accreditation/reaccreditation of BSA c. December 31, 2010
program. d. December 31, 2011
d. To determine and prescribe minimum requirements

AT – Prelim Rev (875 MCQs) Red Sirug Page 63 of 85


54. The APO shall renew its Certificate of Accreditation once b. Provide the basis for excluding admission of the working
every how many years after the date of the Resolution papers as evidence because of the privileged
granting the petition for re-accreditation and the issuance of communication rule.
the said certificate upon submission of the requirements? c. Provide the CPA with evidence and documentation
a. 2 years which may be helpful in the event of a lawsuit.
b. 3 years d. Establish a continuity of relationship with the client
c. 4 years whereby indiscriminate replacement of CPAs is
d. 6 years discouraged.

55. Engagement letters are widely used in practice for 59. The responsibility for adopting sound accounting policies,
professional engagements of all types. The primary purpose maintaining adequate internal control, and making fair
of the engagement letter is to representations in the financial statements rests
a. Remind management of its primary responsibility over the a. With the management.
financial statements. b. With the independent auditor.
b. Satisfy the requirements of the Code of Professional c. Equally with management and the auditor.
Conduct for CPAs. d. With the internal audit department.
c. Provide a starting point for the auditor's preparation of the
preliminary audit program. 60. The ordinary examination of financial statements is not
d. Provide a written record of the agreement with the primarily designed to disclose defalcations and other
client as to the services to be provided. irregularities although their discovery may result. Normal audit
procedures are more likely to detect a fraud arising from
56. Which of the following is not likely a quality control procedure a. Collusion on the part of several employees.
on consultation? b. Failure to record cash receipts for services rendered.
a. Identifies areas and specialized situations where c. Forgeries on company checks.
consultation is required and encourages personnel to d. Theft of inventories.
consult with or in use authoritative sources on other
complex matters. 61. A principal purpose of a letter of representation from
b. Designates individuals as specialists to serve as management is to
authoritative sources and define their authority in a. Serve as an introduction to company personnel and an
consultative situations. authorization to examine the records.
c. Assigns an appropriate person or persons to be b. Discharge the auditor from legal liability for his
responsible for assigning personnel to audits. examination.
d. Specifies the extent of documentation to be provided for c. Confirm in writing management's approval of limitations
the result of consultation in those areas and specialized on the scope of the audit.
situations where consultation is required. d. Remind management of its primary responsibility for
financial statements.
57. According to Philippine Standards on Auditing, because there
are inherent limitations in an audit that affect the auditor's 62. The auditor should not assume that management is dishonest,
ability to detect material misstatements, the auditor is: but the possibility of dishonesty must be considered." This is
a. Neither a guarantor nor an insurer of financial an example of
statements. a. Expectation gap.
b. A guarantor but not an insurer of the statements. b. An attitude of professional skepticism.
c. An insurer but not a guarantor of the statements. c. Due diligence.
d. Both a guarantor and an insurer of the financial d. An ethical requirement.
statements.
63. Which of the following statements is true?
58. The working papers prepared by a CPA in connection with an a. It is usually easier for the auditor to uncover irregularities
audit engagement are owned by the CPA, subject to certain than errors.
limitations. The rationale for this rule is to b. It is usually easier for the auditor to uncover errors
a. Protect the working papers from being subpoenaed. than irregularities.

AT – Prelim Rev (875 MCQs) Red Sirug Page 64 of 85


c. It is usually equally difficult for the auditor to uncover d. Personnel management
errors or irregularities.
d. Usually, none of the given statements is true. 69. The work of each assistant needs to be reviewed by personnel
of at least equal competence. Which of the following is not one
64. Generally, the decision to notify parties outside the client's of the objectives of this requirement?
organization regarding a noncompliance with laws and a. The conclusions expressed are consistent with the result
regulations of the work performed and support the opinion.
a. Independent auditor. b. The work performed and the results obtained have been
b. Management. adequately documented.
c. Outside legal counsel. c. The audit objectives have been achieved.
d. Internal auditors. d. All available evidences have been obtained,
evaluated and documented.
65. An audit made in accordance with Philippine Standards on
Auditing generally should 70. Which of the following statements is true when the CPA has
a. Be expected to provide assurance that noncompliance been engaged to do an attestation engagement?
with laws and regulations will be detected if the internal a. The CPA firm is engaged and paid by the client; therefore,
control is effective. the firm has primary responsibility to be an advocate for
b. Be relied upon to disclose indirect-effect noncompliance the client.
with laws and regulations. b. The CPA firm is engaged and paid by the client, but
c. Encompass a plan to search actively for noncompliance the primary beneficiaries of the audit are the
with laws and regulations which relate to operating statement users.
aspects. c. Should a situation arise where there is no convincing
d. Not be relied upon to provide assurance that all authoritative standard available, and there is a choice of
noncompliance with laws and regulations will be actions which could impact client's financial statements
detected. either positively or negatively, the CPA is free to endorse
the choice which is best in the client's interest.
66. An auditor who believes that a material irregularity may exist d. As long as CPA firms are competent, it is not required
should initially that they remain unbiased.
a. Discuss the matter with those believed to be involved in
the perpetration of material irregularity. 71. When CPAs are able to maintain an independent attitude in
b. Discuss the matter with a higher level of management. fulfilling their responsibility, it is referred to as independence in
c. Withdraw from the engagement. a. Fact.
d. Consult legal counsel. b. Appearance.
c. Conduct.
67. When management refuses to disclose in the financial d. Total.
statements noncompliance to laws and regulations which are
identified by the independent auditor, the CPA may be 72. When the users of financial statements have confidence in the
charged with unethical conduct for independence of the CPA, it is referred to as in independence
a. Withdrawing from the engagement. in
b. Issuing a disclaimer of opinion. a. Fact.
c. Failure to uncover the noncompliance to laws and b. Appearance.
regulations during the prior audits. c. Conduct.
d. Reporting these activities to the audit committee. d. Total.

68. A procedure in which a quality control partner periodically 73. When determining whether independence is impaired because
tests the application of quality control procedures is most of an ownership interest in client company, materiality will
directly related to which quality control element? affect whether ownership is a violation of rule of independence
a. Engagement performance a. In all circumstances.
b. Independence, integrity, and objectivity b. Only for direct ownership.
c. Monitoring c. Only for indirect ownership.

AT – Prelim Rev (875 MCQs) Red Sirug Page 65 of 85


d. Under no circumstances.
79. The responsibility for the fairness of the financial assertions
74. A professional accountant has a professional duty or right that are embodied in the financial statements and in the notes
disclose confidential information in each of the following, to the financial statements rests:
except: a. With the audit committee.
a. To comply with technical standards and ethics b. With management.
requirements. c. With Securities and Exchange Commission.
b. To disclose to the Bureau of Internal Revenue any d. Equally with management and the stockholders.
fraudulent scheme committed by the client on
payment of income tax. 80. When preparing the financial statements, it is acceptable for
c. To comply with the quality review of a member body or the auditor to prepare
professional body a. The footnotes for client.
d. To respond to an inquiry or investigation by a member b. The statement for client.
body or regulatory body. c. A draft of the statements and footnotes for client.
d. A draft of the statements for client.
75. Which of the following best describes the passing of
confidential information from a client to its auditor? The 81. Which of the following statements best describes the auditor's
information: responsibility regarding the detection of material errors and
a. Should in no circumstances be conveyed to third parties. frauds?
b. Is not legally protected and can be subpoenaed by a a. The auditor is responsible for the failure to detect material
competent court. errors and frauds only when such failure results from the
c. Can only be released for peer reviews after receiving misapplication of generally accepted accounting
permission from the client. principles.
d. Should be conveyed to the public if it affects the b. The auditor is responsible for the failure to detect material
"correctness" of the financial statements. errors and frauds only when the auditor fails to confirm
receivables or observe inventories.
76. A member in public practice may perform for a contingent fee c. The audit should be designed to provide reasonable
any professional services for a client for whom the member or assurance that material errors and frauds are
member's firm performs detected.
a. An audit. d. Extended auditing procedures are required to detect
b. A review. unrecorded transactions even if there is no evidence that
c. A compilation used only by management. material errors and frauds may exist.
d. An audit of prospective financial information.
82. The auditor has considerable responsibility for notifying users
77. Which one of the following contingent fee is allowed? as to whether or not the statements are properly stated. This
a. All services performed by a CPA film. imposes upon the auditor a duty to
b. Non-attestation services. a. Provide reasonable assurance that material
c. Non-attestation services, unless the CPA firm was misstatements will be detected.
also performing attestation services for the same b. Be an insurer of the fairness in the statements.
client. c. Be a guarantor of the fairness in the statements.
d. Attestation services. d. Be equally responsible with management for the
preparation of the financial statements.
78. When the auditor issues an erroneous opinion as a
consequence of an underlying failure to comply with the 83. The factor that distinguishes an error from an irregularity is
requirements of generally accepted auditing standards, it a. Whether it is peso amount or a process.
results to b. Whether it is a caused by the auditor or the client.
a. Business failure. c. Materiality.
b. Audit failure. d. Intent.
c. Audit risk.
d. All of them 84. The reason why an auditor accumulates evidence is to

AT – Prelim Rev (875 MCQs) Red Sirug Page 66 of 85


a. Defend himself in the event of a lawsuit.
b. Justify the conclusions he has otherwise reached. 90. An audit plan is a
c. Satisfy the requirements of the bureau of internal revenue. a. Detailed plan of analytical procedures and all substantive
d. Enable him to reach conclusions about the fairness tests to be performed in the course of the audit.
of the financial statements and issue an appropriate b. Document that provides an overview of the company
audit report. and a general plan for the audit work to be
accomplished, timing of the work, and other matters
85. The auditor gives an audit opinion on the fair presentation of of concern to the audit.
the financial statements and associates his or her name with c. Generic document that auditing firms have developed to
them when, on the basis of adequate evidence, the auditor lead the process of the audit through a systematic and
concludes that the financial statements are unlikely to mislead logical process.
a. A prudent user. d. Budget of the time that should be necessary to complete
b. Management. each phase of the audit procedures.
c. The reader.
d. Investors. 91. There is generally an agreement within the auditing profession
and the courts that the auditor
86. The probability that an auditor's procedures leading to the a. Is both a guarantor and an insurer of the financial
conclusion that a material error does not exist in an account statements.
balance when, in fact, such error does exist is referred to as b. Is a guarantor but not an insurer of the statements.
a. Prevention risk. c. Is an insurer but not a guarantor of the statements.
b. Detection risk. d. Is neither a guarantor nor an insurer of financial
c. Inherent risk. statements.
d. Control risk.
92. The auditor is not liable to his client for
87. Which of the following is not included in an audit engagement a. Negligence.
letter? b. Bad faith.
a. Objectives of the engagement c. Errors of judgment
b. Representations that the financial statements were d. Dishonesty.
prepared in accordance with PFRS
c. Management's responsibilities 93. Which of the following services provides the highest level of
d. A clear explanation of the services to be performed on assurance to third parties about a company's financial
the engagement statements?
a. Audit.
88. Which of the following is least likely included in an auditor’s b. Review.
inquiry of management while obtaining information to identify c. Compilation.
the risks of material misstatement due to fraud? d. Each of the above provides the same level of assurance.
a. Are financial reporting operations controlled by and
limited to one location? 94. Most accounting and auditing professionals agree that when
b. Does it have knowledge of fraud or suspect fraud? an audit has failed to uncover material misstatements, and
c. Does it have programs to mitigate fraud risks? the wrong type of audit opinion is issued, the audit firm
d. Has it reported to the audit committee the nature of the a. Has failed to follow generally accepted auditing standards
company's internal control? (GAAS).
b. Should be asked to defend the quality of the audit.
89. Which of the following should the auditors normally interview c. Deserves to lose the lawsuit.
as part of their assessment of fraud risk? d. Should not be held responsible for the financial loss
a. Senior management suffered loss suffered by others.
b. Audit committee
c. Various employees whose duties financial reporting 95. The objective of quality control mandates that a public
responsibilities accounting firm should establish policies and procedures for
d. All of the given choices

AT – Prelim Rev (875 MCQs) Red Sirug Page 67 of 85


professional development which provide reasonable c. To enable successor auditor to perform a more efficient
assurance that all entry-level personnel audit.
a. Prepare working papers which are standardized in form d. To save successor auditor time and money in gathering
and content. data.
b. Will advance within the organization.
c. Develop specialties in specific areas of public accounting. 100. Which statement is incorrect regarding the Code of Ethics for
d. Have the knowledge required to enable them to fulfill Professional Accountants in the Philippines?
responsibilities assigned. a. Professional accountants refer to persons who are
Certified Public Accountants (CPA) and who hold a valid
96. In pursuing its quality control objectives with respect to certificate issued by the Board of Accountancy.
assigning personnel to engagements, a public accounting firm b. Where a national statutory requirement is in conflict
may use policies and procedures such as with a provision of the IFAC Code, the IFAC Code
a. Rotating employees from assignment to assignment on requirement prevails.
a random basis to aid in the staff training effort. c. The Code of Ethics for Professional Accountants in the
b. Allowing staff to select the assignments of their choice to Philippines is mandatory for all CPAs and is applicable to
promote better client relationships. professional services performed in the Philippines on or
c. Assigning a number of employees to each engagement after January 1, 2004.
in excess of the number required so as not to d. Professional accountants should consider the ethical
overburden the staff, and interfere with the quality of the requirements as the basic principles which they should
audit work performed. follow in performing their work.
d. Requiring timely identification of the staffing
requirements of specific engagements so that
enough qualified personnel can be made available. SET 8

97. The provision of services by a firm or network firm to an audit 1. What will an auditor who has been proposed for an audit
client that involve the design and implementation of financial engagement usually do prior to accepting a new client?
information technology systems that are used to generate a. Draft the financial statements of the client as a measure
information forming part of a client's financial statements may of goodwill.
most likely create b. With the permission from the prospective client,
a. Self-interest threat. contact the predecessor auditor to determine if there
b. Self-review threat. are any disagreements between the client and the
c. Intimidation threat. audit firms.
d. Familiarity threat, c. Obtain the potential client's permission to talk to the
former auditor and review work papers.
98. When determining whether independence is impaired because d. Perform a peer review on the potential client in
of an ownership interest in client company,' materiality will accordance with professional standards.
affect whether ownership is a violation of rule of independence
a. In all circumstances. 2. Which of the following statements is true with regard to the
b. Only for direct ownership. relationship among audit risk, audit evidence, and materiality?
c. Only for indirect ownership. a. The lower the inherent risk and control risk, the lower the
d. Under no circumstances. aggregate materiality threshold.
b. Under conditions of high inherent and control risk,
99. A successor auditor is required to communicate with the the auditor should place more emphasis on obtaining
previous auditor. The primary concern in this communication external evidence and should reduce reliance on
is internal evidence.
a. Information which will help the successor auditor in c. Where inherent risk is high and control risk is low, the
determining whether the client management has auditor may safely ignore inherent risk.
integrity. d. Aggregate materiality thresholds should not change
b. To learn about client by examining predecessor's working under conditions of changing risk levels.
papers.

AT – Prelim Rev (875 MCQs) Red Sirug Page 68 of 85


3. Which of the following is most likely to be an overall response d. Help the auditor in evaluating the appropriateness of the
to fraud risks identified in an audit? selection and application of certain accounting policies.
a. Supervise members of the audit team less closely and
rely more upon judgment. 7. The underlying reason for a code of professional conduct for
b. Use less predictable audit procedures. any profession is
c. Use only certified public accountants on the engagement. a. That it is required by congress.
d. Place increased emphasis on the audit of objective b. The need for public confidence in the quality of
transactions rather than subjective transactions. service of the profession.
c. That it provides a safeguard to keep unscrupulous people
4. Which of the following represents a procedure that the auditor out.
may use because plausible relationships among financial d. That it allows Professional Regulation Commission to
statement balances are expected to exist? have a yardstick to measure deficient performance.
a. Attributes testing
b. Enterprise risk assessment 8. Professional accountants may encounter problems in
c. Inherent tests of control identifying unethical behavior or in resolving an ethical conflict.
d. Analytical review When faced with significant ethical issues, professional
accountants should do the following, except
5. Which of the following statements is incorrect regarding a. Follow the established policies of the employing
obtaining an understanding of the entity and its environment? organization to seek a resolution of such conflict.
a. Obtaining an understanding of the entity and its b. Review the conflict problem with the immediate superior if
environment is an essential aspect of performing an audit the organization's policies do not resolve the ethical
in accordance with PSAs. conflict.
b. Understanding of the entity and its environment c. If the problem is not resolved with the immediate
establishes frame of reference within which the auditor superior and the professional accountant determines
plans the audit and exercises professional judgment to go to the next higher managerial level, the
about assessing risks of material misstatement in the immediate superior need not be notified of the
financial statements and responding to those risks decision.
throughout the audit. d. Seek counseling and advice on a confidential basis with
c. The auditor's primary consideration is whether the an independent advisor or the applicable professional
understanding that has been obtained is sufficient to accountancy body or regulatory body to obtain an
assess the risks of material misstatement in the financial understanding of possible courses of action.
statements and to design and perform further audit
procedures. 9. While assessing the risk of material misstatement, the auditors
d. The depth of the overall understanding that is identity risks, relate risk to what could go wrong, consider the
required by the auditor in performing the audit is at magnitude of risks and:
least equal to that possessed by management in a. Assess the risk of misstatements due to noncompliance
managing the entity. to laws and regulations.
b. Consider the complexity of the transactions involved.
6. Inquiries directed towards those charged with governance c. Consider the likelihood that the risks could result in
may most likely material misstatements.
a. Relate to their activities concerning the design, and d. Determine materiality level.
effectiveness of the entity's internal control and whether
management has satisfactorily responded to any findings 10. Which of the following is least likely considered a financial
from these activities. statement audit risk factor?
b. Help the auditor understand the environment in a. Management operating and financing decisions are
which the financial statements are prepared. dominated by top management.
c. Relate to changes in the entity's marketing strategies, b. A new client with no prior audit history.
sales trends, or contractual arrangements with its c. Rate of change in the entity's industry is rapid.
customers. d. Profitability of the entity relative to its industry is
inconsistent.

AT – Prelim Rev (875 MCQs) Red Sirug Page 69 of 85


examination.
11. Which of the following is most likely to be considered a risk d. Confirm in writing management's approval of limitations
factor relating to fraudulent financial reporting? on the scope of the audit.
a. Low turnover of senior management.
b. Extreme degree of competition within the industry. 16. Should the auditor uncover circumstances during the audit
c. Capital structure including various operating subsidiaries. that may cause suspicions of management fraud, the auditor
d. Sales goals in excess of any of the preceding three years. must
a. Withdraw from engagement.
12. Which of the following is correct concerning requirements b. Issue an adverse opinion.
about auditor's communications about fraud? c. Issue a disclaimer.
a. Fraud that involves senior management should be d. Evaluate their implications and consider the need to
reported directly to the audit committee regardless of modify audit evidence.
the amounts involved.
b. All fraud with a material effect on the financial statements 17. In connection with the element of professional development, a
should be reported directly by the auditor to the SEC CPA firm's system of quality control should ordinarily provide
c. Fraud with a material effect on the financial statements that all personnel
should ordinarily be disclosed by the auditor through the a. Possess judgment, motivation, and adequate experience.
use of an emphasis of a matter paragraph added to the b. Seek assistance from persons having appropriate level
audit report. of knowledge, judgment, and authority.
d. The auditor has no responsibility to disclose fraud outside c. Demonstrate compliance with peer review directives.
the entity under any circumstances. d. Have the knowledge required to enable them to fulfill
responsibilities assigned.
13. Which of the following factors most likely would heighten an
auditor's concern about the risk of fraudulent financial 18. The implementation of quality control procedures that are
reporting? applicable to the individual audit engagement is the
a. Large amounts of liquid assets that are easily convertible responsibility of the
into cash. a. CPA firm.
b. Low growth and profitability as compared to other entity's b. Engagement quality control reviewer.
in the same industry. c. Expert contracted by the firm in connection with the audit
c. Financial management's participation in the initial engagement.
selection of accounting principles. d. Engagement team.
d. An overly complex organizational structure involving
unusual lines of authority. 19. It involves a study or evaluation of the quality of audit of
financial statements through a review of quality control
14. Which of the following is most likely to be an overall response measures established by on CPA firms and individual CPAs in
to fraud risks identified in an audit? public practice to ensure compliance with accounting and
a. Only use certified public accountants on the engagement. auditing standards and practices
b. Place increased emphasis on the audit of objective a. External audit
transactions rather than subjective transactions. b. Compliance audit
c. Supervise members of the audit team less closely and c. Peer review
rely more upon judgment. d. Quality review
d. Use less predictable audit procedures.
20. Which of the following risks is entirely a quality criterion based
15. A principal purpose of a letter of representation from on professional judgment?
management is to a. Inherent risk
a. Remind management of its primary responsibility for b. Control risk
financial statements. c. Detection risk
b. Serve as an introduction to company personnel and an d. Audit risk
authorization to examine the records.
c. Discharge the auditor from legal liability for his 21. Inherent risk is not a characteristic of the

AT – Prelim Rev (875 MCQs) Red Sirug Page 70 of 85


a. Major types of transactions. 27. The auditor's responsibility for failure to detect fraud arises
b. Client’s business. a. Whenever the amounts involved are material.
c. Substantive procedures. b. When such failure clearly results from
d. Effectiveness of the client’s accountants. noncompliance to generally accepted auditing
standards.
22. Misstatements must be compared to some measurement base c. Only when such failure clearly results from negligence so
before a decision can be made about the materiality of the gross as to sustain an inference of fraud on the part of
failure to follow GAAP. A commonly accepted measurement the auditor.
base would be d. Only when the examination was specifically designed to
a. Net income. detect fraud.
b. Total assets.
c. Working capital. 28. The factor that distinguishes constructive fraud from actual
d. All of the above. fraud is
a. Materiality.
23. The primary deliverable of an engagement to perform based b. Quality of internal control.
on procedures prescribed by the intended user of the report c. Type of error or irregularity.
is/are: d. Intent.
a. The Review Report
b. Report of Factual Findings 29. If a CPA recklessly abandons standards of due care and
c. Management Letter diligence while performing an audit, he or she may be held
d. The financial statements liable to unknown third parties for:
a. Gross negligence.
24. Fraudulent financial reporting is often called: b. Fraudulent misconduct.
a. Theft of assets c. Gross misconduct.
b. Employee fraud d. Contributory negligence.
c. Management fraud
d. Defalcation 30. Of the following statements, which best distinguishes ordinary
negligence from gross negligence?
25. The ordinary examination of financial statements is not a. The more material the undetected error the greater the
primarily designed to disclose defalcations and other likelihood of ordinary negligence.
irregularities although their discovery may result. Normal audit b. Gross negligence is most probable when the auditor fails
procedures are more likely to detect a fraud arising from to detect errors that occurred under conditions of strong
a. Theft of inventories. internal control.
b. Collusion on the part of several employees. c. Failure to detect material errors, whether internal control
c. Failure to record cash receipts for services rendered. is strong or weak, suggests gross negligence.
d. Forgeries on company checks. d. Failure to exercise reasonable care denotes ordinary
negligence, whereas failure to exercise minimal care
26. Working papers prepared by a CPA in connection with an indicates gross negligence.
audit engagement are owned by the CPA, subject to certain
limitations. The rationale for this rule is to 31. An auditor who believes that a material irregularity may exist
a. Protect the working papers from, being subpoenaed. should initially
b. Provide the CPA with evidence and documentation a. Consult legal counsel.
which may be helpful in the event of a lawsuit. b. Discuss the matter with those believed to be involved in
c. Provide the basis for excluding admission of the working the perpetration of the material irregularity.
papers as evidence because of the privileged c. Discuss the matter with a higher level of management.
communication rule. d. Withdraw from the engagement.
d. Establish a continuity of relationship with the client
whereby indiscriminate replacement of CPAs is 32. Which of the following statements is correct concerning the
discouraged. auditor's responsibility with respect to illegal acts?
a. An auditor must design tests to detect both direct-effect

AT – Prelim Rev (875 MCQs) Red Sirug Page 71 of 85


and indirect-effect illegal acts.
b. An auditor must design tests to detect both immaterial 37. For assurance engagements which are neither audits nor
and material direct-effect illegal acts. reviews of historical financial information, the following
c. An auditor must design tests to obtain reasonable standard applies:
assurance of detecting material direct-effect illegal a. PSAs
acts. b. PSAEs
d. An auditor must design tests to detect both material c. PSREs
direct-effect and material indirect-effect illegal acts. d. PSRSs

33. If specific information comes to an auditor's attention that 38. Unlike consulting services, in assurance services:
implies the existence of possible illegal acts that could have a a. There is confirmation that financial statement assertions
material, but indirect effect on-the financial statements, the are accurate.
auditor should next b. Misstated account balances are generally corrected by an
a. Report the matter to an appropriate level of management independent audit.
at least one level above those involved. c. Ineffective internal controls may exist.
b. Apply audit procedures specifically directed to d. Competing interests may exist between management
ascertaining whether an illegal act has occurred. and the users of statements.
c. Seek the advice of an informed expert qualified to
practice law as to possible contingent liabilities. 39. The attest function:
d. Discuss the evidence with the client's audit committee, or a. Is an essential part of every engagement performed by a
others with equivalent authority. CPA.
b. Includes the preparation of a written report of the
34. When planning the audit, if the auditor has no reason to CPA's conclusion.
believe that illegal acts exist, the auditor should c. Requires a complete review of all transactions during the
a. Include audit procedures which have a strong probability period under examination.
of detecting illegal acts. d. Requires a review of a sample of transactions during the
b. Make inquiries of management regarding their period under examination.
policies and regarding their knowledge of violations,
and then rely on normal audit procedures to detect 40. Which of the following is an assurance service?
errors, irregularities, and illegalities. a. Performance measurement
c. Still include some audit procedures designed specifically b. Systems design and installation
to uncover illegalities. c. Tax planning
d. Ignore the topic. d. Personal financial planning

35. When the auditor knows that an illegal act has occurred, the 41. Non-assurance engagements include all of the following
auditor must services, except:
a. Issue an adverse opinion. a. Agreed-upon procedures engagements.
b. Withdraw from the engagement. b. Compilations of financial or other information.
c. Report it to the proper government authorities. c. Management and tax consulting.
d. Consider the effects on the financial statements, d. Examination of prospective financial information.
including the adequacy of disclosure.
42. Unlike consulting services, assurance services:
36. Assurance services differ from consulting services in that they a. Make recommendation to management.
I. Focus on providing advice. b. Report on how to use information.
c. Are two-party contracts.
II. Involve monitoring of one party by another.
d. Report on the quality of information.
a. I only.
b. II only. 43. Identify the correct statement.
c. Both I and II. a. Accounting services do not provide attestation.
d. Neither I nor II. b. Management consulting services provide attestation in all

AT – Prelim Rev (875 MCQs) Red Sirug Page 72 of 85


cases. 49. A practitioner should accept an assurance engagement only if
c. An audit provides limited assurance by attesting to the a. The practitioner's conclusion is to be contained in a
fairness of the client's assertions. written report.
d. A review provides positive assurance by attesting the b. The subject matter is the responsibility of either the
reliability of the client's assertions. intended users or the practitioner.
c. The subject matter is in the form of financial information.
44. An attestation engagement is one in which a CPA is engaged d. The criteria to be used are not available to the intended
to: users.
a. Assemble prospective financial statements based on the
assumptions of the entity's management without 50. Assurance engagement risk is the risk
expressing any Testify as an expert witness in accounting, a. Of expressing an inappropriate conclusion when the
auditing or tax matters, given certain stipulated facts. subject matter information is not materially misstated.
b. assurance. b. That the practitioner expresses an inappropriate
c. Issue a written communication expressing a conclusion when the subject matter information is
conclusion about the reliability of a written assertion materially misstated.
that is the responsibility of another party. c. Of expressing an inappropriate conclusion when the
d. Provide tax advice or prepare a tax return based on subject matter information is either materially misstated or
financial information the CPA has not audited or reviewed. not materially misstated.
d. Through loss from litigation, adverse publicity, or other
45. Assurance engagements encompass the following types of events arising in connection with a subject matter
services, except: reported on.
a. Audit of historical financial statements.
b. Review engagements. 51. The following are components of assurance engagement risk,
c. Management consulting. except
d. Attestation services. a. Detection risk
b. Business risk
46. Criteria that are embodied in laws or regulations, or issued by c. Inherent risk
authorized or recognized bodies of experts that follow a d. Control risk
transparent due process are called:
a. Suitable criteria 52. The predominant type of attestation service performed by
b. General criteria CPAs is:
c. Established criteria a. Audit.
d. Specifically developed criteria b. Review.
c. Compilation.
47. Which of the following is not a characteristic of suitable criteria? d. Management consulting.
a. Completeness
b. Neutrality 53. Which of the following is incorrect regarding a compilation
c. Relevance engagement?
d. Comparability a. The CPA uses his auditing expertise to collect,
classify and summarize financial information.
48. The criteria to be used for a particular engagement can either b. The CPA should exercise due care.
be established or specifically developed. Established criteria c. The engagement ordinarily entails reducing detailed data
are those to a manageable and understandable form.
a. Designed for the purpose of the engagement. d. The procedures performed do not enable the accountant
b. Embodied in laws or regulations or issued by autho- to express any form of assurance.
rized or recognized bodies of experts that follow a
transparent due process. 54. The type of assurance that is provided by the CPA on a
c. Made available to the intended users. compilation report is:
d. Not made available to the intended users. a. Limited assurance.
b. No assurance.

AT – Prelim Rev (875 MCQs) Red Sirug Page 73 of 85


c. Low assurance. form of an assertion by the responsible party that is made
d. Negative assurance. available to intended users. These engagements are called
a. Recurring engagements
55. A report on factual findings is the end product of the auditor b. Non-assurance engagements
when performing: c. Direct reporting engagements
a. Audit. d. Assertion-based engagements
b. Review.
c. Agreed-upon procedures. 62. What type of assurance engagement is involved when the
d. Compilation. practitioner expresses a positive form of conclusion?
a. Positive assurance engagement
56. What assurance is provided by the auditor in an agreed-upon b. Limited assurance engagement
procedures engagement? c. Reasonable assurance engagement
a. No assurance. d. Absolute assurance engagement
b. Low.
c. Moderate. 63. What type of assurance engagement is involved when the
d. Reasonable. practitioner expresses a negative form of conclusion?
a. Negative assurance engagement
57. The term "accountant" has been used by Auditing and b. Assertion-based assurance engagement
Assurance Standards Council to refer to a CPA in public c. Limited assurance engagement
practice who is engaged to: d. Reasonable assurance engagement
a. Audit financial statements.
b. Review financial statements. 64. Assurance engagements involve:
c. Apply agreed-upon procedures. a. Two separate parties: a professional accountant and
d. Compile financial statements. intended users.
b. Two separate parties: a professional accountant and a
58. Which of the following best describes relationships among responsible party.
auditing, attest and assurance services? c. Three separate parties: a practitioner, a
a. Attest is a type of auditing service. responsible party and intended users.
b. Assurance is a type of attest service. d. Three separate parties: a professional accountant, a
c. Auditing is a type of assurance service. responsible party, and intended users.
d. Auditing and attest services represent two distinctly
different types of services. 65. The auditor's evaluation of the likelihood of material employee
fraud is normally done initially as a part of
59. Which of the following is the single feature that most clearly a. The assessment of whether to accept the audit
distinguishes auditing, attestation, and assurance: engagement.
a. CPA's approach to the service. b. Understanding the entity's internal control structure.
b. Type of service. c. The Tests of Controls.
c. Scope of services. d. The Tests of Transactions.
d. Training necessary to perform the service.
66. In which circumstance is a CPA firm's independence most
60. For assurance engagements regarding historical financial likely to be impaired?
information, reasonable assurance engagements are called: a. An individual on the audit has a close relative who is a
a. Review engagements. receptionist for the client.
b. Audit engagements. b. The father of the audit senior holds a material financial
c. Compilation engagements. interest in the client of which the senior is unaware.
d. Agreed-upon procedures engagements. c. The spouse of a staff member on the audit has an
immaterial common stock investment in the audit
61. In some assurance engagements, the evaluation or client.
measurement of the subject matter is performed by the d. The partner in charge of the office's compensation is
responsible party, and the subject matter information is in the affected by office profitability, a portion of which arises

AT – Prelim Rev (875 MCQs) Red Sirug Page 74 of 85


from this audit. company.
c. Undetected errors have resulted in materially misleading
67. Which of the following partners is least likely to be considered financial statements.
a "covered member" for purposes of rendering assurance d. The auditor fails to detect fraud resulting from
service to of Company A, a nonaudit client, performed by the management override of the control structure.
head office of a national CPA firm?
a. The partner in charge of the Davao office. 73. An expectation of the public is that the auditor will recognize
b. The partner in charge of the entire CPA firm. that the primary users of audit services are:
c. A partner in the Cebu office of the CPA firm who a. The employees
maintains a small, immaterial investment in Company b. The investors and creditors
A. c. The SEC
d. A partner in the Davao office who worked on the d. The board of directors
Company A for a different assurance engagement in
previous years, but currently has no responsibilities with 74. The overriding objective of the International Auditing
respect to the engagement Standards that are issued by the International Auditing
Practices Committee of the IFAC is
68. While performing services for their clients, professionals have a. To override a country’s regulations governing the audit
always had a duty to provide a level of care which is of financial statements.
a. Reasonable. b. To improve the uniformity of auditing practices and
b. Greater than average. related services throughout the world.
c. Superior. c. To provide a uniform application of specific audit
d. Guaranteed to be free from error. procedures that are acceptable worldwide.
d. To replace generally accepted auditing standards.
69. The principal issue to be resolved in cases involving alleged
negligence is usually 75. Management’s assertions in the financial statements are
a. The amount of the damages suffered by the users of the relevant to the audit process because:
financial statements. a. They provide evidence that auditors have prepared
b. Whether to impose punitive damages on defendant. financial statements in accordance with GAAP
c. The level of care required to be exercised. b. They embody the audit procedures that will be
d. Whether defendant was involved in fraud. performed by the audit engagement team.
c. They include representations of financial statements
70. Privity of contract exists between the in accordance with the applicable reporting criteria
a. Auditor and the Securities and Exchange Commission. d. They relate to regulator’s expectations about audit
b. Auditor and client. results.
c. Auditor and third parties.
d. All of the above 76. A CPA firm is considered independent when it performs which
of the following services for a publicly traded audit client?
71. A CPA is criminally liable if he a. Serving as a member of the client’s board of directors.
a. Refuses to turn over the schedules or working papers b. Accounting information system design and
prepared by the client staff to the client. implementation.
b. Performs an audit in a negligent manner. c. Tax return preparation as approved by the board of
c. Intentionally allows an omission of a material fact directors.
required to be stated in a financial statement. d. Determining which accounting policies will be adopted
d. Was not able to submit the audited financial statements by the client.
on time.
77. Jessie Cruz, CPA, forgot to test a client’s assessment of
72. The auditor's defense of contributory negligence is most likely goodwill impairment during an audit. Such an act is probably
to prevail when an example of:
a. Third party injury has been minimal. a. Due diligence
b. The client is privately held as contrasted with a public b. Reckless professional behavior

AT – Prelim Rev (875 MCQs) Red Sirug Page 75 of 85


c. Ordinary negligence member accepted a fee for services and the fee was
d. Fraud a. Fixed by a public authority.
b. Based on a price quotation submitted in competitive
78. Similar to auditors in the CPA realm, internal auditors also bidding.
strive to possess: c. Based on the result of judicial proceedings.
a. Independence d. Payable after a specified finding was attained.
b. Competence
c. Objectivity 84. For which of the following services is a CPA professional
d. All of the above required to be independent?
a. Audits of historical financial statements
79. A review engagement differs in scope as compared to an audit b. Review services
due to c. Examination of prospective financial statements
a. The subject matter of the audit. d. All of the above
b. The quantity and type of evidence obtained.
c. Ethical requirements with respect to independence. 85. For which of the following services is a CPA professional not
d. The users of the financial statements. required to be independent?
a. Tax returns preparation
80. Which of the following statements concerning the intended b. Audits of historical financial statements
user of a professional accountant's report is incorrect? c. Review engagement
a. The responsible party and the intended user may both be d. Examination of a forecast
within the same organization.
b. The intended user should never be established by 86. It occurs when a firm or a member of the assurance team
agreement between the practitioner and the could benefit from a financial interest in, or other self-interest
responsible party or those engaging or employing conflict with, an assurance client.
the practitioner. a. Self-interest threat
c. In some circumstances, the intended user may be b. Self-review threat
established by law. c. Advocacy threat
d. The responsible party may also be one of the intended d. Familiarity threat
users.
87. The CPA title was conceived and created, under the first
81. The network firms are required to be independent of the client accountancy law, for professional accountants engaged in
a. For assurance engagements provided to an audit which of the following?
client. a. Accounting education
b. For assurance engagements provided to clients that are b. Public accounting
not audit clients, when the report is not expressly c. Management accounting
restricted for use by identified users. d. Government accounting
c. For assurance engagements provided to clients that are
not audit clients, when the assurance report is expressly 88. Regarding practice of accountancy, which of the following
restricted for use by identified users. certificates is issued to a CPA first, a certificate of registration
d. All of the above or a certificate of accreditation?
a. Certificate of registration
82. One difference between auditors and other professionals is b. Certificate of accreditation
that most professionals c. Both are issued at the same time
a. Need not be concerned about remaining independent. d. Neither are issued to CPAs.
b. Do not have requirements for continuing education
beyond college. 89. Examples of circumstances that may create self-interest threat
c. Do not have to pass a rigorous examination. include:
d. Are not expected to act in public interest. a. Contingent fees relating to assurance engagements.
b. A direct financial interest or material indirect financial
83. The Code of Professional Conduct would be violated if a interest in an assurance client.

AT – Prelim Rev (875 MCQs) Red Sirug Page 76 of 85


c. A loan or guarantee to or from an assurance client or any b. Eman only
of its directors or officers. c. Neither Eman and Noel
d. All of the above d. Noel only

90. Which of the following least likely create "self-interest threat"? 95. If a member of the assurance team, or their immediate family
a. Undue dependence on total fees from an assurance member receives, by way of, for example, an inheritance, gift
client. or, as a result of a merger, a direct financial interest or a
b. Concern about the possibility of losing the engagement. material indirect financial interest in the assurance client, a
c. Having a close business relationship with an assurance self-interest threat would be created. The following safeguards
client. should be applied to eliminate the threat or reduce it to an
d. Pressure to reduce inappropriately the extent of work acceptable level:
performed in order to reduce fees a. Disposing of the financial interest at the earliest practical
date.
91. The integrated national professional organization of Certified b. Removing the member of the assurance team from the
Public Accountants accredited by the BOA and the PRC per assurance engagement.
PRC accreditation No. 15 dated October 2, 1975. c. Either a or b
a. Philippine Institute of Certified Public Accountants d. Neither a nor b
(PICPA)
b. Auditing and Assurance Standards Council (AASC) 96. Occurs when, by virtue of a close relationship with an
c. Financial Reporting Standards Council (FRSC) assurance client, its directors, officers or employees, a firm or
d. Education Technical Council (ETC) a member of the assurance team becomes too sympathetic
to the client's interests.
92. A CPA-lawyer, acting as a legal counsel to one of his audit a. Self-interest threat
client, is an example of b. Self-review threat
a. Self-interest threat c. Advocacy threat
b. Self-review threat d. Familiarity threat
c. Advocacy threat
d. Familiarity threat 97. A former officer, director or employee of the assurance client
serves as a member of the assurance team. This situation will
93. Intimidation threat least likely create
a. Is not a threat to independence. a. Self-interest threat.
b. Occurs when a member of the assurance team may b. Self-review threat.
be deterred from acting objectively and exercising c. Intimidation threat.
professional skepticism by threats, actual or d. Familiarity threat.
perceived, from the directors, officers or employees
of an assurance client. 98. Immediate family includes
c. Occurs when, by virtue of a close relationship with an a. Parent.
assurance client, its directors, officers or employees, a b. Sibling.
firm or a member of the assurance team becomes too c. Non-dependent child.
sympathetic to the client's interests. d. Spouse.
d. Occurs when a firm, or a member of the assurance team,
promotes, or may be perceived to promote, an assurance 99. Which of the following is not likely a threat to independence?
client's position or opinion to the point that objectivity may, a. Acting as an advocate on behalf of an assurance client in
or may be perceived to be, compromised. litigation or in resolving disputes with third parties.
b. Long association of a senior member of the assurance
94. Eman, a CPA, has a law practice. Eman has recommended team with the assurance client.
one of his clients to Noel, a CPA. Noel has agreed to pay c. Threat of replacement over a disagreement with the
Eman 10% of the fee for services rendered by Noel to Eman's application of an accounting principle.
client. Who, if anyone, is in violation of the Code of Ethics? d. Owning immaterial indirect financial interest in an
a. Both Eman and Noel audit client.

AT – Prelim Rev (875 MCQs) Red Sirug Page 77 of 85


b. Many audit conclusions are made on the basis of
SET 9 examining a sample of evidence.
c. The work, under taken by the auditor is permeated by
1. This is the traditional service provided by CPA firms. judgment.
a. Tax services d. Fatigue and human weaknesses can cause
b. Compilation auditors to overlook pertinent evidence.
c. Reviews
d. Audits 7. Which of the following is not one of the general principles
governing the audit of financial statements?
2. The billing arrangement based on actual time spent by audit a. The auditor should conduct the audit in accordance with
engagement team is PSA.
a. Flat sum basis b. The auditor should comply with the Philippine Code of
b. Per diem basis Professional Ethics.
c. Maximum fee basis c. The auditor should plan and perform the audit with an
d. Retainer basis attitude of professional skepticism.
d. The auditor should obtain sufficient appropriate
3. Professionals are expected to conduct themselves at a higher evidence primarily through inquiry and analytical
level than procedures to be able to draw reasonable conclusions.
a. Anyone else.
b. Their clients. 8. Which of the following statements does not properly describe
c. Most other members of society. an element of the theoretical framework of auditing?
d. Most other members of their profession. a. An audit benefits the public.
b. The data to be audited can be verified.
4. A candidate who fails in two (2) complete CPA examinations c. Auditors act on behalf of management.
shall be: d. Short-term conflicts may exist between managers who
a. Reverted to undergraduate status. prepare data and auditors who examine the data.
b. Forever banned from taking another set of examinations
c. Considered as having passed the examination, under the 9. Which of the following criteria is unique to the independent
rule of efforts and professional mercy. auditor's attest function?
d. Required to take a refresher course before making a. Independence.
another attempt at the CPA Board exams. b. General competence.
c. Due professional care.
5. Which of the following statements is correct concerning an d. Familiarity with the particular industry of each client.
auditor's responsibilities regarding financial statements?
a. The auditor's report should provide an assurance as to the 10. Auditing is based on the assumption that financial data are
future viability of the entity. verifiable. Data are verifiable when two or more qualified
b. Making suggestions that are adopted about the form individuals,
and content of an entity's financial statements impairs an a. Working together, can prove, beyond doubt, the
auditor's independence. accuracy of the data.
c. An auditor's responsibilities for audited financial b. Working independently, can prove, beyond
statements are confined to the expression of the reasonable doubt, the truthfulness of the data.
auditor's opinion. c. Working independently, each reach essentially
d. The fair presentation of audited financial statements in similar conclusions.
accordance with an applicable financial reporting d. Working together, can agree upon the accuracy of the
framework is an implicit part of the auditor's data.
responsibilities.
11. The QRC shall have the following functions:
6. Which of the following statements does not properly a. Conduct quality control review on applicants for
describe a limitation of an audit? registration to practice public accountancy and
a. Many financial statement assertions cannot be audited. render a report on such quality review.

AT – Prelim Rev (875 MCQs) Red Sirug Page 78 of 85


b. Mete out sentences of imprisonment to CPAs who fail to c. The auditor may express an unqualified opinion on
present a valid system of quality control. financial statements that are material misstated.
c. Revoke the certificate of registration and professional ID d. The client's financial statements may be materially
of an individual CPA, firm, or partnership of CPAs who false and misleading.
have not observed quality control measures and who has
not complied with the standards of quality prescribed for 17. Which of the following is a cause of information risk?
the practice of public accountancy. a. Voluminous data.
d. All of these. b. Biases and motives of the provider of information.
c. Remoteness of the information.
12. The need for independent audits of financial statements can d. Each of the above is a cause of information risk.
be attributed to all of the following conditions except:
a. Validity 18. In financial statement audits, the audit process should be
b. Remoteness conducted in accordance with
c. Consequence a. The audit program
d. Complexity of subject matter b. Philippine Accounting Standards
c. Philippine Standards on Auditing
13. The underlying conditions that create demand by users for d. Philippine Financial Reporting Standards
reliable information include the following except:
a. Decisions are not time-sensitive. 19. The overall objective of forensic auditing is to
b. Transactions that are numerous and complex. a. Attest to the efficiency with which resources are
c. Users separated from accounting records by distance employed
and time. b. Assist members of the organization in the effective
d. Financial decisions that are important to investors and discharge of their responsibilities
users. c. Unearth the truth and provide evidence in legal and
financial disputes
14. Why does a company choose to have an independent d. Provide assurance that financial data have been
auditor report on its financial statements? accurately recorded
a. Independent auditors will always detect management
fraud. 20. Which of the following types of auditing is performed most
b. Independent auditors guarantee the accuracy of the commonly by CPA's on a contractual basis?
financial statements. a. Internal auditing
c. The company preparing the statements may have a b. Income tax auditing
vested interest in reporting certain results. c. External auditing
d. An independent audit is designed to search for d. Government auditing
deficiencies in the company's internal controls.
21. Which of the following types of audits is performed to
15. Which of the following statements does not describe a determine whether an entity's financial statements are fairly
condition that creates a demand for auditing? stated in conformity with generally accepted accounting
a. Users can directly assess the quality of information. principles?
b. Information can have substantial economic a. Operational audit
consequences for a decision maker. b. Compliance audit
c. Expertise is often required for information preparation c. Performance audit
and verification. d. Financial statement audit
d. Conflict between an information preparer and a user can
result in biased information. 22. Which of the following has the primary responsibility for the
fairness of the representations made in the financial statements?
16. Information risk refers to the risk that a. Audit committee
a. The client may not be able to remain in business. b. Client's management
b. Errors and frauds would not be detected by the auditor's c. Independent auditor
procedures. d. Board of Accountancy

AT – Prelim Rev (875 MCQs) Red Sirug Page 79 of 85


c. Fatigue and human weaknesses can cause
23. The best statement of the responsibility of the auditor with auditors to overlook pertinent evidence.
respect to audited financial statement is: d. Many financial statement assertions cannot be audited.
a. The auditor's responsibility on fair presentation of
financial statements is limited only up to the date 28. Financial statements audits:
of the audit report. a. Reduce the cost of capital
b. The auditor's responsibility is confined to the b. Report on compliance with laws and regulations
expression of opinion on the financial statements c. Assess management's efficiency
audited. d. Overlook information risk
c. The responsibility over the financial statements rests with
the management and the auditor assumes responsibility 29. In relation to auditing, which of the following is an incorrect
with respect to the notes of financial statements. phrase?
d. The auditor is responsible only to his qualified opinion but a. Auditing is a systematic process.
not for any other type of opinion. b. Auditing subjectively obtains and evaluates evidence.
c. Auditing evaluates evidence regarding assertions.
24. The level of assurance provided by a professional d. Auditing communicates results to interested users.
accountant on an audit report is:
a. Low 30. Broadly defined, the subject matter of any audit consists
b. Reasonable of
c. Moderate a. Economic data
d. None b. Financial statements
c. Assertions about economic actions and events
25. Reasonable assurance means: d. Operating data
a. Gathering of all available corroborating evidence for the
auditor to conclude that there are no material 31. Auditing is based on the assumption that financial data and
misstatements in the financial statements, taken as a statements are
whole. a. Verifiable
b. Gathering of the audit evidence necessary for the auditor b. In conformity with the identified applicable financial
to conclude that the financial statements, taken as a reporting framework
whole, are free from any misstatements. c. Presented fairly
c. Gathering of the audit evidence necessary for the auditor d. Consistently applied
to conclude that the financial statements are free of
material unintentional misstatements. 32. The firm should establish a system of quality control designed
d. Gathering of the audit evidence necessary for the to provide it with reasonable assurance regarding:
auditor to conclude that there are no material a. Compliance with professional standards
misstatements in the financial statements, taken as a b. Compliance with regulatory and legal requirements
whole. c. Appropriateness of reports issued by the firm or
engagement partners
26. Which of the following is one of the limitations of an audit? d. All of the above
a. Nature of evidence obtained
b. Inadequacy of the accounting records 33. A process designed to provide an objective evaluation, before
c. Confidentiality of information the report is issued, of the significant judgments the
d. Scope limitations imposed by the entity engagement team made and the conclusions they reached in
formulating the report.
27. Which of the following statements does not properly a. Peer review
describe a limitation of an audit? b. Inspection
a. Many audit conclusions are made on the basis of c. Engagement quality control review
examining a sample of evidence. d. Monitoring
b. The work, under taken by the auditor is permeated by
judgment.

AT – Prelim Rev (875 MCQs) Red Sirug Page 80 of 85


34. A process comprising an ongoing consideration and c. PICPA
evaluation of the firm's system of quality control, including a d. ACPAPP
periodic inspection of a selection of completed engagements,
designed to enable the firm to obtain reasonable assurance 40. The chairman and the members of the AASC and FRSC shall
that its system of quality control is operating effectively. have a term of
a. Inspection a. 1 year.
b. Monitoring b. 3 years.
c. Peer review c. 3 years with no renewal.
d. Engagement quality control review d. 3 years renewable for another term.

35. The firm should obtain written confirmation of compliance with 41. The benefits of an operational audit generally include all of
its policies and procedures on independence from all firm the following except
personnel required to be independent by the Code of Ethics at a. Decreased costs.
least: b. Increased revenue
a. Annually c. Increased productivity.
b. Every 3 years d. Increased reliability of the financial statements.
c. Every 2 years
d. Every 5 years 42. The auditor is required to comply with all PSAs relevant to the
audit of an entity's financial statements. A PSA is relevant to
36. The firm should obtain acceptance and continuance the audit when:
information as it considers necessary in the following I. The PSA is in effect.
circumstances: II. The circumstances addressed by the PSA exist.
a. When accepting an engagement with a new client a. I only
b. When deciding whether to continue an existing b. II only
engagement c. Either I or II
c. When accepting a new engagement with an existing d. Both I and II
client
d. All of these. 43. Preplanning the audit involves several key activities which
include the following except:
37. The following methods are most likely to develop capabilities a. Investigating the client's background.
and competence, except: b. Communicating with the prospective client's prior auditor
a. Professional education. to inquire about any disagreements with the client.
b. Continuing professional development, including training. c. Understanding the client's reasons for obtaining an audit.
c. Work experience and coaching by less experienced d. Determining the likelihood of issuing an unqualified
staff audit opinion on the client's financial statements.
d. Self-study modules on professional accounting and
auditing literature. 44. Which of the following is incorrect regarding professional
competence?
38. Consultation is usually undertaken for: a. Professional accountants may portray themselves as
a. Difficult or contentious matters. having expertise or experience they do not possess.
b. Matters for which there is readily available support. b. Professional competence may be divided into two
c. Matters for which the judgment of the engagement leader separate phases.
is deemed sufficient. c. The attainment of professional competence requires
d. Straightforward matters involving accounting and auditing. initially a high standard of general education.
d. The maintenance of professional competence requires a
39. The standard-setting body whose chairman had been or continuing awareness of development in the accountancy
presently a senior accounting practitioner in any scope of profession.
accounting practice is
a. AASC 45. The following procedures relate to Skills and Competence,
b. FRSC except

AT – Prelim Rev (875 MCQs) Red Sirug Page 81 of 85


a. Identify criteria which will be considered in evaluating
individual performance and expected proficiency. 50. Which of the following is not one of the duties of the
b. Provide procedures for maintaining the firm's Commission on Audit
standards of quality for the work performed. a. Keep the general accounts of the government
c. Establish qualifications and guidelines for evaluating b. Promulgate accounting rules and regulations
potential hirees at each professional level. c. Define the scope of its audit and examination
d. Provide, to the extent necessary, programs to fill the d. Assume fiscal responsibility for the government and
firm's needs for personnel with expertise in specialized its instrumentalities
and industries.
51. The early appointment of the independent auditor has many
46. It is essential that users regard CPA firms as advantages to the auditor and the client. Which of the
a. Competent. following advantages is least likely to occur as a result of early
b. Unbiased. appointment of the auditor?
c. Technically proficient. a. The auditor will be able to plan the audit work so that it
d. All of the above may be done expeditiously.
b. The auditor will be able to complete the audit work in
47. The Code of Professional Ethics states, in part, that a CPA less time.
should maintain integrity and objectivity. Objectivity refers to c. The auditor will be able to better plan for the observation
the CPA's ability to of the physical inventories.
a. Determine accounting practices that were consistently d. The auditor will be able to perform the examination more
applied. efficiently and will be finished at an early date after the
b. Maintain an impartial attitude on all matters which year-end.
come under his review.
c. Determine the materiality of items. 52. Which of the following is the most likely first step an auditor
d. Insist on all matters regarding audit procedures. would perform at the beginning of an initial audit engagement?
a. Prepare a rough draft of the financial statements and of
48. A professional accountant may be associated with a tax return the auditor's report.
that b. Study and evaluate the system of internal administrative
a. Contains a false or misleading statement. control.
b. Contains statements or information furnished recklessly c. Tour the client's facilities and review the general
or without any real knowledge of whether they are true or records.
false. d. Consult with and review the work of the predecessor
c. Omits or obscures information required to be submitted auditor prior to discussing the engagement with the client
and such omission or obscurity would mislead the management.
revenue authorities.
d. Uses of estimates if such use is generally acceptable 53. An extensive understanding of the client's business and
or if it is impractical under the circumstances to industry and knowledge about the company's operations are
obtain exact data. essential for doing an adequate audit. For a new client, most
of this information is obtained
49. Prior to beginning the field work on a new audit engagement in a. From the Securities and Exchange Commission.
which a CPA does not possess expertise in the industry in b. From the predecessor auditor.
which the client operates, the CPA should c. From the permanent file.
a. Reduce audit risk by lowering the preliminary levels of d. At the client's premises.
materiality.
b. Design special substantive tests to compensate for the 54. When the continuing auditor intends to use information about
lack of industry expertise. the entity and its environment obtained in prior periods, the
c. Engage financial experts familiar with the nature of the auditor should:
industry. a. Assess control risk as "high" for the assertions affected
d. Obtain a knowledge of matters that relates to the by the prior-period information.
nature of the entity's business.

AT – Prelim Rev (875 MCQs) Red Sirug Page 82 of 85


b. Seek permission with the client in using the prior period c. Having an appropriate a system of quality control.
information obtained by the auditor. d. Continuing professional education.
c. Determine whether changes have occurred that may
affect the relevance of such information in the 60. At the beginning of the current audit engagement, the auditor
current audit. should perform key preplanning activities which include all of
d. Determine whether to equitably reduce the audit fee due the following except:
to lower audit effort expended during the engagement. a. Performing procedures regarding client acceptance or
continuance.
55. Relevant industry conditions include the following, except: b. Evaluating compliance with ethical requirements.
a. The market and competition, including demand, capacity c. Establishing an understanding of the terms of the
and price competition. engagement.
b. Regulatory framework for a regulated industry. d. Considering internal control.
c. Cyclical or seasonal activity.
d. Product technology relating to the entity's products. 61. When a CPA is approached to perform an audit for the first
time, the CPA should make inquiries of the predecessor
56. An auditor obtains an understanding of the entity and its auditor. This is a necessary procedure because the
environment in order to predecessor may be able to provide the successor with
a. Make constructive suggestions concerning improvements information that will assist the successor in determining
to the client's internal control. a. Whether, in the predecessor's opinion, internal control of
b. Understand the events and transactions that may the company has been satisfactory.
have an effect on the client's financial statements. b. Whether the engagement should be accepted.
c. Evaluate whether the aggregation of known c. Whether the predecessor's work should be utilized.
misstatements causes the financial statements taken as a d. Whether the company follows the policy of rotating its
whole to be materially misstated. auditors.
d. Develop an attitude of professional skepticism concerning
management's financial statement assertions. 62. Which of the following will an auditor least likely discuss with
the former auditors of a potential client prior to acceptance?
57. Maintaining or providing access to adequate reference a. Disagreements with management regarding accounting
libraries and other authoritative sources is a procedure that is principles
most likely performed to comply with the policy of b. Integrity of management
a. Monitoring c. Fees charged for services
b. Ethical requirements d. Reasons for changing audit firms
c. Assignment
d. Consultation 63. Which of the following statements are factors to consider
whether the firm has the capabilities, competence, time and
58. A process designed to provide an objective evaluation, before resources to undertake a new audit engagement from a
the report is issued, of the significant judgments the prospective client:
engagement team made and the conclusions they reached in I. Firm personnel have knowledge of relevant
formulating the report is called: industries.
a. Inspection II. The firm has sufficient personnel with the necessary
b. Documentation capabilities and competence.
c. Monitoring III. Experts are available, if needed.
d. Engagement quality control review IV. The firm is able to complete the engagement within
the reporting deadline.
59. A CPA firm would be reasonably assured of meeting its a. I and II only
responsibility to provide professional services that conform b. II and IV only.
with professional standards by: c. II, III and IV only
a. Maintaining an attitude of independence in its d. I, II, III and IV
engagements.
b. Adherence to generally accepted auditing standards.

AT – Prelim Rev (875 MCQs) Red Sirug Page 83 of 85


64. The element of the audit planning process most likely to be 68. Which of the following is an element of "directing an audit
agreed upon with the client before implementation of the audit assistant" objective?
strategy is the determination of the a. Identifying in advance the staffing requirements of a
a. Methods of statistical sampling to be used in confirming particular audit engagement.
accounts receivable. b. Resolving any differences in professional judgment
b. Pending legal matters to be included in the inquiry of the between audit personnel.
client's attorney. c. Resolution of differences in audit findings.
c. Evidence to be gathered to provide a sufficient basis for d. Informing assistants of their responsibilities and the
the auditor's opinion. objectives of the procedures they are to perform.
d. Schedules and analyses to be prepared by the
client's staff. 69. A CPA establishes quality control policies and procedures for
deciding whether to accept a new client or continue to perform
65. The understanding between the client and the auditor as to services for a current client. The primary purpose for
the degree of responsibilities to be assumed by each is establishing such policies and procedures is
normally set forth in a(an) a. To enable the auditor to attest to the integrity or reliability
a. Management letter. of a client.
b. Representation letter. b. To comply with the quality control standards established
c. Engagement letter. by regulatory bodies.
d. Comfort letter. c. To minimize the likelihood of association with
clients whose management lacks integrity.
66. Which of the following best describes the purpose of the d. To lessen the exposure to litigation resulting from failure
engagement letter? to detect irregularities in client financial statements.
a. The engagement letter conveys to management the
detailed steps to be applied in the audit process. 70. Which of the following is prohibited by the Code of
b. The engagement letter relieves the auditor of some Professional Ethics for CPAs?
responsibility for the exercise of due care. a. Use of a firm name which includes the name of a retired
c. By clearly defining the nature of the engagement, the partner.
engagement letter helps avoid and resolve b. Accepting an engagement or employment which one
misunderstandings between CPA and client cannot reasonably expect to complete or discharge
regarding the precise nature of the work to be with professional competence.
performed and the type of report to be issued. c. Announcement in a newspaper of the opening of a public
d. The engagement letter should be signed by both the accounting office.
client and the CPA and should be used only for d. Engaging in civic activities during business hours.
independent audits.
71. Which of the following statements best describes why the
67. Which of the following statements would least likely appear in profession of certified public accountants has deemed it
an auditor's engagement letter? essential to promulgate a code of ethics and to establish a
a. Our engagement is subject to the risk that material errors, mechanism for enforcing observance of the code?
fraud, and defalcations, if they exist, will not be detected. a. The law requires an establishment of a code of ethics.
b. During the course of our audit we may observe b. A distinguishing mark of a profession is its
opportunities for economy in, or improved controls over, acceptance of responsibility to the public.
your operations. c. A prerequisite to success is the establishment of an
c. After performing our preliminary analytical ethical code that stresses primarily the professional's
procedures we will discuss with you the other responsibility to clients and colleagues.
procedures we consider necessary to complete the d. An essential means of self-protection for the profession is
engagement. the establishment of flexible ethical standards by the
d. Fees for our services are based on our regular per diem profession.
rates, plus travel and other out-of-pocket expenses.
72. Which of the following is not one of the qualifications of the
members of the Board of Accountancy?

AT – Prelim Rev (875 MCQs) Red Sirug Page 84 of 85


a. He/She must not be a director or officer of PICPA at the Accountancy for negligence, incompetence, or any other
time of his appointment. just cause.
b. He/She must be a natural-born citizen and a resident of c. A person shall be considered to be in the
the Philippines. professional practice of accounting if, as an officer in
c. He/She must be a duly Certified Public Accountant a private enterprise, he makes decisions requiring
with at least ten years of experience in practice of professional accounting knowledge.
public accountancy. d. After three years, subject to certain conditions, the Board
d. He/She must not have any direct or indirect pecuniary of Accountancy may order the reinstatement of a CPA
interest in any school, college, university, or institution whose certificate of registration has been revoked.
offering a BS Accountancy course or institution
conducting review classes in preparation for the licensure
examination at the time of his appointment to the Board.

73. The following statements relate to the RA 9298. Which


statement is true?
a. The Professional Regulation Commission has the
authority to remove any member of the Board of
Accountancy for negligence, incompetence, or any other
just cause.
b. No person shall be appointed as a member of the
Board of Accountancy unless he has been in the
practice of accountancy for at least 10 years, among
others.
c. Insanity is not a ground for proceeding against a CPA.
d. After three years, subject to certain conditions, the Board
of Accountancy may order the reinstatement of a CPA
whose certificate of registration has been revoked.

74. Special/temporary permit may be issued by the Board to the


following persons except:
a. A foreign CPA engaged as a professor or lecturer in the
fields essential to accountancy education in the
Philippines.
b. A foreign CPA called for consultation or specific purpose
which is essential for the development of the country and
that there are no Filipino CPAs qualified for such
consultation or specific purpose.
c. A foreign CPA who can prove that the country of
which he or she is a citizen admits citizens of the
Philippines to the practice of the same profession
without restriction.
d. A foreign CPA with specialization in any branch of
accountancy and his/her service is essential for the
advancement of accountancy in the Philippines.

75. The following statements relate to RA 9298. Which statement


is true?
a. Insanity is not a ground for proceeding against a CPA.
b. The Professional Regulation Commission has the
authority to remove any member of the Board of

AT – Prelim Rev (875 MCQs) Red Sirug Page 85 of 85

You might also like